Anda di halaman 1dari 286

6 th

Edition

GMAT Data Sufficiency Guide


Joern Meissner

TURBOCHARGE
GMAT and GMAT CAT are registered trademarks
YOUR PREP
of the Graduate Management Admission Council (GMAC).
GMAC does not endorse nor is it affiliated in any way with
the owner of this product or any content herein.

+1 (212) 316-2000 www.manhattanreview.com


Turbocharge your GMAT:
Data Sufficiency Guide
part of the 6th Edition Series
April 20th, 2016

 Most comprehensive book explaining the


concepts of Data Sufficiency
 Emphasis on getting answers without even
solving questions: Conceptual Approach
 Seven common mistakes & exceptions
 Strategies to solve Data Sufficiency ques-
tions
 150 GMAT-like Data Sufficiency questions
Great collection of 700+ level questions
Ample questions with Alternate Ap-
proaches
 A special section: Hints
(A minimum help for every question)

www.manhattanreview.com
19992016 Manhattan Review. All Rights Reserved.
ii Data Sufficiency Guide

Copyright and Terms of Use

Copyright and Trademark


All materials herein (including names, terms, trademarks, designs, images, and graphics) are
the property of Manhattan Review, except where otherwise noted. Except as permitted herein,
no such material may be copied, reproduced, displayed or transmitted or otherwise used with-
out the prior written permission of Manhattan Review. You are permitted to use material
herein for your personal, noncommercial use, provided that you do not combine such material
into a combination, collection, or compilation of material. If you have any questions regarding
the use of the material, please contact Manhattan Review at info@manhattanreview.com.

This material may make reference to countries and persons. The use of such references is for
hypothetical and demonstrative purposes only.

Terms of Use
By using this material, you acknowledge and agree to the terms of use contained herein.

No Warranties
This material is provided without warranty, either express or implied, including the implied
warranties of merchantability, of fitness for a particular purpose and noninfringement. Man-
hattan Review does not warrant or make any representations regarding the use, accuracy or
results of the use of this material. This material may make reference to other source materials.
Manhattan Review is not responsible in any respect for the content of such other source ma-
terials, and disclaims all warranties and liabilities with respect to the other source materials.

Limitation on Liability
Manhattan Review shall not be responsible under any circumstances for any direct, indirect,
special, punitive, or consequential damages (Damages) that may arise from the use of this
material. In addition, Manhattan Review does not guarantee the accuracy or completeness of
its course materials, which are provided as is with no warranty, express or implied. Man-
hattan Review assumes no liability for any Damages from errors or omissions in the material,
whether arising in contract, tort or otherwise.

GMAT is a registered trademark of the Graduate Management Admission Council.


GMAC does not endorse, nor is it affiliated in any way with, the owner of this product or any
content herein.
10-Digit International Standard Book Number: (ISBN: 1-62926-065-7)
13-Digit International Standard Book Number: (ISBN: 978-1-62926-065-5)

Last updated on April 20th, 2016.

Manhattan Review, 275 Madison Avenue, Suite 1429, New York, NY 10016.
Phone: +1 (212) 316-2000. E-Mail: info@manhattanreview.com. Web: www.manhattanreview.com

www.manhattanreview.com 19992016 Manhattan Review


Data Sufficiency Guide iii

About the Turbocharge your GMAT Series

The Turbocharge Your GMAT Series is carefully designed to be clear, comprehensive, and
content-driven. Long regarded as the gold standard in GMAT prep worldwide, Manhattan Re-
views GMAT prep books offer professional GMAT instruction for dramatic score improvement.
Now in its updated 6th edition, the full series is designed to provide GMAT test-takers with
complete guidance for highly successful outcomes. As many students have discovered, Man-
hattan Reviews GMAT books break down the different test sections in a coherent, concise,
and accessible manner. We delve deeply into the content of every single testing area and zero
in on exactly what you need to know to raise your score. The full series is comprised of 16
guides that cover concepts in mathematics and grammar from the most basic through the
most advanced levels, making them a great study resource for all stages of GMAT preparation.
Students who work through all of our books benefit from a substantial boost to their GMAT
knowledge and develop a thorough and strategic approach to taking the GMAT.

 GMAT Math Essentials (ISBN: 978-1-62926-057-0)


 GMAT Number Properties Guide (ISBN: 978-1-62926-058-7)
 GMAT Arithmetics Guide (ISBN: 978-1-62926-059-4)
 GMAT Algebra Guide (ISBN: 978-1-62926-060-0)
 GMAT Geometry Guide (ISBN: 978-1-62926-061-7)
 GMAT Word Problems Guide (ISBN: 978-1-62926-062-4)
 GMAT Sets & Statistics Guide (ISBN: 978-1-62926-063-1)
 GMAT Combinatorics & Probability Guide (ISBN: 978-1-62926-064-8)
 GMAT Data Sufficiency Guide (ISBN: 978-1-62926-065-5)
 GMAT Quantitative Question Bank (ISBN: 978-1-62926-066-2)
 GMAT Sentence Correction Guide (ISBN: 978-1-62926-067-9)
 GMAT Critical Reasoning Guide (ISBN: 978-1-62926-068-6)
 GMAT Reading Comprehension Guide (ISBN: 978-1-62926-069-3)
 GMAT Integrated Reasoning Guide (ISBN: 978-1-62926-070-9)
 GMAT Analytical Writing Guide (ISBN: 978-1-62926-071-6)
 GMAT Vocabulary Builder (ISBN: 978-1-62926-072-3)

19992016 Manhattan Review www.manhattanreview.com


iv Data Sufficiency Guide

About the Company


Manhattan Reviews origin can be traced directly back to an Ivy League MBA classroom in 1999.
While teaching advanced quantitative subjects to MBAs at Columbia Business School in New
York City, Professor Dr. Joern Meissner developed a reputation for explaining complicated
concepts in an understandable way. Remembering their own less-than-optimal experiences
preparing for the GMAT, Prof. Meissners students challenged him to assist their friends, who
were frustrated with conventional GMAT preparation options. In response, Prof. Meissner
created original lectures that focused on presenting GMAT content in a simplified and intel-
ligible manner, a method vastly different from the voluminous memorization and so-called
tricks commonly offered by others. The new approach immediately proved highly popular
with GMAT students, inspiring the birth of Manhattan Review.

Since its founding, Manhattan Review has grown into a multi-national educational services
firm, focusing on GMAT preparation, MBA admissions consulting, and application advisory
services, with thousands of highly satisfied students all over the world. The original lectures
have been continuously expanded and updated by the Manhattan Review team, an enthusiastic
group of master GMAT professionals and senior academics. Our team ensures that Manhattan
Review offers the most time-efficient and cost-effective preparation available for the GMAT.
Please visit www.ManhattanReview.com for further details.

About the Founder


Professor Dr. Joern Meissner has more than 25 years of teaching experience at the graduate
and undergraduate levels. He is the founder of Manhattan Review, a worldwide leader in test
prep services, and he created the original lectures for its first GMAT preparation class. Prof.
Meissner is a graduate of Columbia Business School in New York City, where he received a PhD
in Management Science. He has since served on the faculties of prestigious business schools
in the United Kingdom and Germany. He is a recognized authority in the areas of supply chain
management, logistics, and pricing strategy. Prof. Meissner thoroughly enjoys his research,
but he believes that grasping an idea is only half of the fun. Conveying knowledge to others
is even more fulfilling. This philosophy was crucial to the establishment of Manhattan Review,
and remains its most cherished principle.

www.manhattanreview.com 19992016 Manhattan Review


Data Sufficiency Guide v

The Advantages of Using Manhattan Review


I Time efficiency and cost effectiveness.
For most people, the most limiting factor of test preparation is time.
It takes significantly more teaching experience to prepare a student in less time.
Our test preparation approach is tailored for busy professionals. We will teach you
what you need to know in the least amount of time.

I Our high-quality and dedicated instructors are committed to helping every student
reach her/his goals.

19992016 Manhattan Review www.manhattanreview.com


vi Data Sufficiency Guide

International Phone Numbers and Official Manhattan Review Websites


Manhattan Headquarters +1-212-316-2000 www.manhattanreview.com
USA & Canada +1-800-246-4600 www.manhattanreview.com
Argentina +1-212-316-2000 www.review.com.ar
Australia +61-3-9001-6618 www.manhattanreview.com
Austria +43-720-115-549 www.review.at
Belgium +32-2-808-5163 www.manhattanreview.be
Brazil +1-212-316-2000 www.manhattanreview.com.br
Chile +1-212-316-2000 www.manhattanreview.cl
China +86-20-2910-1913 www.manhattanreview.cn
Czech Republic +1-212-316-2000 www.review.cz
France +33-1-8488-4204 www.review.fr
Germany +49-89-3803-8856 www.review.de
Greece +1-212-316-2000 www.review.com.gr
Hong Kong +852-5808-2704 www.review.hk
Hungary +1-212-316-2000 www.review.co.hu
India +1-212-316-2000 www.review.in
Indonesia +1-212-316-2000 www.manhattanreview.id
Ireland +1-212-316-2000 www.gmat.ie
Italy +39-06-9338-7617 www.manhattanreview.it
Japan +81-3-4589-5125 www.manhattanreview.jp
Malaysia +1-212-316-2000 www.review.my
Mexico +1-212-316-2000 www.manhattanreview.mx
Netherlands +31-20-808-4399 www.manhattanreview.nl
New Zealand +1-212-316-2000 www.review.co.nz
Philippines +1-212-316-2000 www.review.ph
Poland +1-212-316-2000 www.review.pl
Portugal +1-212-316-2000 www.review.pt
Qatar +1-212-316-2000 www.review.qa
Russia +1-212-316-2000 www.manhattanreview.ru
Singapore +65-3158-2571 www.gmat.sg
South Africa +1-212-316-2000 www.manhattanreview.co.za
South Korea +1-212-316-2000 www.manhattanreview.kr
Sweden +1-212-316-2000 www.gmat.se
Spain +34-911-876-504 www.review.es
Switzerland +41-435-080-991 www.review.ch
Taiwan +1-212-316-2000 www.gmat.tw
Thailand +66-6-0003-5529 www.manhattanreview.com
Turkey +1-212-316-2000 www.review.com.tr
United Arab Emirates +1-212-316-2000 www.manhattanreview.ae
United Kingdom +44-20-7060-9800 www.manhattanreview.co.uk
Rest of World +1-212-316-2000 www.manhattanreview.com

www.manhattanreview.com 19992016 Manhattan Review


Contents

1 Welcome 1

2 Concepts of Data Sufficiency 3


2.1 What is data sufficiency and why is it tested on the GMAT? . . . . . . . . . . . . . . 4
2.2 The idea of data sufficiency . . . . . . . . . . . . . . . . . . . . . . . . . . . . . . . . . . 6
2.3 How is it tested on the GMAT? . . . . . . . . . . . . . . . . . . . . . . . . . . . . . . . . 22
2.4 Two types of DS questions . . . . . . . . . . . . . . . . . . . . . . . . . . . . . . . . . . . 25
2.5 Approach to the DS questions . . . . . . . . . . . . . . . . . . . . . . . . . . . . . . . . . 28
2.5.1 Decision Tree . . . . . . . . . . . . . . . . . . . . . . . . . . . . . . . . . . . . . . 33
2.5.2 Two ways to get a question wrong . . . . . . . . . . . . . . . . . . . . . . . . . . 34
2.6 Seven Common Mistakes . . . . . . . . . . . . . . . . . . . . . . . . . . . . . . . . . . . 35
2.7 Beware of the common exceptions . . . . . . . . . . . . . . . . . . . . . . . . . . . . . . 46
2.8 A couple of strategies for solving DS Questions . . . . . . . . . . . . . . . . . . . . . . 54

3 Practice Questions 61

4 Answer-key 111

5 Hints 115

6 Solutions 127

7 Talk to Us 273

vii
viii Data Sufficiency Guide

www.manhattanreview.com 19992016 Manhattan Review


Chapter 1

Welcome

Dear Students,
Here at Manhattan Review, we constantly strive to provide you the best educational content
for standardized test preparation. We make a tremendous effort to keep making things better
and better for you. This is especially important with respect to an examination such as the
GMAT. A typical GMAT aspirant is confused with so many test-prep options available. Your
challenge is to choose a book or a tutor that prepares you for attaining your goal. We cannot
say that we are one of the best, it is you who has to be the judge.

There is hardly any book dedicated solely on Data Sufficiency. This book ties to decode the
enigma called: Data Sufficiency. The book devotes a lot of scope for understanding the logic
behind Data Sufficiency; how it is tested on the GMAT; Seven Common Mistakes & Exceptions;
and Strategies for solving DS questions. Conceptual approach, developed to hone an under-
stating that many problems can be sorted even without solving them, is the sole of the book.

You will find a lot of variety in the problems discussed. Alternate approaches to few tricky
questions are worth appreciating. You will find many 700+ level of questions in the book.

The book provides a special section called Hints. Hints serve great purpose for those students
who though could not solve question correctly, do not wish to see solution straightaway. They
want a minimum help for the question. Hint for a question is one to three lines generic or
specific statements.

The Manhattan Reviews Data Sufficiency book is holistic and comprehensive in all respects.
Should you have any queries, please feel free to write to me at info@manhattanreview.com.

Happy Learning!

Professor Dr. Joern Meissner


& The Manhattan Review Team

1
2 Concepts of Data Sufficiency

www.manhattanreview.com 19992016 Manhattan Review


Chapter 2

Concepts of Data Sufficiency

3
4 Concepts of Data Sufficiency

2.1 What is data sufficiency and why is it tested on the GMAT?


Suppose you want to go from City X to City Y and want to take the cheapest route to reach
City Y. What all information do you need?

Youd need the cost of taking each route to reach City Y from City X so that you can see which
route costs the minimum. Right?

Now, if you are provided with this information, would you need any other information to de-
termine the cheapest route from City X to City Y?

No, you wouldnt need any other information. The information that you have been provided
is enough for your purpose. In other words, we can say that this information is sufficient to
solve your problem.

Now, on the other hand, if instead of the cost of each route, you are told the length of road in
miles on each route. Such an information will help you in determining the cheapest route since
the fuel costs are dependent on the length of the road. However, is this information enough
or sufficient?

The answer is No. Its not only the length of the road that determines the cost of a route but
also other factors such as whether its light traffic or heavy traffic road (heavier the traffic,
lesser the mileage, and thus, more the cost), whether there are any tolls on the road etc.

So, we can say that the knowledge of the length of the road on each route is not sufficient to
solve your problem.

Now, this is what data sufficiency is all about. That is, determining which data is sufficient to
solve a problem and which data is not.

But one may ask, Ok. This is all fine. But why is it tested on the GMAT?

To answer the question, we need to just look back at the purpose of GMAT. GMAT is an en-
trance test for MBA programs, and thus, it tests a candidates ability to do well in an MBA
program and later in his/her management career. So, data sufficiency has got to do with some-
thing that determines your success in your management career.

Now, in a managerial position, youll be required to undertake decisions on a regular basis.


How will take decisions? On the basis of some data or information. Right?

Now, what all information would you require to take any decision? Would you gather all the
information that is available in the market regarding your area of concern?

No. It wouldnt be the right way. Not only is information costly but also its analysis time-
consuming. So, if you are looking at unnecessary information, you are not only wasting
companys financial resources in buying that information but also wasting your and others
precious time in analyzing that information. Therefore, the ideal way is not to seek all the
information out there but to seek only that much information which is sufficient for your

www.manhattanreview.com 19992016 Manhattan Review


Concepts of Data Sufficiency 5

purpose. Isnt it?

Sometimes the information managers seek for are already available, but the form and shape
of the information is such that managers are not able to see that with a little processing, the
available information can be of use. The information disguised and hidden among other infor-
mation sometimes clutters their sight.

Any information or data that is over and above what is sufficient is not required, and as a man-
agement professional, you will be expected to have the wisdom to figure out what constitutes
sufficient information.

And so that is the reason why it is tested on the GMAT.

Now, if you pay attention, you can see that this business of identifying sufficient information
requires you to have complete understanding of what you want and how you can have it. Now,
what you want is a solution to your problem, and unless you are very clear about your prob-
lem, you cannot find a way to solve it. How you can have it is through the information or
data you need to solve the problem. But there may be multiple ways in which a problem can be
solved, and thus, you can arrive at a solution by using different kinds of information. A very
simple example to demonstrate this is: If you want to calculate the number of goals scored by
a team in a match, you can calculate the number either if you are given the number of goals
scored by each team member in the match or if you are given the number of goals scored by
the opponent team, the difference in the numbers of goals of the two teams, and who won the
match. Isnt it? Each of these data sets would be sufficient to solve your problem i.e. calculate
the number of goals by the team.

Therefore, in data sufficiency, you need to keep your horizons wide to entertain possibilities
to solve a problem other than you originally thought. Thus, this section on GMAT requires one
to look at a problem from multiple perspectives.

19992016 Manhattan Review www.manhattanreview.com


6 Concepts of Data Sufficiency

2.2 The idea of data sufficiency


In this section, well build upon the idea of data sufficiency discussed in the previous section.
Lets say you want to calculate the average score of the three students in a particular exam.

Q: What is the average score of the three students: Tom, Jim, and Harry in the exam?

Now, if you are told the individual scores of the three students, will you be able to answer the
above question?

(1) The scores of Tom, Jim, and Harry in the exam are 68, 72, and 70 respectively.

Is the above statement sufficient to answer the question?

Yes. Because once you know the individual scores of the three students, you can easily
calculate their average score by adding up the scores and then dividing the sum by 3.

However and this is very important when you are asked whether the given statement
is sufficient to answer the question, you dont need to calculate the average score. You
just need to figure out whether the given information is sufficient or not sufficient to
answer the given question i.e. calculate the average score.

Now, lets say you are given the following statement:

(2) The sum of the scores of Tom, Jim, and Harry in the exam is 210.

Is the above statement sufficient to answer the question?

Again the answer is Yes, since once you know the sum of the scores of the three students,
you can divide the sum by 3 to get the average score. So, with the information given in
the statement, you can answer the original question posed. Again, the point to empha-
size here is that you do not need to divide 210 by 3 and calculate the average score of
the three students. You just need to figure out whether you can calculate the average
score or not. In this case, since you can calculate the average score using the informa-
tion in the statement, you can say that the statement is sufficient to answer the question.

Lets consider another statement.

www.manhattanreview.com 19992016 Manhattan Review


Concepts of Data Sufficiency 7

(3) Out of Tom, Jim, and Harry, Tom scored the lowest with 68 and Jim scored the highest
with 72.

Is the above statement sufficient to answer the question?

The answer is No. Why? Because in this case, we do not know the score of Harry. Given
this statement, we know that Harry could have scored 69, 70, or 71. Now, for each of
these possibilities, the average score of the three students will be different. (We do not
need to calculate and check that the score will be different in these three possibilities. We
can conceptually understand that if one of the scores changes, the average score should
also change.) So, all we can say, using this statement, is that the average score of the
three students will be one of the three numbers (corresponding to three possibilities).
We CANNOT find a UNIQUE value for the average score of the three students. Therefore,
since the original question demanded a unique value for the average score of the three
students, we CANNOT answer the original question using this statement. Thus, in other
words, we can say that this statement is NOT sufficient to answer the question.

Now, if our original question had been:

Q: What is the range of scores scored by Tom, Jim, and Harry?

Which of our three statements would be sufficient to answer this question?

(1) The scores of Tom, Jim, and Harry in the exam are 68, 72, and 70 respectively.

(2) The sum of the scores of Tom, Jim, and Harry in the exam is 210.

(3) Out of Tom, Jim, and Harry, Tom scored the lowest score with 68 and Jim scored the
highest score with 72.

To calculate the range, which is the difference between the highest score and the lowest score,
we just need the highest and the lowest scores. We can see that while statements 1 and 3
provide the highest and the lowest scores, Statement 2 does not. Statement 2 just provides
the sum of the scores of the three students. In this case, all three of them could have scored 70
each, in which case the range will be 0 (Zero), or their scores could be 40, 70, and 100, in which
case their range will be 60. So, we can see that by using Statement 2, we cannot determine a
unique value for the range of score of the three students.

Therefore, we can say the following:

(1) Statement 1 is sufficient to answer the question

19992016 Manhattan Review www.manhattanreview.com


8 Concepts of Data Sufficiency

(2) Statement 2 is not sufficient to answer the question

(3) Statement 3 is sufficient to answer the question

Lets take another question.

Q: Among Tom, Jim, and Harry, did Tom score the highest in the exam?

Which of our three statements would be sufficient to answer this question?

(1) The scores of Tom, Jim, and Harry in the exam are 68, 72, and 70 respectively.

(2) The sum of the scores of Tom, Jim, and Harry in the exam is 210.

(3) Out of Tom, Jim, and Harry, Tom scored the lowest score with 68 and Jim scored the
highest score with 72.

Statement 1:

Is statement 1 sufficient to answer the question?

If you are thinking No, you are making a common mistake. Statement 1 is indeed sufficient
to answer the question. Using statement 1, we can confidently say that Tom did not score the
highest in the exam. So, Statement 1 is enough or sufficient to answer the given question.
Now, in this case, the answer that comes out using the statement is No. Tom did not score
the highest. However, our concern here is not about the actual answer to the question but
whether the question can be answered or not. Since the question can be answered, we know
that the statement is sufficient to answer the question.

Statement 2:

Is statement 2 sufficient to answer the question?

No. Statement 2 is not sufficient to answer the question since Statement 2 does not tell us
about the individual scores. Tom could have scored the highest or the lowest among the three
students. We cannot say anything based on Statement 2.

Statement 3:

Is statement 3 sufficient to answer the question?

Like Statement 1, Statement 3 is sufficient to answer the question. Using Statement 3, we can
conclusively say that Tom did not score the highest among the three students. Therefore, we
can provide a unique value (No) to our original question using this statement.

www.manhattanreview.com 19992016 Manhattan Review


Concepts of Data Sufficiency 9

Practice Drill
Now, lets do a few more questions to solidify the understanding gained so far. Please try to
solve each question yourself before you look at the solution. From our experience, one gains
much more from his or her mistakes than from a laid out question and solution.

Each drill below will consist of a question followed by several individual statements. Your
job is to determine which of the statements are sufficient to answer the question and which
statements are not. Please note that multiple statements may be sufficient to answer the given
question.

(1) If x is an integer, what is the value of x?


x
1. =3
2
2. x 2 = 1
3. x > 0
4. 3 < x < 5
5. (x 1) (x 1.5) = 0

Solution:

In this question, we are asked to find the value of x. However, we are also given some
information about x. The information is that x is an integer. So, while evaluating the
sufficiency of the statements, we can use this information.
x
1. Sufficient. If we solve this equation = 3, we get x = 6, which is an integer. So,
2
we can see that this statement is sufficient to answer the question. However, please
note that it was not required of us to calculate the value of x. We calculated the
value of x because it was easy and sort of directly jumping at us. However, in case
of a bit more complex equation, if you are sure that the equation will give you a
unique value of x, you should skip the calculations altogether.

2. Not sufficient. This statement is not sufficient to answer the question since there
are two unique solutions to this quadratic equation. x could be equal to 1 or 1.
Now, we cannot answer the question saying The value of x is either 1 or 1. When
we are asked the value of a variable, we need to provide a UNIQUE value of the same.
Since, in this case, we cannot provide a unique value of x, we say that the statement
is not sufficient to answer the question.

3. Not sufficient. Clearly, just knowing that x > 0 cannot help us provide a unique
value of x. x could be 1, 2, 3, and so on. However, x cannot be 0.5, 0.75 or 3.12.
Why? Because we are already given in the question that x is an integer.

4. Sufficient. However, anyone who didnt pay much attention to the question would
get this wrong. If we just know that x is a number and that it lies between 3 and

19992016 Manhattan Review www.manhattanreview.com


10 Concepts of Data Sufficiency

5, we cannot provide a unique value of x. x could be 3.01, 3.34, or any number


between 3 and 5. And thus, one would consider this statement insufficient.

However, we are specifically told in the question that x is an integer. Now, once we
know that 3 < x < 5, we can confidently say that x can only be equal to 4 because
there is only integer that satisfies this inequality. Since we can provide a unique
value for x using this statement, this statement is sufficient to answer the question.

5. Sufficient. We know that when the product of two expressions is equal to 0, one of
the expressions has to be 0. So, we have

Either (x 1) = 0

Or

(x 1.5) = 0

So, we can see that either x = 1 or x = 1.5. Now, from our question, we know that x is
an integer. Since 1.5 is not an integer, x cannot be equal to 1.5. Therefore, x has to be
equal to 1. So, from this statement, we are able to find a unique value for x. Thus, we
can say that this statement is sufficient to answer the question.

Takeaway: The key takeaway from this drill is to pay attention to the information given
in the question stem. Such information can make or break the case against the sufficiency
of a statement.

(2) Is integer x divisible by 15?

1. x is a product of three consecutive odd integers


2. x = 30y, where y is a real number
3. x 2 8x + 15 = 0
4. 4x is divisible by 30
5. x 2 is a multiple 45

Solution:

This question is different from the first question in that this question doesnt ask us for a
value of x. It just asks whether x is divisible by 15. So, in this drill, we are not concerned
whether the statements can provide a unique value of x. We are only concerned whether
the statements can provide a unique answer either Yes or No to the question asked.
In other words, if the statement restricts x to only those numbers, which are divisible by
15, then the statement is sufficient to answer the question. In such a case, the answer
will be unequivocal YES.

www.manhattanreview.com 19992016 Manhattan Review


Concepts of Data Sufficiency 11

The statement will be sufficient even in the case when it restricts x to only those num-
bers which are not divisible by 15. In such a case, the answer to the question will be
unequivocal NO.

However, the statement will be insufficient when it allows x to have values, some of
which are divisible by 15 and some of which are not. In such a case, well not be able to
answer the question.

1. Insufficient. x could be a product of 1, 3, and 5, in which case x will be a multiple


of 15. However, x could also be a multiple of 7,9, and 11, in which case x will not
be a multiple of 15. Therefore, using this statement, we cannot provide a unique
answer to the question.

1 1
2. Insufficient. We are given that y is a real number. So, y could be 1, , or a
3 10
few more possible values. When y = 1, x = 30, in which case x is divisible by 15.
1
When y = , x = 10, in which case x is not divisible by 15. Therefore, using this
3
statement, we cannot provide a unique value to the question.

1
One point to understand here is that even though is also a real number, we cannot
4
1 15
take y = , since it will give x = , which is not an integer. Since we are given
4 2
that x is an integer in the question stem, we have to restrict ourselves to only those
1
values of y, which provide an integral value of x. Both 1 and provided integral
3
value of x. Therefore, both of these values were valid values of y, and thus, could
be taken as cases to test the sufficiency of the statement.

3. Sufficient. The expression x 2 8x + 15 can be factorized as (x 3) (x 5). So, we


have

(x 3) (x 5) = 0

Now, we know that when the product of two expressions is equal to 0, one of the
expressions has to be 0. So, we have

Either (x 3) = 0

Or

(x 5) = 0

So, we can see that either x = 3 or x = 5. Now, neither 3 nor 5 is divisible by 15, so
we can see that x is never divisible by 15. Therefore, using this statement, we can
unequivocally answer the question. Thus, this statement is sufficient to answer the
question.

19992016 Manhattan Review www.manhattanreview.com


12 Concepts of Data Sufficiency

Now, some of the students who are looking for a unique value of x say that the
statement is insufficient to answer the question. However, as discussed earlier, the
idea is not to find a unique value of x. The idea is to find a unique answer to the
question asked. Since the question is a Yes or No question, we just need to see if
we can provide one of these answers unequivocally. If so, the statement is sufficient.

One point to mention here is that you do not necessarily need to solve the quadratic
equation to find the value of x and then see if it is divisible by x. Another way to
find out if this statement is sufficient to answer the question or not is as below:

We know that for a number to be divisible by 15, it has to be of the form 15m, where
m is an integer. So, these numbers could be 30, 15, 0, 15, 30, and so on.

Now, we can see that for x = 0, the expression x 2 8x + 15 will be equal to 15. Thus
x = 0 cannot be a solution to the given quadratic equation x 2 8x + 15 = 0.

Similarly, for all positive values 15, 30, and so on, the expression x 2 8x + 15 will
be equal to positive numbers and not zero. Therefore, these values also cannot be
the solution to the given quadratic equation.

Similarly, for all negative values of x such as 15, 30, and so on, the expression
x 2 8x + 15 will be equal to positive numbers and not zero. Therefore, these values
also cannot be the solution to the given quadratic equation.

Therefore, we can see that there is no value of the form 15m that can satisfy the
given quadratic equation. So, we can surely say that x, which satisfies the quadratic
equation, cannot be divisible by 15. Therefore, this statement is sufficient to answer
the given question.

So, we can see that we have found out whether the statement is sufficient or not
without solving the quadratic equation or finding the value of x. This is a perfectly
valid way of solving data sufficiency questions. So, keep this in mind as you go about
solving the rest of the questions.

4. Sufficient. 4x is divisible by 30 means the same as 2x is divisible by 15, since 2 is


a common factor between 4x and 30. Now if 2x is divisible by 15, what can we say
about the divisibility of x by 15?

We can see that since 2 and 15 do not have a factor in common, for 2x to be divisible
by 15, x has to be a multiple of 15. Another way to understand it is that if 2x is
divisible by 15, it has to be divisible by both 3 and 5. Now, since 2 doesnt have a
factor common either with 3 or 5 (in other words, 2 is co-prime w.r.t. both 3 and 5),
x has to be a multiple of both 3 and 5.

A number which is a multiple of both 3 and 5 will be a multiple of 15. Therefore, x


is a multiple of 15. In other words, x is always divisible by 15.

www.manhattanreview.com 19992016 Manhattan Review


Concepts of Data Sufficiency 13

Therefore, this statement is sufficient to answer the question.

In this statement too, we have used the information given in the question stem that
x is an integer. If we were not given that x is an integer or if we had not used this
information, this statement would not have turned out to be sufficient.

5. Sufficient. This statement is one of the challenging ones to deal with. We are given
that x 2 is a multiple of 45. So, lets first look at the prime factors of 45. The prime
factors of 45 are 3 and 5. So, clearly x will be a multiple of 3. But will x be a
multiple of 5 too? The answer is yes because x 2 cannot be a multiple of a prime
number unless x too is. Why? Simply because you cannot produce a prime number
by multiplying any other two integers. So, if a prime number exists as a factor of
x 2 , where x is an integer, then that prime number is also a factor of x. Therefore, 5
also is a factor of x.

Now, we know that both 3 and 5 are factors of x, so we can say that x is divisible
by 15. Therefore, the statement is sufficient to answer the question.

Takeaway: There are multiple takeaways from this drill. However, the one that needs
special attention is about finding the relevant cases to prove the insufficiency of a state-
ment. Here, it is important to observe that one can prove the insufficiency of a statement
by taking sample cases but not the sufficiency of a statement. For example: In Statement
1, we took two cases {1, 3, 5} and {7, 9, 11}, and since these two cases produced differ-
ent answers to the question, we concluded that the statement is insufficient to answer the
question. So, if we can find any two scenarios allowed by a statement that produce two
different answer to the question, we can say that the statement is insufficient to provide a
unique answer to the question. However, we cannot prove the sufficiency of a statement
by seeing that two or even more cases produce the same answer. Why? Because there may
exist a case which may not have looked at but which produced a different answer. For
example: if we just looked at the cases {1, 3, 5}, {3, 5, 7}, and {5, 7, 9} for Statement 1
all of which produced the same answer Yes then, we might have called the statement
sufficient. However, as we see, this is not the case. Therefore, by taking different numbers
or cases, you can prove the insufficiency of a statement but not the sufficiency.

(3) In Isosceles triangle PQR, what is the measure of angle R?

1. Angle Q = 30
2. Angle P = 100
3. Angle Q is one-eighth the measure of Angle P
4. Sum of Angle P and Angle R is equal to measure of Angle Q
5. PQ = PR = 2 units, and QR = 3 units

Solution:

19992016 Manhattan Review www.manhattanreview.com


14 Concepts of Data Sufficiency

We are talking about an isosceles triangle PQR. An isosceles triangle is the one in which
two of the three sides are equal and two of the angles (corresponding to equal sides) are
also equal. However, we do not know as of now which sides or angles are equal. We are
asked about the value of angle R. Therefore, any statement that provides a unique value
for angle R is sufficient for our question, and any statement that allows multiple values
or indeterminable values for angle R is not sufficient.

1. Insufficient. Angle Q could be one of the equal angles, in which case the angles
of the triangles will be 30 , 30 , and 120 . In this case, angle R could be equal to
either 30 or 120 . Therefore, we can see that this statement allows two possible
value for angle R. So, this statement is not sufficient to answer the question.

Please note that we need not look into the case in which angle Q would be the un-
equal angle and thus the angles of the triangles will be 30 , 75 , and 75 . The reason
we need not look at this case is that we have already seen that this statement is not
sufficient. Thus, no matter what this case shows us, the statement will remain not
sufficient.

2. Sufficient. In this case, since angle P = 100 , it cannot be one of the equal angles
because in such a case, the sum of the angles of the triangles will exceed 180 .
Therefore, the only case possible in which Angle P = 100 is the one in which the
other two angles are equal. Since we know that sum of the three angles of a triangle
equals 180 , the other two equal angles will be equal to 40 each. Therefore, angle R
will be equal to a unique value i.e. 40 . Thus, this statement is sufficient to answer
the question.

Please note here that even though Statement 1 and Statement 2 seem to provide the
exact same information i.e. the measure of one angle of a triangle, one of them is
sufficient while the other is insufficient. A person who has dealt with Statement 1
is likely to reject Statement 2 as insufficient without much analysis because of the
obvious similarity of the two statements. However, as we have seen, he would be
wrong. So, the learning here is that we should give each statement its due analysis
and not be eager to judge it based on its similarity to other statements. One other
thing that could help us avoid this trap is to focus on the difference between the
two statements and see if the difference could make an impact on the sufficiency
of the statement. For example: In this drill, as soon as an angle becomes equal to
or greater than 90 , it cannot be one of the equal angles, and thus leaves only one
possibility for the remaining angles. We can see that in Statement 1, the angle is
less than 90 , while in Statement 2, the angle is greater than 90 . So, if we have
this understanding clear, we can see that the difference between Statement 1 and
Statement 2 can impact the sufficiency of the statements.

In general, the data sufficiency questions contain such traps, and thus, we should
keep our eyes and minds open to such possibilities and not reject (or accept) state-
ments based on superficial analysis or similarity with other statements.

www.manhattanreview.com 19992016 Manhattan Review


Concepts of Data Sufficiency 15

3. Insufficient. Angle Q could be one of the equal angles, in which case the angles of
the triangles will be 18 , 18 , and 144 . In this case, angle Q is 18 and angle P
(which is 8 times angle Q) is 144 . Angle R is the remaining measure i.e. 18 .

However, angle Q could also be the unequal angle. In such a case, angle P and an-
gle R will be equal. In this case, the measures of the angle of the triangles will be
10 , 80 , and 80 . Angle Q: 10 , Angle P: 80 , and Angle R: 80 .

We can see that in one of the above two cases, angle R is equal to 18 , and in the
other case, angle R is equal to 80 . Therefore, this statement does not provide a
unique value for angle R.

So, this statement is not sufficient to answer the question.

4. Insufficient. Given this statement and the fact that sum of the angles of a triangle is
equal to 180 , we can see that angle Q will be equal to 90 , and sum of angles P and
R will also be equal to 90 . Now, angle R could be any value between 0 and 90 i.e.
0 < R < 90 . Since this statement does not provide us a unique value for angle R,
this statement is not sufficient to answer the question.

5. Sufficient. Using the lengths of the sides given in the statement, we can draw a
unique triangle. It is important to understand here that if are given three sides of
a triangle, only one triangle is possible i.e. only one set of measures of angles is
possible. In other words, if we are given measures of three sides, it is not possible
to create two triangles one with one set of measures of angles and another with a
different set of measures of angles.

Now, once we have our unique triangle drawn, we can find the measure of the angle
R. Therefore, this statement will provide a unique value for the measure of angle R.
Thus, this statement is sufficient to answer the question.

Please note that we do not need to draw the triangle or find the value of angle R.
Once we are sure that a unique value of angle R is possible from this statement, we
can confidently say that this statement is sufficient to answer the question.

(By the way, for people who are curious, the measure of angle R in this case will be
41.41 )

(4) Last year, Joe spent a total of $8000 on his tuition fee, hostel rent, and conveyance. How
much did Joe spend on hostel rent?

1. Joe spent a total of $7000 on tuition fee and hostel rent.


2. Joe spent a total of $6000 on tuition fee and conveyance.
3. Joe spent twice as much on tuition fee as on hostel rent and conveyance combined

19992016 Manhattan Review www.manhattanreview.com


16 Concepts of Data Sufficiency

4. The ratio of the amounts Joe spent on tuition fee, hostel rent, and conveyance is
3:2:1

Solution:

In this drill, unlike the previous three drills, we are not asked directly about the value of
a mathematical variable. Rather, a situation is described. To efficiently solve the prob-
lem, we should create the mathematical equivalent of the situation so that going into the
statements, we are clear about what we want.

Let Joes tuition fee be x, hostel rent be y, and conveyance be z.

We are given that

x + y + z = 8000

We are asked to find the value of y.

Any statement that provides a unique value of y is sufficient to answer the question, and
any statement that does not is not sufficient to answer the question.

1. Insufficient. The statement says

x + y = 7000

So, y could be any value from 0 to 7000.

Since this statement does not provide a unique value for y, it is not sufficient to
answer the question.

2. Sufficient. The statement says

x + z = 6000

Now, we already know that

x + y + z = 8000

Using these two equations, we can see that y = 2000.

Since this statement provides a unique value for y, it is sufficient to answer the
question.

www.manhattanreview.com 19992016 Manhattan Review


Concepts of Data Sufficiency 17

Please pay attention to statements A and B. While both look similar in appearance,
one is sufficient and the other is insufficient.

3. Insufficient. This statement says


x =2 y +z

We are already given that

x + y + z = 8000

Substituting the value of x from the first equation in the second equation, we have

3 y + z = 8000

8000
The above equation allows y to take any value from 0 to .
3

Since this statement does not provide a unique value for y, it is not sufficient to
answer the question.

4. Sufficient. This statement says that

x y
= =z
3 2

Using above relationship, we can write both x and z in terms of y.

Then, we can substitute such values in the given equation x + y + z = 8000, and
derive a unique value of y.

Since this statement provides a unique value of y, it is sufficient to answer the ques-
tion.

Note: You might have noticed in the questions we have done so far that in some of the
statements, we actually derive the value of the variable, and in some statements, we just
say that a unique value is possible and leave it at that. Why are we inconsistent?

Actually, our choice to fully solve or conceptually argue depends on the two factors:
Effort and Confidence.

Solving the equations and finding a unique value of the variable leaves no doubt that a
unique value is possible. However, of course, solving the equations requires effort.

On the other hand, conceptually arguing that a unique value is possible requires less ef-
fort i.e. saves us the effort of solving the equation. However, it doesnt make as confident
as we are when we solve the equations and find value. Why? Because there is always a

19992016 Manhattan Review www.manhattanreview.com


18 Concepts of Data Sufficiency

tiny chance that the given scenario may be an exception to our concept. In such a case,
wed be wrong.

So, essentially, there is a balance we need to maintain because if we always solve the
equations, we may end up taking more than the required time, and if we always argue
conceptually, we may end up getting more questions wrong. So, the choice is to be made
for every question, depending on ones comfort and confidence. Gradually, you should
try to reach a point where you need to completely solve as fewer equations as possible.
You should at least be able to avoid the complex equations as far as possible because
they will consume your precious time unnecessarily.

(5) What is the value of a + b + c?


1. (a + b + c)2 = 1
2. (a + b + c) (a + b c) = 0
3. (a + b + c)3 = 1
4. The median of {a, b, c} is 5
5. The mean of {a, b, c} is 10

Solution:

In this question, we are asked to calculate the value of a + b + c. We arent given any
information about a, b, and c. When nothing is given in a question, it means (or we
should assume) that we are dealing with real numbers.

1. Insufficient. If (a + b + c)2 = 1, then a + b + c could be equal to either 1 or 1.


Since this statement does not provide a unique value for a + b + c, it is not sufficient
to answer the question.

2. Insufficient. We know that if the product of two numbers is 0, then one of the num-
bers has to be equal to 0. So, we have

(a + b + c) = 0

OR

(a + b c) = 0

In the former case, a + b + c will be equal to 0. However, in the latter case,


a + b + c = 2c, and since c can take any assume, the expression can also assume any
value.

Since this statement does not provide a unique for a + b + c, it is not sufficient to
answer the question.

www.manhattanreview.com 19992016 Manhattan Review


Concepts of Data Sufficiency 19

3. Sufficient. If (a + b + c)3 = 1, then a + b + c has to be equal to 1. So, this statement


provides a unique value for a + b + c. Therefore, this statement is sufficient to an-
swer the question.

Please note how this statement turned out to be sufficient while Statement 1 wasnt.
The reason is that the squares of both positive and negative numbers are positive.
So, if we know the square of a number, we do not know whether the number is pos-
itive or negative. This was the case in Statement 1.

However, the cube of a positive number is positive and of a negative number is neg-
ative. Therefore, if we are given the cube of a number, we can determine the number
since the number will carry the same sign as its cube. This is the case with Statement
3.

In general, the case with the squares applies to all even powers i.e. if we are given
the value of a number raised to an even power, we cannot decide the sign of the
number it could be either positive or negative. However, if we are given the value
of a number raised to an odd power, the sign of the number will be the same as the
sign of the value of the number raised to odd power.

4. Insufficient. If the median of {a, b, c} is 5, we know that one of a, b, and c is 5 and


out of the remaining two numbers, one is greater than or equal to 5 and the other
less than or equal to 5. However, the three numbers could be 5, 5, and 5, or the three
numbers could be 0, 5, 100. There are endless possibilities for the three numbers
with this statement.

So we can see that this statement does not provide a unique for a + b + c. Therefore,
it is not sufficient to answer the question.

5. Sufficient. If the mean of {a, b, c} is 5, it means that

a+b+c
=5
3

In this case, a + b + c will have a unique value i.e. 15. Therefore, this statement
provides a unique value for a + b + c, and is thus sufficient to answer the question.

Takeaway: In this drill, we had to calculate the value of a + b + c. We found statements C


and E sufficient to calculate the value of a + b + c. What is important to notice here is that
in both statements C and E, we did not know the individual values of a, b, or c. So, if we
have to find the value of an expression involving multiple variables, we need not have the
values of the individual variables to find out the value of the expression. It is important to
keep this in mind since some students try to find the value of all the variables involved in
the expression.

(6) Is pqr = 1?

19992016 Manhattan Review www.manhattanreview.com


20 Concepts of Data Sufficiency

1. pq = 1
2. q2 = pr
3. pr = 0
2
4. p = q =
r

Solution:

This is a Yes/No type of question. We have to find out whether the value pqr is equal to 1.

Any statement that conclusively proves that pqr is equal to 1 will be a sufficient state-
ment.

Any statement that conclusively proves that pqr is not equal to 1 will also be a sufficient
statement.

Any statement that allows both the possibilities: one in which pqr is equal to 1 and one
in which pqr is not equal to 1 is not a sufficient statement.

1. Insufficient. If pq = 1, then pqr = r . So, if r = 1, then pqr = 1. In such a case, the


answer to the question is YES. However, if, for example, r = 2, then pqr = 2. In this
case, the answer to the question is NO.

Since this statement does not provide a unique answer to the question, it is insuffi-
cient to answer the question.

2. Insufficient. If q2 = pr , then pqr = q3 . Now, if q = 1, then pqr = 1, and the


answer to the question is YES. However, if, for example, q = 2, then pqr = 8, and
the answer to the question is No.

Since this statement does not provide a unique answer to the question, it is insuffi-
cient to answer the question.

3. Sufficient. If pr = 0, then pqr = 0, irrespective of the value of q since any number


multiplied by 0 yields 0. Therefore, the answer to the question is NO.

Since this statement provides a unique answer to the question, it is sufficient to an-
swer the question.

2 2 4
4. Sufficient. In this scenario, pqr = r , which yields pqr = r . Thus,
r r r
we have pqr = 4. Therefore, there is only one answer to the question i.e. No.

Since this statement provides a unique answer to the question, it is sufficient to an-
swer the question.

www.manhattanreview.com 19992016 Manhattan Review


Concepts of Data Sufficiency 21

Now, having built some good understanding of data sufficiency and applied the same on
a number of questions, lets see how this concept is actually tested on the GMAT i.e. the
format of the data sufficiency questions on GMAT.

19992016 Manhattan Review www.manhattanreview.com


22 Concepts of Data Sufficiency

2.3 How is it tested on the GMAT?


Every Data Sufficiency or, in short DS, question on GMAT consists of a question stem followed
by two (information) statements and five option statements. Heres one example:

If John bought a total of 15 candies, how many did he eat?

1. He ate less than half the candies he bought

2. He ate more than six candies

(A) Statement (1) ALONE is sufficient, but statement (2) ALONE is not sufficient to answer the
question asked.

(B) Statement (2) ALONE is sufficient, but statement (1) ALONE is not sufficient to answer the
question asked.

(C) BOTH statements (1) and (2) TOGETHER are sufficient to answer the question asked, but
NEITHER statement ALONE is sufficient to answer the question asked.

(D) EACH statement ALONE is sufficient to answer the question asked.

(E) Statements (1) and (2) TOGETHER are NOT sufficient to answer the question asked, and
additional data specific to the problem are needed.

Question stem
If John bought a total of 15 candies, how many did he eat?

Information Statements

(1) He ate less than half the candies he bought

(2) He ate more than six candies

Option Statements

(A) Statement (1) ALONE is sufficient, but statement (2) ALONE is not sufficient to answer
the question asked.

(B) Statement (2) ALONE is sufficient, but statement (1) ALONE is not sufficient to answer
the question asked.

(C) BOTH statements (1) and (2) TOGETHER are sufficient to answer the question asked,
but NEITHER statement ALONE is sufficient to answer the question asked.

(D) EACH statement ALONE is sufficient to answer the question asked.

www.manhattanreview.com 19992016 Manhattan Review


Concepts of Data Sufficiency 23

(E) Statements (1) and (2) TOGETHER are NOT sufficient to answer the question asked,
and additional data specific to the problem are needed.

The format of a DS question is fixed and so are the five option statements. The five option
statements that you see above are the ones you are going to see in the actual GMAT. So, lets
get familiarized with them.

It is easier to understand the option statements by using simple logic. When you are given a
statement to test its sufficiency with respect to a question, the statement can be either Suffi-
cient (S) or Not Sufficient (NS). Right?

Now, if you are given two statements, the following four combinations are possible:

(1) S1: Sufficient AND S2: Not Sufficient

(2) S1: Not Sufficient AND S2: Sufficient

(3) S1: Sufficient AND S2: Sufficient

(4) S1: Not Sufficient AND S2: Not Sufficient

Now, you can see that in the first three cases above, well find an answer to the question since
at least one of the statements is sufficient. However, in the fourth case, well not have an
answer since none of the statements is sufficient.

Now, in the fourth case, what we can do is that we can try to combine the information in
both the statements and see if the combined information is sufficient to find the answer to the
question.

So, we break down the fourth case into two sub-cases

4a. S1 and S2 together are sufficient to answer the question

4b. S1 and S2 together are not sufficient to answer the question

Now, if we put these two cases in place of the fourth case above, well have five cases as below:

(1) S1: Sufficient AND S2: Not Sufficient

(2) S1: Not Sufficient AND S2: Sufficient

(3) S1: Sufficient AND S2: Sufficient

(4) S1 AND S2 Together: Sufficient

(5) S1 AND S2 Together: Not Sufficient

19992016 Manhattan Review www.manhattanreview.com


24 Concepts of Data Sufficiency

These are essentially the five option statements in a DS question, though in a slightly different
order. If I swap the positions of the 3rd and 4th statements above, well have the same order
as in GMAT DS questions.

(1) S1: Sufficient AND S2: Not Sufficient

(2) S1: Not Sufficient AND S2: Sufficient

(3) S1 AND S2 Together: Sufficient

(4) S1: Sufficient AND S2: Sufficient

(5) S1 AND S2 Together: Not Sufficient

It is important that you understand and internalize these statements since you are going to
see them over and over again in the DS questions. However, some of the internalization will
also happen when you solve the DS questions.

A short form of the option statements can be:

(A) (1) only

(B) (2) only

(C) (1) and (2) combined

(D) Either (1) or (2)

(E) Not Enough

Please note that all the option statements ask about the sufficiency of the statements and not
the answer to the question asked in the question stem. So, the idea it is worth repeating
is not to solve the question but to determine whether the question can be solved using the
information in the statements. The question can be solved if we can find a unique answer to
the question.

www.manhattanreview.com 19992016 Manhattan Review


Concepts of Data Sufficiency 25

2.4 Two types of DS questions

There are two types of DS questions that appear on GMAT

Yes or No

What is the value?

We have already seen these types of questions in our multiple answer drills. The idea in both
of these types is to see if a unique answer can be found to the question. In a Yes/No question,
a unique answer will mean either an unequivocal YES or an unequivocal NO. If a statement or
the combination of statements allows the possibility of both YES and No, then the statements
are not sufficient.

Similarly, in the case of a value question, we have to assess whether we can find a unique
value to the variable or expression in the question. If a unique value can be found, the state-
ment or the statements are sufficient. If a unique value cannot be found, then the statement
or the statements are not sufficient.

For Yes or No questions, there are two common traps that students mostly get into.

(1) Different values, Same answer: These are the questions in which you get more than
one value for the variable(s). However, for each value of the variable, you get the same
answer (either Yes or No) to the question asked. Since the value is not important here,
just whether we get a Yes or No to the question, the statement is sufficient to answer
the question. However, some test takers seeing that multiple values of the variable are
possible reject the statement as Not Sufficient.

(2) No does not mean Not Sufficient: These are the questions in which you get a unique
answer No to the question asked. Since there is a unique answer to the question, the
statement is sufficient to answer the question. However, some test takers seeing No as
an answer reject the option as Not Sufficient.

Lets do a couple of questions to understand what we just learnt.

Here are two questions each followed by a statement. Your job is to assess whether the state-
ment is sufficient to answer the question.

Q1: Is m < 3?

(1) m4 = 9

Solution: We are given that m4 = 9. Taking square root of both sides, well have

19992016 Manhattan Review www.manhattanreview.com


26 Concepts of Data Sufficiency

m2 = 3 (Please note that m2 cannot be equal to 3, since the square of a real number cannot
be a negative number)

Taking square root again of both the sides, well have



m= 3 OR m = 3

Now, even though two values are possible for m, both of these values are less than 3. There-
fore, the answer to the question is YES. So, the statement is indeed sufficient to answer the
question.

Q2: Is m = 5?

(1) 2m 25 = 4m 45

Solution: Lets simplify the expression in the statement. By simplifying, well get

2m = 20

This will give us m = 10.

So, the answer to the question is NO. Since this statement provides a unique answer to the
question, even though the answer is NO, it is sufficient to answer the question.

Please note that you didnt need to solve the equation and find the value of m. Since a linear
equation in one variable was given, it had to give a unique value of m. Once the value is found,
one can easily see if it is equal to 5 or not. So, using this understanding, you could have simply
marked the statement as sufficient and moved on.

For what is the value? questions, test takers often make one of the following two errors:

(1) Unwarranted Assumptions: Some test takers often assume numbers to be integers or
positive even when no such information is provided in the question stem. Such assump-
tions make insufficient statements seem sufficient.

(2) Ignoring Common sense: In some of the questions, the scenario commands the variables
to take only a specific set of value e.g. positive values or only integral values. Ignoring
such common sense restrictions make sufficient statements seem insufficient statements.

Lets do a couple of drills to understand the above two errors.

Here are two questions each followed by a statement. Your job is to assess whether the state-
ment is sufficient to answer the question.

Q1: What is the value of m?

www.manhattanreview.com 19992016 Manhattan Review


Concepts of Data Sufficiency 27

(1) m2 = 3m

Solution: We are given m2 = 3m. Now, one clear solution to this equation is m = 3. However,
before marking the statement as sufficient, we need to ask ourselves, Is this the only solu-
tion?. The answer is No because m = 0 is also a solution to the equation. However, many
people ignore this solution because they divide both sides by m to get m = 3. However, to
divide both sides by a number, you need to assume that the number is not 0. However, we
cannot assume m to be non-zero.

Therefore, there are two values possible for m from this equation. Thus, this statement is not
sufficient to answer the question.

Q2: Julie organized a conference in which she invited x number of people. What is the value
of x?

(1) x 2 x 90 = 0

Solution: We are given x 2 x 90 = 0, which can be factorized as (x 10)(x + 9) = 0

Now, from the above equation, either x 10 = 0 or x + 9 = 0.

It means either x = 10 or x = 9

So, the equation doesnt seem to give a unique value for x. However, we already know that
x stands for the number of people invited for the conference. Thus, x cannot be a negative.
Therefore, the only possible value for x from this equation is 10.

Since the equation provides a unique value for x, the statement is sufficient to answer the
question.

19992016 Manhattan Review www.manhattanreview.com


28 Concepts of Data Sufficiency

2.5 Approach to the DS questions

Now, we come to one of the most important parts of this book: how to approach DS questions?

When you see a DS question, what is the first thing you should do? What is the most effective
way to solve a DS question?

Heres a 4-step process that we think works really well with most of the test takers.

(1) Read the question stem well to glean the maximum information out of it. Even though
most of the DS question stems are short and easy to understand, it is important that you
read them carefully so as not to miss any information contained in it.

(a) When the question stem describes a situation in words, it is generally helpful to
convert the words into mathematical form so that you know precisely what you are
looking for from the information statements.
(b) In cases when the question stem contains a mathematical expression or equation
that can be simplified, please do so before reading the information statements.

Lets take a few examples to understand the points made above:

If x is a product of three consecutive odd integers, what is the value of x?

1. All three integers are less than 6


2. All three integers are positive

(A) Statement (1) ALONE is sufficient, but statement (2) ALONE is not sufficient to an-
swer the question asked.
(B) Statement (2) ALONE is sufficient, but statement (1) ALONE is not sufficient to an-
swer the question asked.
(C) BOTH statements (1) and (2) TOGETHER are sufficient to answer the question asked,
but NEITHER statement ALONE is sufficient to answer the question asked.
(D) EACH statement ALONE is sufficient to answer the question asked.
(E) Statements (1) and (2) TOGETHER are NOT sufficient to answer the question asked,
and additional data specific to the problem are needed.

The correct answer is C. However, to mark C and not E, you have to glean all the informa-
tion out of the question stem. The question provides two sets of information about the
integers whose product equals x.

All three integers are odd

www.manhattanreview.com 19992016 Manhattan Review


Concepts of Data Sufficiency 29

They are consecutive. So, if one is m, the other two are m + 2, m + 4.

Now, if you miss even one piece of information, you cannot solve the question, and thus
will end up marking the option E.

We can also refer to one of the questions we did earlier:

Julie organized a conference in which she invited x number of people. What is the value
of x?

Now, in this question too, it was important that you recognized that x can take only
positive integral values. x can be neither a fraction nor a negative number.

This is what we call gleaning the maximum information out of the question stem.

In case when the question stem describes a situation in words, it is generally helpful to
convert the situation into a mathematical equivalent. The more complex the situation is,
the better it is to convert it into its mathematical equivalent.

Lets take an example to understand this.

Rick invested $40,000 for one year. He invested a part of this amount at a simple
annual interest of x percent per year, and the rest at a simple annual interest of
y percent per year. If Rick earned a total interest of $2,720 in one year from his
invested of $40,000, what is the value of x?
3y
1. x =
4
2. The ratio of the amount invested at the rate of x percent per year to the amount
invested at the rate of y percent per year was 3 : 2.

(A) Statement (1) ALONE is sufficient, but statement (2) ALONE is not sufficient to an-
swer the question asked.
(B) Statement (2) ALONE is sufficient, but statement (1) ALONE is not sufficient to an-
swer the question asked.
(C) BOTH statements (1) and (2) TOGETHER are sufficient to answer the question asked,
but NEITHER statement ALONE is sufficient to answer the question asked.
(D) EACH statement ALONE is sufficient to answer the question asked.
(E) Statements (1) and (2) TOGETHER are NOT sufficient to answer the question asked,
and additional data specific to the problem are needed.

19992016 Manhattan Review www.manhattanreview.com


30 Concepts of Data Sufficiency

In general, you cant comprehend exactly what all information you need to solve for x
unless you convert the situation into its mathematical equivalent.

To convert the situation into an equation, we need to assign a variable, lets say m, for
the amount invested at x percent per year. Then, the amount invested at y percent per
year becomes 40000 m. So, with the given information, we have the following equation:

x y
m + (40000 m) = 2720
100 100

So, we have one equation and three variables. Now, lets look at the information state-
ments.

3y
1. Using x = to calculate y in terms of x and using that in the equation, we get
4
x 4x
m + (40000 m) = 2720
100 300

We can see that we still have two variables and one equation. So, we cannot solve
the same for a unique value of x. Therefore, Statement 1 alone is not sufficient.

2. Using Statement 2, we have

m 3
=
40000 m 2

Solving the equation, we get m = 24000.

Now, if we substitute this value in the original equation, we get

x y
24000 + 16000 = 2720
100 100

So, we still have two variables and one equation. Thus, we cannot get a unique value
of x using statement 2 alone. Therefore, Statement 2 alone is not sufficient.

Lets try combining the two statement to see if we can get an answer. Lets substitute
the value of m = 24000 in the equation we got while analyzing the first statement.
We have

x 4x
24000 + 16000 = 2720
100 300

We can see that now we have an equation in only one variable, and thus, we can have
a unique value of x. Therefore, Statements 1 and 2 together are sufficient to answer
the question.

Even though we solved the question in its entirety, the main point to be derived from this
example is that you should try to form mathematical expressions when the situation is

www.manhattanreview.com 19992016 Manhattan Review


Concepts of Data Sufficiency 31

described in words.

Now, there are cases when the questions are described in mathematical form. However,
the given mathematical form is not easy to deal with as is but can be simplified. In such
cases, you should simplify the mathematical form as much as possible before looking at
the information statements. The answer to the question is C.

Lets look at an example.

If 3x+y = 98 , what is the value of 4y ?

1. x 2 = 100
2. x y = 4

(A) Statement (1) ALONE is sufficient, but statement (2) ALONE is not sufficient to an-
swer the question asked.
(B) Statement (2) ALONE is sufficient, but statement (1) ALONE is not sufficient to an-
swer the question asked.
(C) BOTH statements (1) and (2) TOGETHER are sufficient to answer the question asked,
but NEITHER statement ALONE is sufficient to answer the question asked.
(D) EACH statement ALONE is sufficient to answer the question asked.
(E) Statements (1) and (2) TOGETHER are NOT sufficient to answer the question asked,
and additional data specific to the problem are needed.

In the question stem, we are given information in the form of an exponential expression,
and we can see that none of the information statements are in the form of an exponential
expression. Therefore, before proceeding to the information statements, we should try
to simply the expression.

8
To simply the expression, lets write 98 as (32 ) , which becomes 316 . So now, we have

3x+y = 316

From this, we get

x + y = 16

So, we have an equation in two variables. Before we look at the information statements, it
is important to understand that even though we are asked the value of 4y , we shouldnt
worry about finding 4y from the information in the statements. If we can find a unique

19992016 Manhattan Review www.manhattanreview.com


32 Concepts of Data Sufficiency

value of y from the statements, we know that we can find a unique value of 4y . There-
fore, it will help us if we treat the question as what is the value of y?, rather than as
given.

Now, lets look at the statements.

1. From x 2 = 100, we get two possible values of x i.e. 10 or 10. Since each of
these values will produce a different value of y when plugged into the equation
x + y = 16, this statement does not provide us with a unique value of y. Therefore,
this Statement (1) alone is not sufficient to solve the question.
2. Using the equation x y = 4 and the equation from the question stem x + y = 16,
we know that we can get a unique value for y since we have two variables and two
equations. Therefore, Statement (2) alone is sufficient to answer the question.

The point to be learned from the above question is that we should try to simplify
the mathematical form given in the question stem as much as possible. A simplified
form will help us evaluate the sufficiency of the information statements much more
quickly and with greater accuracy. The answer to the question is B.

(2) After reading the question stem, read the first information statement and test whether
it is sufficient alone. In case you see that the second information statement is much eas-
ier to deal with, you can start with the second information statement. It doesnt matter
in which order you test the statements for sufficiency.

(3) Read the other information statement and test whether it is sufficient alone. When
reading this information statement, ignore the information provided in the previous in-
formation statement. Using the information from the previously read information state-
ment while evaluating the second information statement is one of the common types of
errors.

(4) If none of the statements alone is sufficient, combine the information in the two state-
ments and test if the combined information is sufficient.

Use the decision tree to determine the correct option statement.

www.manhattanreview.com 19992016 Manhattan Review


Concepts of Data Sufficiency 33

2.5.1 Decision Tree

Is Statement 1 Alone
Sufficient?
YES NO

Is Statement 2 Alone Is Statement 2 Alone


Sufficient? Sufficient?
YES NO YES NO

Correct Correct Correct Is Statements 1 and 2


Answer Answer Answer Together Sufficient?
:D :A :B YES NO

Correct Correct
Answer Answer
:C :E

Heres one way to use this decision tree when you sit for the GMAT.

AD
(1) Write down on the scratch pad.
BCE

(2) Evaluate Statement 1

(a) If Statement 1 is sufficient, cross off BCE since only A or D can be the answer
(b) If Statement 1 is not sufficient, cross off AD

(3) Now, evaluate Statement 2

(a) If Statement 2 is sufficient


i. If Statement 1 was also sufficient, then D is the answer
ii. If Statement 1 was not sufficient, then B is the answer
(b) If Statement 2 is not sufficient
i. If Statement 1 was sufficient, then A is the answer
ii. If Statement 1 was not sufficient, then cross off B out of BCE (AD would have
been crossed out when Statement 1 was found insufficient) i.e. now, only C or E
could be the answer

(4) Now, evaluate Statements 1 and 2 together

(a) If they together are sufficient, then C is the answer


(b) If they together are not sufficient, then E is the answer
BD
If you evaluate the second statement first, then you will need to write on the scratch
ACE
pad, and follow the above process accordingly.

19992016 Manhattan Review www.manhattanreview.com


34 Concepts of Data Sufficiency

Lets briefly recap our 4-step process.

(1) Read the question stem, glean the maximum information out of it, and if necessary,
simplify it.

(2) Read the first information statement, or the second one, if it is easier, and evaluate its
sufficiency.

(3) Read the other information statement and evaluate its sufficiency.

(4) If none of the statements alone is sufficient, combine the information in the two state-
ments and then test if the combined information is sufficient.

2.5.2 Two ways to get a question wrong


Well, there are only two ways to get a question wrong.

(1) Judging a sufficient statement to be insufficient

(2) Judging an insufficient statement to be sufficient

There are many reasons that the test takers end up making one of the above two errors.
However, there are some reasons that are fairly common. In the next section, we look at seven
such common reasons or, in other words, mistakes. We have already talked about most of
these mistakes in different sections so far, but here we bring them and study them together.
Besides, learning about our common mistakes is such an important part of preparation that
these are worth repeating.

www.manhattanreview.com 19992016 Manhattan Review


Concepts of Data Sufficiency 35

2.6 Seven Common Mistakes

1. Carry Over

What is the value of x?

1. x > 2

2. x 2 = 16

(A) Statement (1) ALONE is sufficient, but statement (2) ALONE is not sufficient to answer the
question asked.

(B) Statement (2) ALONE is sufficient, but statement (1) ALONE is not sufficient to answer the
question asked.

(C) BOTH statements (1) and (2) TOGETHER are sufficient to answer the question asked, but
NEITHER statement ALONE is sufficient to answer the question asked.

(D) EACH statement ALONE is sufficient to answer the question asked.

(E) Statements (1) and (2) TOGETHER are NOT sufficient to answer the question asked, and
additional data specific to the problem are needed.

Solution:

The first statement is clearly insufficient since x can take any value greater than 2. The second
statement also allows two values of x: 4 and 4. Therefore, Statement 2 alone is also insuffi-
cient.

However, some test takers, while evaluating Statement 2, mistakenly use the information from
Statement 1. By doing so, they reject 4 as the value of x, and since only one value remains
i.e. 4, they mistake Statement 2 to be sufficient.

However, we can have a unique value of x only when we combine both statements. Therefore,
the correct option is C.

This question demonstrates what we call Carry Over mistake since the test taker carries over
information from Statement 1 to Statement 2.

Lets look at another question.

Is the below triangle an equilateral triangle?

19992016 Manhattan Review www.manhattanreview.com


36 Concepts of Data Sufficiency

(1) y = z

(2) x = 40

(A) Statement (1) ALONE is sufficient, but statement (2) ALONE is not sufficient to answer the
question asked.

(B) Statement (2) ALONE is sufficient, but statement (1) ALONE is not sufficient to answer the
question asked.

(C) BOTH statements (1) and (2) TOGETHER are sufficient to answer the question asked, but
NEITHER statement ALONE is sufficient to answer the question asked.

(D) EACH statement ALONE is sufficient to answer the question asked.

(E) Statements (1) and (2) TOGETHER are NOT sufficient to answer the question asked, and
additional data specific to the problem are needed.

Solution:

What is the answer to the above question?

E?

No. The answer is B.

When we evaluate Statement 1, we can clearly see that it is insufficient since we need all the
three angles to be equal for the triangle to be considered equilateral. If two angles are equal,
as is given in Statement 1, the triangle might be equilateral or might not be. We cannot say.
Both possibilities exist.

Now, after going through Statement 1, when we look at Statement 2, which provides infor-
mation about just one angle, we immediately reject its sufficiency since in our mind, we are
looking for three angles to be equal. If Statement 1, which provided information about two
angles, was not sufficient, how Statement 2 could be, we reason out.

However, if we pay attention, Statement 2 tells us that one of the angles is 40 . Therefore,
this triangle cannot be an equilateral triangle since each angle of an equilateral triangle is 60 .
Thus, Statement 2 alone is sufficient.

Now, if Statement 2 were Statement 1, then we might not have been so eager to reject it. How-
ever, because we had evaluated Statement 1 and saw that we needed not just two but all three

www.manhattanreview.com 19992016 Manhattan Review


Concepts of Data Sufficiency 37

angles to be equal, we rejected Statement 2 on seeing that it provided information on just one
angle.

This kind of mistake also we refer to as Carry Over since we carry over a fixed approach to
solve the question from one statement to another. Clearly, there was another way to evaluate
the sufficiency of the statement than to just look for three angles to be equal. However, State-
ment 1 fixed that one approach into our minds.

GMAC of course knows that this trick works and uses the same especially in harder questions.
Therefore, be aware.

2. Unwarranted Assumptions
What was the speed at which Tom was driving when he had completed half the total distance
of the trip?
(1) Tom completed 400 miles in 8 hours

(2) Tom drove at an average speed of 50 miles per hour during the first four hours.

(A) Statement (1) ALONE is sufficient, but statement (2) ALONE is not sufficient to answer the
question asked.

(B) Statement (2) ALONE is sufficient, but statement (1) ALONE is not sufficient to answer the
question asked.

(C) BOTH statements (1) and (2) TOGETHER are sufficient to answer the question asked, but
NEITHER statement ALONE is sufficient to answer the question asked.

(D) EACH statement ALONE is sufficient to answer the question asked.

(E) Statements (1) and (2) TOGETHER are NOT sufficient to answer the question asked, and
additional data specific to the problem are needed.

Solution:

To get any other answer, you will have to make some unwarranted assumptions. Lets under-
stand it.

In the question, we are asked the speed of Tom at the instant when he had completed half
the trip. So, we are talking about his instantaneous speed, and not his average speed, at the
moment he completed half the distance of the trip.
 Total Distance 
From the first statement, we can calculate his average speed for the trip Average Speed =
Total Time
However, we dont want average speed, and Statement 1 doesnt provide us with the instanta-
neous speed when Tom completed half the trip. Therefore, Statement 1 alone is insufficient.

From the second statement, we get his average speed for the first four hours. Clearly, this
statement also doesnt provide the instantaneous speed that we are looking for. Therefore,

19992016 Manhattan Review www.manhattanreview.com


38 Concepts of Data Sufficiency

this statement alone is also insufficient.

Even if we combine both the statements, we only get to know that his average speed for the
entire trip (which comes out to be 50 miles per hour by dividing 400 miles by 8 hours) is the
same as his average speed for the first four hours i.e. for the first half of the trip (since in the
first four hours, he would have completed 200 miles i.e. half the trip). However, again, we do
not have an idea of Toms speed at the moment when he completed half the trip. He could be
traveling at his average speed or higher than average speed or lower than average speed. We
cannot say.

Therefore, the answer is option E.

However, many people end up choosing one of the other options, assuming his instantaneous
speed to be the same as his average speed for the entire trip or his average speed for the first
half of the trip. Both of these assumptions are unwarranted. Even from our everyday experi-
ence, we know that we do not drive at our average speed all the time; sometimes, we are at a
higher speed; sometimes, we are at a lower speed.

This question demonstrates one example of unwarranted assumptions. Many a time, test tak-
ers assume numbers to be positive or integers even if nothing is given in the question stem.
We cannot make any such assumptions. If nothing is given about a number in the question
stem, we have to assume it to be a real number, not an integer or a positive number.

At times, test takers make assumptions using geometry figures given with the questions. How-
ever, GMAT clearly prohibits from making any assumptions using geometry figures. Just be-
cause the lines look equal, we cannot assume them to be equal. Just because angles look equal,
we cannot assume them to be equal. Just because lines look parallel, we cannot assume them
to be parallel. We have to go strictly by the text accompanying the figure. If no such equalities
are presented in the text, we shouldnt assume them. The only information that you can get
from a figure is the right angle, which is denoted by  as drawn by below:

In the above figure, the  symbol denotes that the two lines are perpendicular to each other,
and you are required (i.e. you mustnt ignore) to use this information while assessing the suf-
ficiency of the statements.

Also, there could be cases in which two or more angles are both represented by same number,
lets say x , in the figure. In those cases also, you are required to use this information in the
question.

In essence, you should use the information that is given to you clearly. You shouldnt assume
things just because they look like.

www.manhattanreview.com 19992016 Manhattan Review


Concepts of Data Sufficiency 39

3. Ignoring Common sense

What is the total number of coins that Mike and Terry have?

(1) Mike has 25% more coins than Terry.

(2) The total number of coins that Mike and Terry have is between 21 and 28.

(A) Statement (1) ALONE is sufficient, but statement (2) ALONE is not sufficient to answer the
question asked.

(B) Statement (2) ALONE is sufficient, but statement (1) ALONE is not sufficient to answer the
question asked.

(C) BOTH statements (1) and (2) TOGETHER are sufficient to answer the question asked, but
NEITHER statement ALONE is sufficient to answer the question asked.

(D) EACH statement ALONE is sufficient to answer the question asked.

(E) Statements (1) and (2) TOGETHER are NOT sufficient to answer the question asked, and
additional data specific to the problem are needed.

Solution:

To solve this question, you have to use common sense that number of coins has to be an inte-
ger i.e. it cannot be a fraction. If you do not use this understanding, you will end up marking
E as the answer.

Lets go through the solution.

Statement 1 just tells us that Mike has 25% more coins than Terry. Now, Mike could have 25
coins and Terry 20 coins, or Mike could have 50 coins and Terry 40 coins, or any other combi-
nation of integers such that Mike has 25% more coins than Terry. Since more than one solution
is possible, Statement 1 alone is not sufficient.

Statement 2 tells us that the total number of coins is between 21 and 28. So, they could have
21, 22, or 23 or any number of coins between 21 and 28. Clearly, this statement also doesnt
provide a unique answer, and is therefore insufficient.

Now, if we combine Statement 1 and Statement 2, we have two sets of information from the
two statements.

Lets say that Terry has x number of coins. Then, from Statement 1, Mike has 1.25x number
of coins. So, the total number of coins is 2.25x. And since we are talking about the number
of coins, 2.25x has to be an integer. Now, for 2.25x to be an integer, x has to be a multiple of 4.

So, lets say x is equal to 4c, where c is some other integer.

19992016 Manhattan Review www.manhattanreview.com


40 Concepts of Data Sufficiency

Now, the total number of coins is 2.25 4c i.e. 9c.

So, we see that the total number of coins has to be a multiple of 9.

From Statement 2, we know that the total number of coins is between 21 and 28.

Now, what are the multiples of 9 between 21 and 28?

Only 27.

So, we get a unique answer.

Therefore, Statements 1 and 2 together are sufficient to answer the question. Thus, C is the
correct option.

So, from this example, we see that we have to use some common sense while evaluating an-
swers. Other common sense assumptions would include assuming the number of people to be
a positive integer and assuming the quantity of something to be a positive number. You have
to use the context of the question to make these common sense assumptions. If you ignore
them, youd end up marking sufficient statements as insufficient.

As you can see, in the previous type of mistake, we talked about making unwarranted as-
sumptions i.e. assuming things that arent there, and in this, we talked about not making
common sense assumptions i.e. ignoring things that are there. So, as you go about practicing
and solving DS questions, keep an eye on the situations when you make one of these kinds of
errors.

4. No does not mean insufficient

Is integer x divisible by 18?

(1) x is divisible by 30

(2) x is a multiple of three prime numbers

(A) Statement (1) ALONE is sufficient, but statement (2) ALONE is not sufficient to answer the
question asked.

(B) Statement (2) ALONE is sufficient, but statement (1) ALONE is not sufficient to answer the
question asked.

(C) BOTH statements (1) and (2) TOGETHER are sufficient to answer the question asked, but
NEITHER statement ALONE is sufficient to answer the question asked.

(D) EACH statement ALONE is sufficient to answer the question asked.

(E) Statements (1) and (2) TOGETHER are NOT sufficient to answer the question asked, and
additional data specific to the problem are needed.

www.manhattanreview.com 19992016 Manhattan Review


Concepts of Data Sufficiency 41

Solution:

The answer is option B. However, many test takers mark the correct option as either C or E.

Lets look at the solution.

Statement 1 tells us that x is a multiple of 30. Now, if we can think of two multiples of 30,
one of which is a multiple of 18 and one which is not, then well have proved Statement 1
insufficient. A multiple of 30 which is not a multiple of 18 is 30 itself. Now, the systematic
way to find a multiple of 30 which is also a multiple of 18 is by finding the LCM of 18 and 30,
which comes out to be 90. So, we have two multiple of 30 i.e. 30 and 90, one of which is a
multiple of 18 and one which is not. Therefore, Statement 1 alone is not sufficient to answer
the question.

Statement 2 tells us that x is a multiple of three prime numbers. Now, if we look at the factors
of 18, they are 2, 3, and 3. So, any number which is divisible by 18 must contain two 3s in its
factors. Now, since x is a multiple of three prime numbers and no prime number except 3 has
3 as its factor, x cannot have its factor two 3s i.e. x cannot be divisible by 9. Therefore, x is
not divisible by 18. Since this statement provides a unique answer (No) to the question, the
statement is sufficient to answer the question.

So, option B is the correct option.

Now, some test takers mark option E as the correct option because they reject Statement 2 on
seeing No as the answer to the question. However, one cannot put the entire blame on these
test takers for such mistake because some of it has got to do with how our brain works. When
our brain sees No i.e. something negative as an answer, it sticks to the negativity, thereby
producing a negative final answer i.e. insufficient.

So, be careful during the exam not to reject statements producing a consistent No answer as
insufficient.

Now, going back to the question, some test takers also mark option C as the answer. They
reject Statement 1 for the reasons discussed above, and then they reject Statement 2 too for
either the reasons discussed above or for some other reason. And then they combine the two
statements. By combining the two statements, they figure out that x has to be 30 since 30 is
the only number that satisfies both Statement 1 and Statement 2. Having figured out a unique
value of x, they know that this is sufficient to answer the question, thereby marking C as the
answer. However, in this question, we need not figure out the unique value of x to answer
the question. Therefore, Statement 2 alone is also sufficient. Even though Statement 2 doesnt
provide a unique value of x, it provides a unique answer to the question asked.

5. Overlooking information in the question

What is the sum of a certain pair of consecutive odd integers, x and y?

19992016 Manhattan Review www.manhattanreview.com


42 Concepts of Data Sufficiency

(1) xy < 0

(2) y > 0

(A) Statement (1) ALONE is sufficient, but statement (2) ALONE is not sufficient to answer the
question asked.

(B) Statement (2) ALONE is sufficient, but statement (1) ALONE is not sufficient to answer the
question asked.

(C) BOTH statements (1) and (2) TOGETHER are sufficient to answer the question asked, but
NEITHER statement ALONE is sufficient to answer the question asked.

(D) EACH statement ALONE is sufficient to answer the question asked.

(E) Statements (1) and (2) TOGETHER are NOT sufficient to answer the question asked, and
additional data specific to the problem are needed.

Solution:

Most test takers will get this question right if they extract all the information given in the
question stem.

The stem provides the following information about x and y:

Both are integers

Both are odd

They are consecutive. So, either x = y + 2 or y = x + 2

Statement 1 tells us that xy < 0, it means the one of x and y is positive and the other is
negative. Now, the only two consecutive odd integers that have different signs are 1 and 1.
And their sum is 0. Therefore, Statement 1 provides a unique answer for the question. Thus,
Statement 1 is sufficient.

Statement 2 tells us that y > 0. In this case, x and y could be 1 and 1 (lets say y = 1), or
x and y could be 1 and 3. Now, since these two possibilities will produce different answers
for the question asked, Statement 2 alone is not sufficient to answer the question. Therefore,
Statement 2 alone is not sufficient.

Thus, the correct option is A.

The idea behind this example is that one should glean out all the information from a question.
For this, one may need to pay attention to each and every word in the question stem. The cost
of missing an information is significant: one may end up marking a sufficient statement as
insufficient and thus, end up getting the question wrong.

www.manhattanreview.com 19992016 Manhattan Review


Concepts of Data Sufficiency 43

One other popular case in which test takers fail to glean out all the information from a question
is the one in which the question talks about non-negative integers. On reading non-negative
integers, some people assume the number to be a positive integer. However, non-negative
integers also include the integer 0, which may play a significant role in assessing the sufficiency
of the statements. One must also be careful between non-negative numbers and non-negative
integers. Non-negative numbers are all the real numbers (not only integers) greater than 0.

6. Calculating the answer

What is the square root of x?

(1) The cube root of x is 81

(2) The 6th root of x is 9

(A) Statement (1) ALONE is sufficient, but statement (2) ALONE is not sufficient to answer the
question asked.

(B) Statement (2) ALONE is sufficient, but statement (1) ALONE is not sufficient to answer the
question asked.

(C) BOTH statements (1) and (2) TOGETHER are sufficient to answer the question asked, but
NEITHER statement ALONE is sufficient to answer the question asked.

(D) EACH statement ALONE is sufficient to answer the question asked.

(E) Statements (1) and (2) TOGETHER are NOT sufficient to answer the question asked, and
additional data specific to the problem are needed.

Solution:

Statement 1 provides the cube root of x. If we cube 81, well get x. Once we have x, we can
have the square root of x. Thus, this statement will provide a unique answer to the question.
Therefore, Statement 1 is sufficient.

Statement provides 6th root of x. If we raise 9 to the power 6, well have x. Once we have x,
we can have the square root of x. Thus, this statement will provide a unique answer to the
question. Therefore, Statement 2 is also sufficient.

Therefore, the correct option is D.

The question wasnt tricky, and almost everyone gets this question correct. The key under-
standing from this question is that we shouldnt get into calculating the exact value of x or
square root of x. Once we know such a value can be calculated, we should mark the statement
as sufficient and move on. Unnecessarily getting into calculations will waste our precious time
during the test.

19992016 Manhattan Review www.manhattanreview.com


44 Concepts of Data Sufficiency

7. The C trap

If a = 5 and x is a positive integer, is ax > 23?

(1) x 2 < 27

(2) x > 2

(A) Statement (1) ALONE is sufficient, but statement (2) ALONE is not sufficient to answer the
question asked.

(B) Statement (2) ALONE is sufficient, but statement (1) ALONE is not sufficient to answer the
question asked.

(C) BOTH statements (1) and (2) TOGETHER are sufficient to answer the question asked, but
NEITHER statement ALONE is sufficient to answer the question asked.

(D) EACH statement ALONE is sufficient to answer the question asked.

(E) Statements (1) and (2) TOGETHER are NOT sufficient to answer the question asked, and
additional data specific to the problem are needed.

Solution:

The answer is Option B. However, some test takers striving to find a unique value of x mark
option C as the answer.

Lets look at the solution first.

The questions asks Is ax > 23?. However, the question also provides a value of a and the
fact that x is a positive integer. Now, ax > 23 means 5x > 23, which means x > 4.6. Now,
since x is an integer, this question is same as asking Is x 5?

With this simplified form of the question, lets look at the statements.

Statement 1 says that x 2 < 27, which means that (taking square root of both sides) x < a
number greater than 5 (since we know that 5 is the square of 25, so square root of 27 will
be slightly higher than 5). Now, this means that x 5. So, this statement allows x to be 5,
in which case the answer to the question will be YES. But this statement also allows x to be
less than 5, in which case the answer to the question will be NO. So, this statement does not
produce a unique answer to the question. Therefore, Statement 1 alone is not sufficient to
answer the question.

Statement 2 says that x > 2, which means (taking square of both sides) x > 4. Since x is an
integer, x > 4 means the same as x 5. Therefore, we see that Statement 2 provides a unique
answer YES to the question asked. Thus, Statement 2 is sufficient to answer the question.

www.manhattanreview.com 19992016 Manhattan Review


Concepts of Data Sufficiency 45

Therefore, the answer is Option B.

Now, we can see that even though Statement 2 is sufficient to answer the question, it doesnt
provide a unique value for x. However, if we combine statements 1 and 2 i.e. x 5 and x 5,
well have a unique value of x i.e. x = 5. Some test takers who are looking for a unique value
of x combine the two statements and mark option C as the answer. Clearly, this is incorrect.
We are not looking for a unique value of x but for a unique answer to the question asked.

There are other scenarios too in which option C looks more tempting since two statements
always seem better than one statement. However, one should mark C as the option only when
one is sure that no statement alone will be sufficient to answer the question.

In this section, we looked at seven of the most common mistakes that test takers make. We
looked at them so that you can avoid the same in your actual test. However, you may not be
making these errors or even if you were, you may not make them now. But you may have your
own set of mistakes that you make on a regular basis. It is important you nab them before your
GMAT. The way to understand and catch patterns in your mistakes is through maintaining an
error log. An error log is a document (mostly, an excel sheet) in which you track all the errors
you have made and your analysis why made them. As you keep a track of your errors, youll
eventually start seeing a pattern in them i.e. your own list of common errors. And once you
see them, you will have taken the first and the most important step in preventing them.

19992016 Manhattan Review www.manhattanreview.com


46 Concepts of Data Sufficiency

2.7 Beware of the common exceptions

Data Sufficiency is about finding whether the statements can solve the question and not about
solving the question itself. Therefore, many a time we skip the calculations and solving the
question and it is recommended to do so to save time. In those situations, we logically
understand the scenario and assess whether a solution is possible without really solving the
question.

The approach works most of the times. However, at times, we find ourselves at the wrong end.
We apply the concept and figure out the sufficiency of the statement, only to later find that
the situation at hand presented an exception to our concept. So, even though the concept is
generally valid, the situation presented an exception to our concept. For example: generally,
we know that if we have two equations in two variables, then we can find a unique solution for
the two variables. However, is it always true? Not really.

Similarly, there are exceptions to our other generally valid concepts. We list exceptions to a
few of the concepts. Clearly, the list is not exhaustive. The idea here is that while assessing
the sufficiency of a statement in a DS question, pay attention to whether the situation at hand
presents an exception to the rule or the concept. If it does not present an exception to the
concept, you may apply the concept and assess the sufficiency of the statement. However, if it
does, you will need to look at the situation specifically to see if the statement or the statements
are sufficient.

Exception 1: Two equations in two variables dont necessarily mean a unique solu-
tion is possible

How much does a pencil cost?

(1) 8 pencils and 6 pens cost $4

(2) 20 pencils and 15 pens cost $10

(A) Statement (1) ALONE is sufficient, but statement (2) ALONE is not sufficient to answer the
question asked.

(B) Statement (2) ALONE is sufficient, but statement (1) ALONE is not sufficient to answer the
question asked.

(C) BOTH statements (1) and (2) TOGETHER are sufficient to answer the question asked, but
NEITHER statement ALONE is sufficient to answer the question asked.

(D) EACH statement ALONE is sufficient to answer the question asked.

(E) Statements (1) and (2) TOGETHER are NOT sufficient to answer the question asked, and
additional data specific to the problem are needed.

www.manhattanreview.com 19992016 Manhattan Review


Concepts of Data Sufficiency 47

Solution:

Clearly, each statement alone is insufficient since each statement involves two variables, and
the questions asks us to find the value of one variable i.e. cost of a pencil.

What if we combine both statements?

Then, well have two equations in two variables. Conceptually, we know that a unique solution
can be found if we have two equations in two variables. However, not so in this case. Lets try
solving the two equations and find the value of the variables.

Lets denote the cost of a pencil by x and the cost of a pen by y. The equations for the two
statements are as below:

8x + 6y = 4

20x + 15y = 10

Now, lets try to solve the equations.

Lets try to eliminate y from the equations since we are looking for the value of x. So, lets
multiply the first equation with 2.5 so that we get 15y i.e. the co-efficient of y in the second
equation.

Multiplying the first equation with 2.5, we get

20x + 15y = 10

Now, this is exactly the same as the second equation. So, if we subtract it from the second
equation to eliminate y, everything else also gets eliminated, and we get 0 = 0. We see that we
cannot find a value of x (or even y) from these equations. Why so?

The reason is that these equations are not really different equations; these are essentially the
same equation disguised in different forms. We know that in an equation, we can multiply or
divide all the components by a number without affecting the equality. We can see that State-
ment 2 is just Statement 1 multiplied by 2.5.

So, we see that when one equation is just a different version of another equation (i.e. we can
multiply or divide one equation by a number to get another equation), then we cannot find
a unique solution of a set of two variables from two equations. The same is also valid for n
variables in n equations. We require all the n equations to be distinct if we want to find a
unique solution for n variables.

So, before judging a set of equations to be sufficient, always check that one equation is not a
multiple of the other equation. The answer is option E.

19992016 Manhattan Review www.manhattanreview.com


48 Concepts of Data Sufficiency

Exception 2: Two equations in three variables dont necessarily mean that a unique
solution is not possible

How much do 4 apples and 7 oranges cost?

(1) 4 apples, 3 oranges, and 2 bananas cost $16.

(2) 12 apples, 13 oranges, and 4 bananas cost $50.

(A) Statement (1) ALONE is sufficient, but statement (2) ALONE is not sufficient to answer the
question asked.

(B) Statement (2) ALONE is sufficient, but statement (1) ALONE is not sufficient to answer the
question asked.

(C) BOTH statements (1) and (2) TOGETHER are sufficient to answer the question asked, but
NEITHER statement ALONE is sufficient to answer the question asked.

(D) EACH statement ALONE is sufficient to answer the question asked.

(E) Statements (1) and (2) TOGETHER are NOT sufficient to answer the question asked, and
additional data specific to the problem are needed.

Solution:

The question asks us to calculate the cost of 4 apples and 7 oranges.

Statement 1 tells us the cost of a particular combination of apples, oranges, and bananas.
Clearly, we cannot extract the cost of 4 apples and 7 oranges from this statement. So, State-
ment 1 alone is insufficient.

Statement 2 tells us the cost of some other combination of apples, oranges, and bananas. It is
clear that we cannot extract the cost of 4 apples and 7 oranges from this statement. Therefore,
Statement 2 alone is also insufficient.

Now, if we combine the two statements, we will have two equations in three variables. First
of all, these equations are distinct i.e. one cannot be obtained by multiplying or dividing the
other equation by a number. However, we know that to find the value of three variables (out
of which two variables we need to answer the question), we need three equations. In general,
to find the value of n variables, we need n equations.

So, are both statements together insufficient?

Nope.

In the equation, we are asked the value of 4 apples and 7 oranges. The question doesnt talk
about bananas. So, lets try to eliminate bananas from these two equations.

www.manhattanreview.com 19992016 Manhattan Review


Concepts of Data Sufficiency 49

To represent the statements in equation form, lets say x is the price of an apple, y is the price
of an orange, and z is the price of a banana.

Now, from two statements, we have two equations

4x + 3y + 2z = 16

12x + 13y + 4z = 50

Since in the question, we are looking for a value of 4x + 7y, lets try to eliminate z from these
equations. To eliminate z, lets multiply the first equation with 2 and then subtract it from the
second equation. Well have
 
12x + 13y + 4z 2 4x + 3y + 2z = 50 2 16

Simplifying it, we have

4x + 7y = 18

Okay! This is what we were looking for in the question. So, we have the value of 4x + 7y. And
since the value is unique, the statements together are sufficient.

Therefore, the answer is option C.

In this question, we saw that even though there were fewer equations than the variables, a
unique answer still existed. So, you shouldnt reject a set of equations just because the num-
ber of variables is higher than the number of equations. You should try to manipulate the
equations to see if what we want can be found.

Lets take one more example:

What is the value of x?

(1) 2x + 3y + 7z = 48

(2) 5x + 6y + 14z = 100

(A) Statement (1) ALONE is sufficient, but statement (2) ALONE is not sufficient to answer the
question asked.

(B) Statement (2) ALONE is sufficient, but statement (1) ALONE is not sufficient to answer the
question asked.

(C) BOTH statements (1) and (2) TOGETHER are sufficient to answer the question asked, but
NEITHER statement ALONE is sufficient to answer the question asked.

(D) EACH statement ALONE is sufficient to answer the question asked.

19992016 Manhattan Review www.manhattanreview.com


50 Concepts of Data Sufficiency

(E) Statements (1) and (2) TOGETHER are NOT sufficient to answer the question asked, and
additional data specific to the problem are needed.

Solution:

Now, in this question, we have to find the value of a specific variable, not a combination of
variables as in the previous question. So, we have to find the value of a variable, but we have
two equations in three variables. Should we mark E as the correct option?

Actually No.

You should try manipulating the equations to see if the value of x can be found doing so. If
you multiply the first equation with 2 and subtract it from the second equation, youll have

5x + 6y + 14z 2 2x + 3y + 7z = 100 2 48

If you solve the above equation, both y and z will get eliminated, and youll have x = 4. Thus,
the correct option is C.

The idea behind presenting these examples is that there are exceptions to the concepts or
rules that we use. Unless we are aware of these exceptions, we may end up marking sufficient
statements as insufficient.

Another point worth noting is from the example above where we had to calculate the cost of 4
apples and 7 oranges. Both statements together did not provide us with the cost of an apple or
the cost of an orange. However, in spite of not having these values, we could calculate the cost
of 4 apples and 7 oranges. Therefore, we can have the value of an expression even without
having the value of individual variables contained in the expression. Thus, we should not rush
to calculate the value of the variables if we are just asked the value of an expression.

Exception 3: A quadratic equation doesnt always mean two solutions

What is the value of x?

(1) 2x 28 = 5(x 11)

(2) 2x 2 = 36x 162

(A) Statement (1) ALONE is sufficient, but statement (2) ALONE is not sufficient to answer the
question asked.

(B) Statement (2) ALONE is sufficient, but statement (1) ALONE is not sufficient to answer the
question asked.

(C) BOTH statements (1) and (2) TOGETHER are sufficient to answer the question asked, but
NEITHER statement ALONE is sufficient to answer the question asked.

www.manhattanreview.com 19992016 Manhattan Review


Concepts of Data Sufficiency 51

(D) EACH statement ALONE is sufficient to answer the question asked.

(E) Statements (1) and (2) TOGETHER are NOT sufficient to answer the question asked, and
additional data specific to the problem are needed.

Solution:

Clearly, Statement 1 is sufficient since it is an equation in one variable. Solving the equation,
we can get the value of x. Therefore, Statement 1 alone is sufficient.

Now, if you are too afraid of spotting an exception even in this case, dont be! There is not go-
ing to be any exception to one equation in one variable. The only way single-variable equation
will not produce a solution is when the x on both the sides cancels out i.e. if we have the same
co-efficient of x on both sides. Clearly, we do not have that in this case. On the left side, the
co-efficient of x is 2, and on the right side, the co-efficient of x is 5. Even this you need not
check because GMAT is not going to provide an equation in one variable in which the variable
cancels out. Why? Because the provision of that equation contains no information at all. It will
be like saying 5 = 5, or 0 = 000 , or something like that. Clearly, there is no information in
these equations. Therefore, no such equation (in which the only variable cancels out) will be
given as an information statement in a GMAT DS question.

Now, when you look at Statement 2, you see that it is a quadratic equation. Generally, a
quadratic equation produces two solutions. So, you figured out this statement alone may not
be sufficient, and since Statement 1 alone is sufficient, the answer should be option A.

Not so, in this case.

Even though a quadratic equation produces two solutions, the two solutions may not always
be different. In other words, a quadratic equation may produce just one solution, as in the
case above. Let try to solve the equation.

We have

2x 2 = 36x 162

We can see that the whole equation can be divided by 2 while retaining integral co-efficient. So,
we divide the whole equation by 2 and bring everything on the left side, we have

x 2 18x + 81 = 0

If you factorize this equation, you get

(x 9) (x 9) = 0

Therefore, we have x 9 = 0, which gives x = 9.

Thus, from this quadratic equation, we get a unique value of x. Therefore, Statement 2 alone
is also sufficient to answer the question asked.

19992016 Manhattan Review www.manhattanreview.com


52 Concepts of Data Sufficiency

Since both Statement 1 and Statement 2 individually are sufficient to answer the question, the
correct option is D.

In this section, we have looked at three cases of exceptions to the common rules/concepts.
There are exceptions to other rules and concepts too. And you may not falter in all excep-
tional cases, but you may falter in some of them. It is recommended that you keep track of the
cases in which you falter so that you learn from your mistakes. You can keep track of these
exceptional cases through an error log, which we recommended keeping in the earlier section
also.

Before, we move onto the next section, lets do one more question.

There are 40 balls of three different colors in a box: blue balls, red balls, and black balls.
How many black colored balls are there in the box?

(1) The ratio of the number of blue balls to the number of red balls is 19:7

(2) The ratio of the number of red balls to the number of black balls is 1:2

(A) Statement (1) ALONE is sufficient, but statement (2) ALONE is not sufficient to answer the
question asked.

(B) Statement (2) ALONE is sufficient, but statement (1) ALONE is not sufficient to answer the
question asked.

(C) BOTH statements (1) and (2) TOGETHER are sufficient to answer the question asked, but
NEITHER statement ALONE is sufficient to answer the question asked.

(D) EACH statement ALONE is sufficient to answer the question asked.

(E) Statements (1) and (2) TOGETHER are NOT sufficient to answer the question asked, and
additional data specific to the problem are needed.

Solution:

Lets first represent the information in the question in the form of an equation. Lets say there
are x blue balls, y red balls, and z black balls.

From the question stem, we get that

x + y + z = 40

The question asks us to calculate the value of z.

Now, when we look at the information statements, we see that the information statements
present the relationships between the number of one color of balls and that of another color

www.manhattanreview.com 19992016 Manhattan Review


Concepts of Data Sufficiency 53

of balls. Now, since we have only one equation, we think that we need the relationship be-
tween the numbers of all three colors of balls so that we can convert the equation to that in
one variable. Then, well be able to get the value of z.

However, this case presents an exception to our understanding.

From Statement 1, we get that

x 19
=
y 7
Now, given this ratio, what are the possible values of x?

We know that x can only be a positive integer since it represents the number of balls. There-
fore, given the ratio, x can be 19, 38, 57, and so on. However, we also observe that the total
number of balls is 40, and since y needs to be at least 7 for the ratio to hold true, x has to be
less than or equal to 33. From all these constraints, the only possible value for x is 19. And
if x = 19, from the given ratio, we get y = 7. And then we have z = 14 by substituting the
values of x and y in the original equation. Therefore, we can see that there is a unique value
of z possible from Statement 1. Thus, Statement 1 is sufficient to answer the question asked.

Statement 2 says

y 1
=
z 2
Now, many possibilities exist with this statement. Two of the possibilities are x = 37, y =
1, z = 2 AND x = 34, y = 2, z = 4. We can see that this statement does not produce a unique
solution i.e. a unique value of z for the question. Therefore, Statement 2 alone is insufficient.

Therefore, the answer is option A.

This example was presented to drive home the point that many other kinds of exceptions are
possible other than what we have studied in this section. So, stay alert for exceptions and not
get complacent that there are only a limited kinds of exceptions possible.

19992016 Manhattan Review www.manhattanreview.com


54 Concepts of Data Sufficiency

2.8 A couple of strategies for solving DS Questions


In this last section of this guide, well talk about a couple of strategies that you can use to
solve DS questions. Even though we have talked about both the strategies in the questions we
have done so far, it is important to have a relook at them since they come in really handy in
solving DS questions as DS questions and not as PS questions i.e. we utilize the understanding
in these strategies that our goal in DS is not to arrive at an answer but to figure out whether
an answer exists.

Strategy 1: Using numbers to prove insufficiency


We have discussed this before that we can prove the insufficiency of a statement by taking two
cases (or set of numbers) which produce different answers to the questions. Once we have
more than one answer possible to the question, we know that the statement is not sufficient
to answer the question.

However, we cannot prove the sufficiency of a statement by taking cases or numbers. The suffi-
ciency can only be proven conceptually or by solving the question and getting a unique answer.

Lets do a question.

Is m2 m4 ?

(1) |m| > 4

(2) m > 0

(A) Statement (1) ALONE is sufficient, but statement (2) ALONE is not sufficient to answer the
question asked.

(B) Statement (2) ALONE is sufficient, but statement (1) ALONE is not sufficient to answer the
question asked.

(C) BOTH statements (1) and (2) TOGETHER are sufficient to answer the question asked, but
NEITHER statement ALONE is sufficient to answer the question asked.

(D) EACH statement ALONE is sufficient to answer the question asked.

(E) Statements (1) and (2) TOGETHER are NOT sufficient to answer the question asked, and
additional data specific to the problem are needed.

Solution:

Statement 1 tells us that the magnitude of m is greater than 4. Now, if such a number is
squared, it will increase in magnitude. Therefore, m2 will always be greater than m. Now,
when we square m2 (which is greater than 16), well get m4 , which will be even greater than
m2 . Therefore, the answer to the question is YES, and this is the only possibility. Since,

www.manhattanreview.com 19992016 Manhattan Review


Concepts of Data Sufficiency 55

Statement 1 provides a unique answer to the question, it is sufficient.

Statement 2 tells us that m > 0. Now, if one tests out values 1, 2, 3, . . . , m4 will always be
greater than m2 (equal only in the case when m = 1). It seems to indicate that the statement is
true since it produces only one answer i.e. YES. However, we havent tested values less than 1.
What if m = 0.1? In such a case, m2 = 0.01 and m4 = 0.0001. Clearly, m4 is less than m2 in
this case. Therefore, we can see that this statement allows cases in which the answers is YES
as well as cases in which the answer is NO. Thus, this statement is not sufficient to answer the
question.

Therefore, the answer is option A.

The point to understand from this question is that unless one tests the values less than 1 in
Statement 2, one will not be able to reject this statement as insufficient. Many a time we end up
testing only integers or positive numbers, but since we are looking for a case that will produce
a different answer, we should look for different kinds of numbers, not just different numbers.
Following are some of the different kinds of numbers you can test out while assessing the
sufficiency of a statement:

Large numbers (10, 100, 1,000,000. . . ) AND Small numbers (0.1, 0.01, 0.0001. . . )

Positive numbers (1, 7, 19, 24. . . ) AND Negative numbers (1, 7, 19, 20 . . . )

Prime numbers (2, 3, 5, 7. . . ) AND Composite numbers (4, 6, 8, 9. . . )

Zero (0) and Non-zero numbers (1, 2, 1, 2, 1.01 . . . )

Odd numbers (1, 3, 5, 1, 3 . . . ) and Even numbers (2, 4, 6, 2, 4 . . . )

Before we move on to the second strategy, lets do one more question.

Is 2x < 0.01?

(1) x < 5

(2) x > 10

(A) Statement (1) ALONE is sufficient, but statement (2) ALONE is not sufficient to answer the
question asked.

(B) Statement (2) ALONE is sufficient, but statement (1) ALONE is not sufficient to answer the
question asked.

(C) BOTH statements (1) and (2) TOGETHER are sufficient to answer the question asked, but
NEITHER statement ALONE is sufficient to answer the question asked.

(D) EACH statement ALONE is sufficient to answer the question asked.

19992016 Manhattan Review www.manhattanreview.com


56 Concepts of Data Sufficiency

(E) Statements (1) and (2) TOGETHER are NOT sufficient to answer the question asked, and
additional data specific to the problem are needed.

The answer is choice E.

Many test takers limit themselves to test only three values of x i.e. 1, 0, and 1. They believe
that these three values test three different kinds of numbers: positive, negative, and zero and
hence, should constitute enough test cases. However, testing only these values may not always
yield the correct answer, as is the case with this question. Even if you test 2 and 2 as values
of x, you are bound to get the answer wrong to this question.

Only if you also test values lower than 7 (along with other values such as 1, 2, 0, or 1,
etc) will you get the correct answer to this question.

For example: In statement 1, if you test x = 4 and x = 8

Case: x = 4

2x = 24 = 16

In this case, the inequality 2x < 0.01 is not satisfied, therefore the answer is NO.

Case: x = 8

1 1
2x = 28 = , which is <
256 100
In this case, the inequality 2x < 0.01 is satisfied, therefore the answer is YES.

Therefore, we can see that Statement 1 is insufficient to answer the question. However, you
wouldnt find Statement 1 insufficient if you restrict yourself to values 1, 0, and 1 or even if
you include 2 and 2 also. You need to also look at values less than or equal to 7 because in
these cases, the inequality begins to hold.

Similarly, in Statement 2, you can test with values x = 7 and x = 0 and see that these two
cases produce different answers. Therefore, Statement 2 alone is insufficient.

Now, even if you combine Statements 1 and 2, you have

10 < x < 5

This situation also allows x = 8 and x = 4, the values we tested above and found producing
different answers. Therefore, Statements 1 and 2 together are insufficient to answer the ques-
tion.

Now, one may ask: how do we know that we need to test values less than 7 to get a different
answer (i.e. YES the inequality is satisfied) in this question?

www.manhattanreview.com 19992016 Manhattan Review


Concepts of Data Sufficiency 57

The answer is that you need to conceptually understand the question to figure out the thresh-
old values around which the answer to the question changes. We need to keep in mind that
while testing values, we are trying to find values which produce different answers to the ques-
tion and thus render the statement(s) insufficient.

Now, in this case, we have to answer whether 2x < 0.01. Now, it is easier to see that as we
decrease the value of x, the value of 2x goes down. We have

1
21 =
2
1
22 =
4
1
23 =
8
And so on...

1 1
Since we know that 0.01 = , the inequality 2x < 0.01 will be satisfied when 2x <
100 100
Understanding this, you should know that you should test more negative values of x.

Statement 1 allows you positive values of x (less than 5), in which case the inequality is ob-
viously not satisfied. But it also allows infinitely large negative values of x. Now, if you have
conceptually understood the question, you need not even test out with large negative values
of x, since you can see that at some large negative values of x, the inequality will be satisfied.
Therefore, Statement 1 is insufficient since it allows both kinds of values of x: in which the
inequality is satisfied and in which the inequality is not satisfied.

Statement 2 allows you positive values of x, in which case the inequality is obviously not
satisfied. But it also allows negative values of x till 10. Now, in this case, since you have
conceptually understood that the inequality will be satisfied for large negative values of x, you
should start with x = 9. When you test with this value of x, youll see that the inequality is
satisfied. Therefore, Statement 2 will be found insufficient since it allows both kinds of values
of x: in which the inequality is satisfied and in which the inequality is not satisfied.

Given the test cases we have taken while individually testing the statements, we can easily see
that combining the two statements will not help. Therefore, the answer is choice E.

The key takeaway from this question is that you should not restrict yourself to only a few test
values but rather try to conceptually understand the question to figure out the values at which
the question may produce different answers.

Strategy 2: Conceptual approach

Is integer x a prime number?

(1) x = 10! + y, where 1 < y < 10

19992016 Manhattan Review www.manhattanreview.com


58 Concepts of Data Sufficiency

(2) x is a multiple of 5

(A) Statement (1) ALONE is sufficient, but statement (2) ALONE is not sufficient to answer the
question asked.

(B) Statement (2) ALONE is sufficient, but statement (1) ALONE is not sufficient to answer the
question asked.

(C) BOTH statements (1) and (2) TOGETHER are sufficient to answer the question asked, but
NEITHER statement ALONE is sufficient to answer the question asked.

(D) EACH statement ALONE is sufficient to answer the question asked.

(E) Statements (1) and (2) TOGETHER are NOT sufficient to answer the question asked, and
additional data specific to the problem are needed.

Solution:

In Statement 1, if y is a non-integer, then x cannot be an integer, which invalidated the


information given in the question stem, so we must consider, though not stated in State-
ment 1, that y is an integer. Now since y is an integer, it has to be one from the set
{2, 3, 4, 5, 6, 7, 8, 9}. Now, we know that 10! is divisible by all the numbers from this set since
by definition 10! = 1 2 3 4 9 10. Therefore, whatever be the value of y, the ex-
pression 10! + y will be divisible by y. Thus, this expression cannot be a prime number (since
it is divisible by a number other than itself and 1). Therefore, this statement is sufficient since
it provides a unique answer NO to the question.

Statement 2 says that x is a multiple of 5. Since x is a multiple of some number, it seems that
it shouldnt be a prime number. Unless we consider the case when x = 5. If x = 5, then x is
a prime number. If x = 10, then x is not a prime number. Therefore, this statement provides
different answers to the question. Thus, this statement is not sufficient to answer the question.

Therefore, the answer is option A.

Now, the point to understand from this question is that in Statement 1, we did not calculate
all the possible values of x (corresponding to 8 possible values of y) to see if it can ever be
prime. We approached the problem conceptually by looking at the numbers involved and ar-
guing whether they can ever sum to a prime number.

Such an approach in which we obviate the need of calculations can be called the conceptual
approach. This is an approach that is specific to DS questions since we dont need to calculate
the answers in DS questions. It is recommended that you use this approach wherever you
deem it appropriate to use.

However, it should also be kept in mind that some DS questions need to be solved completely
like PS questions in order to find out the sufficiency of the statements. But such questions
arent common, and therefore, you can avoid the calculations in most of the DS questions.

www.manhattanreview.com 19992016 Manhattan Review


Concepts of Data Sufficiency 59

An important observation: information statements are true and thus


consistent!
Now, lets suppose you are solving a DS question and you see that each of Statement 1 and
Statement 2 produces unique answer to the question, but the answers are different. For ex-
ample: in a Yes/No question, one statement is producing a unique YES, and the other is
producing a unique NO. In such a scenario, would you mark choice D as the answer since
each statement is producing a unique answer to the question? Or would you mark choice E as
the answer since you cannot find a unique answer by combining the two statements?

Well, you shouldnt do any of these. What you should do is try to locate the error in your
analysis! It is for sure that you have made an error. Now, you just need to locate it.

The reason we can confidently say that you have made an error in this scenario is that in GMAT
questions, two statements cannot produce different unique answers to the question. If each
statement produces a unique answer, the answer has to be the same in both the statements.
For example: if one statement produces x = 4, then if the other statement also produces a
unique value of x, it also needs to produce x = 4. The other statement cannot produce a
unique value of x different from 4.

Now, you may ask why. Why do both statements need to produce the same unique answer?
The reason is that both information statements are true (GMAT doesnt believe in spreading
lies!). Now, if both information statements are true, then they cannot produce two different
unique answers because there can be only one answer to the question. We may not know the
answer, and thus we may be confused between two or three possible answers. But it doesnt
mean that there are two or three answers to the question.

Therefore, since both the information statements are true, they need to be consistent i.e. they
need to provide a consistent answer.

Therefore, if you find that you are getting different unique answers from the two statements,
you can be sure that you have made a mistake in your analysis or calculations.

In the next chapters, you will find 150 GMAT-like quants questions. Followed by the question,
you will find a special section Hint. Hint is a minimum help for a question. It is one to three
lines generic or specific statements. Best of luck!

19992016 Manhattan Review www.manhattanreview.com


60 Concepts of Data Sufficiency

www.manhattanreview.com 19992016 Manhattan Review


Chapter 3

Practice Questions

61
62 Questions

1. A bag contains a mixture of beans and pulses. To achieve 20 percent beans in the mix-
ture, what percent of the mixture should be taken out and replaced with pulses?

(1) The mixture originally has 40 percent beans and 60 percent pulses.
(2) Total quantity of the mixture is 20 lb.

Solve yourself:

2. A nut-mix contains peanuts and cashews. Cashews are more expensive than peanuts by
what percentage?

(1) The nut-mix contains 10% peanuts.


(2) The nut-mix costs 20% more than pure peanuts.

Solve yourself:

3. What is the value of (r + s) as a percent of p?



(1) r is 20 percent of p + s .
(2) s is 10 percent of r .

Solve yourself:

4. A salesman normally gets a fixed salary of $500 and a commission of 5 percent on sales
above $1000 per month. If the pay structure of the salesman was changed in March,
what were the sales of the salesman in March?

(1) In March, the salesman had received a fixed salary of $600 and a commission of 4
percent on sales above $800.
(2) In March, he received $104 more than he would have received according to his
earlier pay structure.

www.manhattanreview.com 19992016 Manhattan Review


Questions 63

Solve yourself:

5. In a class, 30 percent of the boys read The Telegraph every day. Boys constitute 60 per-
cent of the class and 40 percent of the whole class reads The Telegraph. What percentage
of the class reads only gossip pages or only cartoons?

(1) Among the boys who read The Telegraph, 40 percent read only the cartoons and the
rest read only the sports pages.
(2) Among the girls who read The Telegraph, 60 percent read only the gossip pages and
the rest read only the fashion pages.

Solve yourself:

6. To increase revenue, a soft-drink company changed the price of and quantity of drink in
a bottle. What is the percent change in revenue earned for every 100 ml of soft drink?

(1) The company reduced the price of a $5 bottle by giving a discount of 4 percent.
(2) The company reduced the quantity of drink in the bottle from 500 ml to 400 ml.

Solve yourself:

7. Two toy stores, A and B, sell Hot-Wheels. The regular price at store A is 80% of the
manufacturers suggested retail price. Store B normally does not offer any discount on
the manufacturers suggested retail price. During Christmas, both stores offer special
discounts. What is the difference between the prices of the toys of the two stores?

(1) Store A offers a special discount of 20% on its regular price and finally sells at $64.
(2) Store B offers 40% discount on its regular price.

19992016 Manhattan Review www.manhattanreview.com


64 Questions

Solve yourself:

8. At what price was the article sold?

(1) The article was sold at 10 percent profit.


(2) Had the article been sold for $150 more than what it was sold, the profit would have
been 12 percent.

Solve yourself:

9. Is the percent increase in the profit made on the articles in the store after the old man-
ager was replaced by a new one greater than 100 percent? Assume that all articles are
identical.

(1) Articles were initially sold by the old manager at a price resulting in 10% profit for
the store.
(2) After the old store manager was replaced by a new one, articles were sold at a price
resulting in 80% profit for the store.

Solve yourself:

10. What is the percent profit made on selling an article?

(1) The percent profit, when calculated on the selling price of the article is 20 percent.
(2) The selling price is arrived at by increasing the cost price of the article by 50 percent
and then offering a 16.67 percent discount.

Solve yourself:

www.manhattanreview.com 19992016 Manhattan Review


Questions 65

11. What is the average age of the two children?

(1) In 2003, the average age of a family of four adults was 25 years.
(2) In 2013, the average age of the family of four adults and two children was 25 years.

Solve yourself:

12. How many fruits did each girl get if fruits are divided in two parts: one part is distributed
equally among some boys and the other part is distributed equally among some girls?
Assume that none of the fruits had to be cut in pieces.

(1) The number of fruits received by each girl was 2 more than the number of fruits
received by each boy.
(2) There are total 74 fruits and the number of boys is 6 more than the number of girls.

Solve yourself:

13. If a, b, c and d are distinct numbers, can b be the average of a, b, c and d?

(1) a + c = 2d
(2) b + d = 2c

Solve yourself:

19992016 Manhattan Review www.manhattanreview.com


66 Questions

14. The average marks obtained in a test by x students of a class was m. Was (x + 3) the
average marks obtained by three students with the lowest marks in the same test?

(1) If the lowest three marks were excluded, the average score of the class would have
increased by three.
(2) m = 2x.

Solve yourself:

15. P is a set of integers. If a is the average of the odd integers and b is the average of the
even integers in set P, what is the value of |a b|?

(1) P consists of consecutive integers starting from 1.


(2) There are not more than 50 integers in the set P.

Solve yourself:

16. A man collected four different samples of apple juice of four different brands, P, Q, R
and S. He mixed the juices of exactly two brands to achieve a 60 percent concentration.
If samples of P, Q, R and S are available in units of 1 milliliter, was the least total cost of
the mixture obtained by mixing samples P and Q?

(1) The concentrations of the samples are:

Sample name P Q R S

Concentration (%) 66 48 60 54

(2) The prices per milliliter of the samples are:

Sample name P Q R S

Price per ml $1 $4 $7 $4

Solve yourself:

www.manhattanreview.com 19992016 Manhattan Review


Questions 67

17. A chemical agent is kept in bottles having 50 percent concentration of a particular chemi-
cal. A part of the chemical agent was kept aside and equal quantity of water was added to
the remaining chemical agent. The chemical now forms what percentage concentration
in the final mixture?

(1) Each bottle of the chemical agent contains 60 ml of the mixture.


(2) 60 percent part of the chemical agent in the bottle was kept aside and equal quantity
of water was added to the remaining chemical agent.

Solve yourself:

18. A person mixed three varieties of tea priced at $120 per pound, $135 per pound and
$160 per pound. In what ratio did he mix the three varieties of tea?

(1) The price of the mix was $135 per pound.


(2) Only 3 pounds of the variety priced at $135 per pound was used.

Solve yourself:

19. Three containers A, B and C have wine concentrations a, b and c, respectively, such that
a > b > c. If quantities p, q and r , of wine are taken from the three containers A, B and
C, respectively, and mixed, is the concentration of the resulting mixture greater than b?

(1) ab =bc
(2) p>q>r

Solve yourself:

19992016 Manhattan Review www.manhattanreview.com


68 Questions

20. A mixture consists of two spirits A and B; A evaporates at the rate of 2 ml per minute
and B at 3 ml per minute. What will be the volume of the mixture after 10 minutes?

(1) Initial ratio of A and B in the mixture was 4 : 5.


(2) The final ratio of A and B in the mixture was 5 : 4.

Solve yourself:

!
wx
21. If w, x, y and z are positive, is the value of an integer?
yz

3 y
(1) w= x= = 3z
4 2
(2) w + x + y + z = 34

Solve yourself:

22. What is the value of k?


a 2b 3a 4b
(1) = =
4 5 k
(2) a + b = 2k

Solve yourself:

www.manhattanreview.com 19992016 Manhattan Review


Questions 69

23. A man purchased some pens, pencils and erasers. Can the number of erasers purchased
be 3?

(1) The ratio of the number of pens to the number of pencils was the same as the ratio
of the number of pencils to the number of erasers.
(2) Total number of items purchased was 21.

Solve yourself:

24. In an election between two candidates A and B, only 75 percent of the voters decided to
cast their votes, in the ratio 3 : 2 for A and B, respectively. The remaining voters were
undecided on which candidate to vote. However, when a third candidate C also decided
to contest the elections, the remaining voters 25 percent of the voters decided to vote for
A, B, or C, and some of the voters who had initially decided to vote for A, changed their
opinion. If each voter is eligible to vote for only one candidate, does C win the election?

(1) Among the voters who had initially decided not to vote, 40 percent voted for candi-
date C.
(2) Among the voters who had decided to vote for candidate A, 80 percent voted for
candidate C.

Solve yourself:

25. Three friends contributed some funds for a party. What was the amount contributed by
the friend who contributed the highest amount?

(1) The total funds collected were $510.


(2) Had each friend contributed $10 extra, the ratio of their contributions would have
been 2 : 3 : 4.

Solve yourself:

19992016 Manhattan Review www.manhattanreview.com


70 Questions

26. 50 equally efficient people were employed to complete a work in 90 days. How long did
it actually take to complete the work?
(1) After 30 days, the 50 people had completed only 25 percent of the work and worked
at the same rate thereafter.
(2) Had 25 additional people, each having an efficiency equal to each of the initial 50
people, been employed after 30 days, the work would have been completed on time.
Solve yourself:

27. For how many days are rations expected to last in a camp of 100 soldiers? Assume that
each soldier consumes equal quantities of rations per day.
(1) If 100 additional soldiers join the camp after 13 days, each consuming the same
quantity of ration as the initial soldiers, the rations would last another 12 days.
(2) If 20 soldiers leave the camp in the very beginning, the rations would last for 25
percent more number of days.
Solve yourself:

28. A, B and C can complete a piece of work in 24 days, 40 days and 60 days, respectively.
In how many days was the piece of work completed?
(1) A, B and C started working together, and A continued to work till the end.
(2) A, B and C started working together, B and C left the work 2 days and 7 days,
respectively, before the completion of the work.
Solve yourself:

29. Two men, A and B, are assigned to complete a piece of work. A and B started working to-
gether and both worked till the end. How long would A, working alone, take to complete
the work?

www.manhattanreview.com 19992016 Manhattan Review


Questions 71

(1) 11 hours after A and B had started working, 45 percent of the work was still remain-
ing.
(2) After the work was completed, it was found that A had done 60 percent of the work.
Solve yourself:

30. 4 men and 10 women were assigned to complete a work. Can they complete the work
within 15 days? Assume that all men are equally efficient and all women are equally
efficient.
1
(1) 4 men and 10 women employed initially completed of the work in less than 6
3
days.
(2) If 2 additional men and 2 additional women had been employed, they would have
1
completed of the work in 5 days (the additional men and the additional women
2
have efficiencies equal to the men and women employed initially, respectively).
Solve yourself:

31. A man travelled from one city to another. What is the distance between the two cities?
1
(1) He travelled the first of his distance at a uniform speed of 15 miles per hour.
4
Thereafter, he increased his speed to 20 miles per hour.
(2) The time for which he travelled at 20 miles per hour was 5 hours greater than the
time for which he travelled at 15 miles per hour.
Solve yourself:

32. Two friends, X and Y start from two points M and N and move towards each other and
meet at a point P on the way. How much distance has Y travelled up to the meeting
point?

19992016 Manhattan Review www.manhattanreview.com


72 Questions

(1) X and Y travel at speeds of 30 miles per hour and 20 miles per hour, respectively.
(2) Had Y travelled at 20 percent higher speed, X and Y would have met at a point which
is 10 miles away from P.

Solve yourself:

33. X and Y start traveling in the same direction, from a particular point, at 7:00 am and 7:45
am, respectively. At what time do they meet?

(1) Speed of Y is double that of X.


(2) After 5 hours, Y is 10 miles ahead of X.

Solve yourself:

34. A man, while driving on a highway at uniform speed, observes his cars odometer reading
just turned to 18992 miles. What is the speed at which the man was driving?

(1) After 5 hours, the odometer reading of the car showed a five-digit palindrome as the
total distance travelled (A palindrome is a number which reads the same forward as
well as backward).
(2) After 2 hours, the man found that he had travelled less than 80 miles.

Solve yourself:

35. The distance between two cities, A and B, is 900 miles. A bus starts from A for B at 7:00
am. Another bus starts from B for A at 8:30 am. What is the speed of the first bus?
4
(1) The speed of the second bus is of the speed of the first bus.
7
(2) The two buses cross each other at 7:00 pm on the same day.

www.manhattanreview.com 19992016 Manhattan Review


Questions 73

Solve yourself:

36. A man invests two equal sums of money in two banks, both under simple interest. What
is the rate of interest in the second bank?

(1) The interests obtained from the two banks after 2 years and 3 years, respectively
are in the ratio 4 : 5.
(2) The rate of interest in the first bank is 6 percent per annum.

Solve yourself:

37. A man invests $1400 in a bank under simple interest. What was the final value of the
investment after 3 years?

(1) Had the investment been done for 2 years, the amount would have been $1600.
(2) Had the rate of interest been doubled, the amount after three years would have been
$300 greater than the actual interest obtained.

Solve yourself:

38. A man took a loan under simple interest. What was the rate of interest?

(1) The number of years for which the loan was taken was the same as the rate of
interest.
(2) The final value of the loan after the duration of loan was 9 percent higher than the
amount taken as loan.

Solve yourself:

19992016 Manhattan Review www.manhattanreview.com


74 Questions

39. A man invested two different sums of money, under the same rate of simple interest, but
for different periods of time. The total interest he obtained from both the investments
was $2,250. What was the rate of interest?

(1) The sums of money invested are $12,000 and $9,000, respectively.
(2) The durations for which the sums are invested are 3 years and 5 years, respectively.

Solve yourself:

40. A man invests a sum of money in a bank in the beginning of a year and another sum of
money, after an integer number of months, in another bank, both under simple interest.
After how many months from the beginning of the year did he invest the second sum of
money?

(1) The sums of money invested in the two banks are in the ratio 1 : 6, respectively.
The ratio of rates of interest applicable in the two banks are in the ratio 3 : 4,
respectively.
(2) The total interest accumulated after the year end was 4 percent of the total invest-
ment made in the year.

Solve yourself:

41. The difference between the simple interest after 2 years and the compound interest for
1 year on the same sum of money is $111. What is the sum of money?

(1) The rate of interest for simple interest is 5 percent per annum.
(2) The rate of interest for compound interest is 3 percent greater than the rate of
simple interest per annum.

www.manhattanreview.com 19992016 Manhattan Review


Questions 75

Solve yourself:

42. Two equal sums of money are invested, one under simple interest and the other under
compound interest, both at the same rate of interest per annum. What is the rate of
interest?

(1) The difference between the simple interest and compound interest after 2 years is
$110.
(2) If the rate of interest were squared, the interest accumulated under simple interest
after 2 years would have become 4 times its actual value.

Solve yourself:

43. A sum of money is invested under compound interest for a few years. After how many
years will the sum of money become nine times its present value?

(1) The sum of money invested under compound interest at the same rate of interest
per annum became thrice its value in 6 years.
(2) The sum of money, under compound interest, at the same rate of interest per an-
num, would become twenty-seven times its present value in 9 years.

Solve yourself:

44. Grass on a lawn grows everyday by a fixed percent of the height on the previous day.
However, cows graze and reduce the height of the grass by 1 mm every day. What was
the height of the grass on the first day before the cows started grazing? Assume that
the cows graze at the end of the day, after which the grass grows in the beginning of the
next day.

(1) The grass grows every day by 2 percent of the height on the previous day.
(2) The grass is completely grazed in 3 days.

19992016 Manhattan Review www.manhattanreview.com


76 Questions

Solve yourself:

45. What is the rate of interest per annum at which sum of money is invested under com-
pound interest?

(1) The interest accumulated on the sum of money in the first year is $120.
(2) The interest accumulated on the same sum of money in the second year is $144.

Solve yourself:

46. A survey was made of 100 students who study German, French and Japanese. How many
students study both German & French but not Japanese?

(1) 51 students study German, while 31 students study German but not French.
(2) 9 students study German, French and Japanese.

Solve yourself:

47. In a class of 50 students, everyone plays at least one game among baseball, football and
basketball. How many play only baseball?

(1) 60 percent of the students do not play basketball or football.


(2) 4 percent of the students play all three games.

Solve yourself:

www.manhattanreview.com 19992016 Manhattan Review


Questions 77

48. In a class of 120 students, how many passed in only one subject?

(1) 70 students have passed in English.


(2) 80 students have passed in Mathematics.

Solve yourself:

49. In a certain institute, every student studies at least one of two subjects, Accountancy and
Economics. What is the percentage of students who take only Economics?

(1) Out of every seven students taking Accountancy, three take Economics as well.
(2) For every three students taking at least one of the two courses, there is one student
who takes up both.

Solve yourself:

50. A safari is held twice in a day either in the morning or in the evening. A person is allowed
to participate in the safari in both the timings. A total of 108 people participate in the
safari. The number of men and women in the safari are in the ratio 5 : 4. The total
number of men who participate in the safari in the morning is 50 percent of the total
number of men who participate in the safari. If every person participated in the safari
at least once, what is the number of people who participate in the safari both in the
morning and in the evening?

(1) The number of men who participate in the safari only in the morning is 80 percent
of the women who participate in the safari only in the morning. Also, the number
of men who participate in the safari only in the evening is double the number of
women who participate in the safari only in the evening.
(2) The number of women who participate in the safari only in the evening is 60 percent
of the girls who participate in the safari only in the morning.

Solve yourself:

19992016 Manhattan Review www.manhattanreview.com


78 Questions

51. If f (x) is a linear function, what is the value of f (2)?


(1) f (f (x)) = 9x + 8
(2) f (f (2)) = 26
Solve yourself:

52. If f (a + b) = f (a) + f (b), what is the value of f (5)?


(1) f (2) = 4
(2) f (2a) = 2f (a)
Solve yourself:

53. If f (x) is a function, what is the value of f (2)?


 2
(1) f (x) = 2f (x) 1
(2) f (4x) = 8x 2 1
Solve yourself:

54. If f (x) is a linear function, what is the value of f (1)?


(1) f (x 3) = f (x) + 1
(2) f (5) = 2f (3)
Solve yourself:

www.manhattanreview.com 19992016 Manhattan Review


Questions 79

55. If f (x) and g (x) are two functions, what is the value of g (2)?

(1) f (x 1) = 2x + 4

(2) f g (x) = 4x

Solve yourself:

56. N is a two-digit number. The sum of its digits is S and the product of its digits is P . What
is the largest possible value of N?

(1) N + S = 103

(2) 2N = 2S + 9P

Solve yourself:

57. P and Q have few marbles. How many marbles does P have?

(1) P has 10 marbles greater than what Q has.

(2) If P gives away 10 marbles to Q, he will have 10 marbles less than what Q has.

Solve yourself:

58. The equations ax + 2y = 6 and bx + cy = 9 have infinite solutions. What is the value of
(a + b)?

(1) c=3

(2) b =a+1

19992016 Manhattan Review www.manhattanreview.com


80 Questions

Solve yourself:

59. A, B and C have a certain number of marbles. Can C have EXACTLY 10 marbles more
than what A has?
1
(1) A has of what B and C together have.
3
1
(2) B has of what A and C together have.
4
Solve yourself:

60. Is the price of an apple greater than that of an orange?

(1) The price of 10 apples and 15 oranges is $8.


(2) The price of 5 apples is $1.30 greater than the price of 6 oranges.

Solve yourself:

61. The average (arithmetic mean) of 10 distinct odd integers in a set is 55. What is the
median of these 10 integers?

(1) The largest integer is 65.


(2) The smallest integer is 41.

Solve yourself:

www.manhattanreview.com 19992016 Manhattan Review


Questions 81

62. The range of 6 positive integers is 12. What is the value of the smallest number?

(1) The average (arithmetic mean) of the 6 integers is 50.


(2) The numbers have the least possible standard deviation.

Solve yourself:

63. Numbers 1 to 8 are grouped four at a time to form two groups, so that there is no number
common to any group. What is the difference between the medians of the two groups?

(1) The numbers are grouped such that the mean of each group is the same.
(2) The numbers are grouped such that 1 and 8 do not fall in the same group.

Solve yourself:

64. Numbers 1 to 16 are grouped four at a time so that there is no number common to any
group. What is the sum of the medians of all four groups?

(1) The numbers are grouped such that, in any group, there is a constant difference of
2 between consecutive numbers in the group.
(2) The numbers are grouped such that, in any group, there is a constant difference of
4 between consecutive numbers in the group.

Solve yourself:

1
65. The average (arithmetic mean) of 6 positive integers is 15 . What is the largest integer?
2
(1) The integers are all distinct.
(2) The smallest integer is 13.

19992016 Manhattan Review www.manhattanreview.com


82 Questions

Solve yourself:

66. What is the value of x?



(1) 2x + 2 x 3 = 2
(2) x>0

Solve yourself:

67. The roots of the


 quadratic
 equation ax 2 + bx + c = 0 are p and q (p 6= q). What is the
value of p + 1 q + 1 ?

(1) bc =2
(2) a=3

Solve yourself:

68. What is the value of x?

(1) 4x 20.2x + 64 = 0
(2) 4x = x 4

Solve yourself:

69. Is a + b = 0?

www.manhattanreview.com 19992016 Manhattan Review


Questions 83

a b
(1) The roots of the equation + = 1 are equal in magnitude, but opposite
xa xb
in sign.
(2) The equation x (x a) + x (x b) = 0 has equal roots.

Solve yourself:

In the polynomial function f (x) = x 3 px 2 + qx r , what is the value of 3p q ?



70.

(1) Two of the roots of f (x) = 0 are 1 and 2.


(2) One root of f (x) = 0 is 3.

Solve yourself:

71. In how many ways can a password of length 7 characters be designed?

(1) The password only contains distinct vowels and distinct digits, in alternate posi-
tions.
(2) The password must begin with a digit.

Solve yourself:

72. Different code-words are formed using the letters of the word SYSTEMS. How many such
code-words can be formed?

(1) The code-words formed must have 4 letters.


(2) In any code-word, the same letter may be selected as many times it has occurred in
the word SYSTEMS.

19992016 Manhattan Review www.manhattanreview.com


84 Questions

Solve yourself:

73. A few men and a few women are seated in a row. The number of men is one greater than
the number of women. What is the total number of people?
(1) The people are seated so that no two women sit beside one another.
(2) The number of ways of arranging the seating positions of the men and women is
3 (5!) (7!).
Solve yourself:

74. A group consists of a few men and a few women. What is the total number of men and
women in the group?
(1) The ratio of the number of ways in which the people in the group are seated along
a circle so that men and women sit in alternate positions and the number of ways
in which the people in the group are seated in a row so that men and women sit in
alternate positions is 1 : 10.
(2) The number of handshakes possible separately among the men and separately
5
among the women is equal and each is equal to of the number of handshakes
12
possible if the men and women shake hands only with each other such that every
man shakes hands with every woman.
Solve yourself:

75. A number of students have to be selected from a class of 25 students to form a group.
What is the number of students in the group?
(1) The number of possible selections of the students to form the group is the largest
possible value.
(2) The number of students selected is less than 50 percent of the strength of the class.

www.manhattanreview.com 19992016 Manhattan Review


Questions 85

Solve yourself:

76. A boy purchased a few apples and a few oranges from a shop. What is the probability
that he purchased a greater number of apples than oranges?

(1) The cost of an apple is $5 and that of an orange is $2.


(2) The total worth of apples and oranges purchased is $27.

Solve yourself:

77. There are n cards numbered consecutively, from 1 to n. Two cards are drawn. What is
the probability that the number on the first card is less than the number on the second
card?

(1) The two cards are drawn in succession, with replacement.


(2) n = 10.

Solve yourself:

78. A man uses a biased dice to decide whether to go ahead with a particular course of
action. If the dice shows a prime number, he decided to go ahead with the course of
action. What is the probability that the man goes ahead with the course of action?

(1) The probability of the dice showing an even number is double the probability of it
showing an odd number; all odd numbers are equally likely and all even numbers
are also equally likely.
1
(2) The probability of the dice showing the number 1 is less than .
6
Solve yourself:

19992016 Manhattan Review www.manhattanreview.com


86 Questions

79. What is the probability that an integer chosen from a set P, which contains consecutive
positive integers, when divided by 5, leaves a prime remainder?

(1) The value of the range of the integers in set P is 10.


(2) The largest integer in set P is 35.

Solve yourself:

80. In a multiple-choice test, each question is followed by n options. What is the value of n?

(1) The test is designed in such a way that if a candidate appearing for the test, decides
to mark the options for every question in a random manner, the candidate would
expect to receive 0 marks in the test.
(2) For every correct answer, a candidate is awarded 1 mark and for every wrong an-
1
swer, there are marks assigned.
3
Solve yourself:

81. ABC is a triangle with BC = AC = 6. D is a point on AB. What is the length of CD?

(1) ACB = 120 .


(2) CD is the bisector of ACB.

Solve yourself:

www.manhattanreview.com 19992016 Manhattan Review


Questions 87

82. In the diagram below, what is the length of RS?

P T

Q S U

(1) PQ = 28 and TU = 21.

(2) PQ, RS and TU are parallel lines.

Solve yourself:

83. In the diagram below, P = g + c e and Q = a + f . What is the value of (P + Q)?




C D E

(1) BC = CD = DB = DE
1
(2) AB = AE
2

Solve yourself:

84. In the diagram below, triangle ABC is right-angled at B. What is the measure of BAC?

19992016 Manhattan Review www.manhattanreview.com


88 Questions

D
E

B C

(1) BD = BC and AE = AD.


(2) BE = DE.

Solve yourself:

85. In the diagram below, triangle ABC is right-angled at B. D and E are points on AC and AB,
respectively. If BC = 6, what is the area of triangle BPC?
A

E
D
P

B C

(1) BE = BC and BD bisects ABC.


(2) AB = 8.

Solve yourself:

86. In the diagram below, P and Q are the centers of the two circles. What is the area of
triangle XYZ?

www.manhattanreview.com 19992016 Manhattan Review


Questions 89

Y Z

X
P Q

(1) Radius of each circle is 6.

(2) XY = 2.

Solve yourself:

87. In the diagram below, ABCD is a square with a circle inscribed inside. P is a point on
the circle. A rectangle AXPY is constructed with A and P as the opposite vertices of the
rectangle. What is the length of a side of the square?

A Y
B
X P

D C


(1) AP = 2 2

(2) The line AP coincides with the diagonal AC.

Solve yourself:

88. In the diagram below, PQ is a diameter of the circle having center at O. What is the
measure of PTQ?

19992016 Manhattan Review www.manhattanreview.com


90 Questions

T
O

(1) ROS = 40 .
(2) RPO = 55 .

Solve yourself:

89. In the diagram below, a rectangle ABCD is drawn inside a semicircle. What is the length
of AB?
A B

C O D

(1) The radius of the semicircle is 10.


(2) AB : BD = 3 : 2

Solve yourself:

90. In the diagram below, PQR is a triangle, right-angled at Q. A point S is chosen on the
hypotenuse and two perpendiculars, one each on to PQ and QR, are drawn from S such
that they meet PQ and QR at A and B, respectively. What is the area of SAQB as a fraction
of the area of triangle PQR?

www.manhattanreview.com 19992016 Manhattan Review


Questions 91

A S

Q B R

(1) PQ = 24 and QR = 7.
(2) SAQB is a square.

Solve yourself:

91. A right circular cone is such that it perfectly fits inside a hollow sphere. What is the
height of the cone?

(1) The radius of the sphere is 6.


(2) The radius of the cone is 4.

Solve yourself:

92. A piece of thread is uniformly strung around a right circular cylinder so as to form one
complete loop, as shown in the diagram below. What is the length of the thread?

2
(1) The radius of the cylinder is .

(2) The height of the cylinder is 3.

Solve yourself:

19992016 Manhattan Review www.manhattanreview.com


92 Questions

93. Between a cube and a right circular cylinder, does the cube have a higher lateral surface
area? Lateral surface area refers to the area of the sides (the two base areas, one on top
 3
3
and one on bottom, are not taken into account). Assume =
2

(1) The height of the cylinder is twice its radius.


(2) Both the cube and the cylinder have the same volume.

Solve yourself:

94. Is the length of the longest line that can be drawn inside a cube greater than the length
of the longest line that can be drawn inside a right circular cylinder?

(1) Each side of the cube is half the height of the cylinder but twice the radius of the
cylinder.
(2) The sum of the lengths of the radius of cylinder, the height of the cylinder and a
side of the cube is 14.

Solve yourself:

95. The volume of a right circular cylinder equals half the volume of a cube. Does the cylinder
fit inside the cube?

(1) The ratio of height to radius of the cylinder is 2 : 1.


(2) Each side of the cube is less than twice the height of the cylinder.

Solve yourself:

www.manhattanreview.com 19992016 Manhattan Review


Questions 93

96. What is the area of the quadrilateral ABCD shown in the diagram below?
A B

(1) AB = AD = 6.
(2) ADC = 60 .
Solve yourself:

97. What is the length of each side of the regular hexagons as shown in the diagram below?

(1) The shortest distance between A and B is 6.


(2) The shortest straight-line distance between
A and B such that the straight-line does
not enter inside either hexagon is 4 + 2 3.

Solve yourself:

98. ABCDEFGH is a regular octagon. What is the area of triangle ABC?

19992016 Manhattan Review www.manhattanreview.com


94 Questions

A B

H C

G D

F E

(1) AB = 2.
 
(2) AD = 2 2+1 .

Solve yourself:

99. ABCD is a quadrilateral, as shown in the diagram below. What is the minimum possible
sum of areas of triangles AOD and BOC?
A B

D
C

(1) Area of triangles AOB and COD are 16 and 36, respectively.
(2) Area of triangle AOD is equal to the area of triangle BOC.

Solve yourself:

100. A pathway, 6 meters wide, is to be constructed around the boundary of a square lawn
using square stone slabs. If the stone slabs cannot be broken, can the stone slabs be
used to construct the pathway?

(1) Each side of the square lawn is 50 meters.


(2) Each stone slab has side 4 meters.

www.manhattanreview.com 19992016 Manhattan Review


Questions 95

Solve yourself:

101. In the diagram below, AB and CD are parallel to each other. What is the measure of
KXY?

X
A B

K L
C D
Y

(1) LXB and AXK are equal and LXB is 6 greater than YXL.
(2) XY is perpendicular to AB.

Solve yourself:

102. In the diagram below, AB and CD are parallel to each other and XY is perpendicular to
AB. What is the measure of AXL?

X
A B

K L
C D
Y

(1) The measure of YLM is thrice the measure of LXB.


(2) BXL = AXK

19992016 Manhattan Review www.manhattanreview.com


96 Questions

Solve yourself:

103. In the diagram below, what is the measure of BCD?

A B

D E

(1) AB is parallel to DE.


(2) ABC = 550 .

Solve yourself:

104. In the diagram below, what is the measure of the non-reflex angle formed by extending
the lines BD and CE? A reflex angle is one whose measure is greater than 180 but less
than 360 .
A

60#


D x E
B C

(1) x=y
(2) x + y = 40

Solve yourself:

www.manhattanreview.com 19992016 Manhattan Review


Questions 97

105. In the diagram below, AB is parallel to CD and BC is parallel to DF. What is the measure
of CDE?
A B

C D

F
E

(1) ABC = 45
(2) EDF = 40
Solve yourself:

106. What is the value of x?



(1) y 3x
(2) |5x 1| = x + 7

Solve yourself:

107. If x is an integer, what is the value of x?

(1) x |x| 9.
(2) x 2|x| = 729.

Solve yourself:

108. If x + y + z < 1, is z < 1?

19992016 Manhattan Review www.manhattanreview.com


98 Questions

(1) x and y are positive numbers.


(2) xy = 1.
Solve yourself:

109. Is x > y?

(1) |x| > y .
(2) x + y > 0.
Solve yourself:

110. If x and y are integers, is x > 9?


(1) |x| y < 9.

(2) y 1.

Solve yourself:

1
111. Is the slope of the line l equal to ?
5
1
(1) Lines m and n are parallel and sum of their slopes is less than .
2
(2) Line l is perpendicular to line m.
Solve yourself:

www.manhattanreview.com 19992016 Manhattan Review


Questions 99

112. In the diagram below, what is the length of the line segment AC?

Y
A B
P

O Q X

(1) The coordinates of P and Q are (0, 6) and (2, 0), respectively.
(2) The length of the line segment AB is 2.

Solve yourself:

113. Lines m and n are perpendicular to one another. Is the product of the slopes of the lines
less than the product of the Y-intercepts of the two lines?

(1) Only line m passes through the origin.


(2) None of the lines are parallel to either axis.

Solve yourself:


114. The coordinates of points A and B are p, q and (r , s). Is |q| > |s|?

(1) The points A and B are equidistant from the origin.



(2) p > |r |.

Solve yourself:

19992016 Manhattan Review www.manhattanreview.com


100 Questions

115. Is the area of quadrilateral ABCD equal to area of quadrilateral AECD?

(1) The coordinates of the points A, B, C and D are (0, 5), (6, 0), (0, 3) and (2, 0),
respectively.
(2) The point E lines on the line x = 6.

Solve yourself:

116. PQ and QP represent two-digit numbers having P and Q as their digits. RSR is a three-digit
number having the digits R and S. What is the value of P + Q + R + S?

(1) PQ + QP = RSR.
(2) P, Q, R and S are distinct non-zero digits.

Solve yourself:

117. PQP is a three-digit number having digits P and Q; and RQS5 is a four-digit number having
digits R, Q, S and 5. What is the value of R?

(1) PQP P = RQS5


(2) P, Q, R and S are distinct non-zero digits.

Solve yourself:

118. D represents a single digit number from 1 to 9. What is the value of D?

(1) D.D + DD = D.D DD


(2) D is not a prime number.

The dot (.) represents a decimal.

www.manhattanreview.com 19992016 Manhattan Review


Questions 101

Solve yourself:

119. ABA represents a three-digit number having digits A and B; BC3 represents another three-
digit having digits B, C and 3. What is the value of B?

(1) ABA 3 = BC3


(2) A, B, C and 3 are distinct non-zero digits.

Solve yourself:

120. AAAA is a four-digit number having the digit A and BCCC6 is a five-digit number having
digits B, C and 6. What is the value of (C B)?

(1) AAAA A = BCCC6.


(2) A < 6.

Solve yourself:

121. A teacher distributed few pens and few pencils evenly among the students in his class,
with each student getting an equal number of pens and an equal number of pencils. How
many students are there in the class?

(1) The teacher distributed 180 pens and 40 pencils.


(2) The total number of items received by each student is less than 15.

Solve yourself:

19992016 Manhattan Review www.manhattanreview.com


102 Questions

122. The number 2135, when divided by a number n, leaves a remainder r . What is the value
of n?

(1) The number 2315, when divided by n, also leaves the same remainder r .
(2) The number 2585, when divided by n, also leaves the same remainder r .

Solve yourself:

123. What is the remainder when 2n is divided by 8?

(1) n, when divided by 6, leaves remainder 5.


(2) 3n, when divided by 6, leaves remainder 3.

Solve yourself:

124. The remainder, when a number n is divided by 6, is p. The remainder, when the same
number n is divided by 12, is q. Is p < q?

(1) n is a positive number having 8 as a factor.


(2) n is a positive number having 6 as a factor.

Solve yourself:

n
125. Is an integer?
12

n2
(1) is an integer.
144
n
(2) is an integer.
6

www.manhattanreview.com 19992016 Manhattan Review


Questions 103

Solve yourself:


126. If x and y are integers, what is the value of x + y ?
(1) x y = 64.
(2) y x = 36.
Solve yourself:

n
127. Can be an integer?
192
(1) n is a multiple of 24 but not 16.
(2) n is a multiple of 8 but not 48.
Solve yourself:

128. If a and b are single digits from 0 to 9, inclusive, what is the value of (a + b)?
(1) The number 25a7b is divisible by 36.
(2) The number a1b is divisible by 4.
Solve yourself:

129. What is the remainder obtained when n is divided by 7?


(1) (2n + 2) is divisible by 14.

19992016 Manhattan Review www.manhattanreview.com


104 Questions

n2 1 is divisible by 7.

(2)

Solve yourself:

130. p is the value of the positive integer such that (3p + 2) is NOT a prime number. What is
the remainder when (3p + 2) is divided by p?

(1) p is the least possible integer which satisfies (3p + 2) is NOT a prime number.
(2) 3p + 2 < 500.

Solve yourself:


131. If p and q are integers, can q 1 always be expressed as an integer multiple of p?

(1) p>q
(2) q>1

Solve yourself:

132. If a, b and c are any three single digit numbers from 1 to 9, both inclusive, is (a + b + c)
divisible by 9?

(1) The number 2ab3 is divisible by 9.


(2) The number 4bc1 is divisible by 9.

Solve yourself:

www.manhattanreview.com 19992016 Manhattan Review


Questions 105

133. If x is an even number, is y also an even number?

a
(1) = 2.1, where a is a positive integer.
x+y

(2) a x + y = 210, where a is a positive integer.

Solve yourself:

134. What is the remainder when p is divided by 10?

(1) p 11 + 11p , when divided by 10, leaves remainder 4.


(2) p 3 , when divided by 10, leaves remainder 3.

Solve yourself:


135. If p, q, r and s are positive integers, and q s < 130, what is the value of q + s ?

(1) p, when divided by q, leaves remainder 9.


(2) r , when divided by s, leaves remainder 10.

Solve yourself:

136. Is x > y?

(1) x represents the greatest possible value of the exponents of 3, such that 3x can
divide n! and y represents the greatest possible value of the exponent among all
possible prime numbers p, such that p y can divide n!.
(2) n = 20.

19992016 Manhattan Review www.manhattanreview.com


106 Questions

Solve yourself:

If x and y are positive integers, is the number x 2 + 3x + y 2 even?



137.

(1) 4x + 3y is even.

(2) x y is even.

Solve yourself:

138. If n is a positive integer, what is the value of n?

n2 + 6n + 16 is perfectly divisible by (n + 4).



(1)
(2) n>2

Solve yourself:

139. A teacher asks the students in his class to write down four numbers such that each
number, starting from the second number is five greater than the previous number. The
starting numbers can be decided by the students as long as those are neither prime
numbers nor 1 and also, are distinct. What is the largest of the starting numbers chosen
by the students?

(1) There were four students in the class.


(2) None of the students wrote a number greater than 25.

Solve yourself:

www.manhattanreview.com 19992016 Manhattan Review


Questions 107

140. In the diagram below, a portion of the number line is shown, with numbers a, b, c and d
marked. If none among a, b, c and d are 0, is ab < cd?

(1) ab > 0
(2) bd < 0
Solve yourself:

141. The toy manufacturer Leno Inc. has four toy-manufacturing plants A, B, C and D, which
receive raw materials in the form of plastic. The plants have different efficiencies, and
hence, different quantities of raw materials are wasted. The quantity of raw materials
used by each plant and the number of toy units manufactured by the plants in a week
are shown.

Plant Raw material (000 kg) Finished product (units)

A 400 100,000

B 360 120,000

C 250 50,000

D 200 30,000

Does Plant B have the lowest rate of wastage of raw materials?


(1) All the plants manufacture the same and the identical toys.
(2) The weight of the toys manufactured in the plants A, B, C and D are in the ratio 2 :
2 : 3 : 4.
Solve yourself:

142. What is the ratio of a two-digit number and its sum of digits?
(1) The ratio is the minimum possible.

19992016 Manhattan Review www.manhattanreview.com


108 Questions

(2) The number has 9 as its unit digit.

Solve yourself:

143. If f (x) = x 3 + kx 2 4x, what is the value of k?

(1) f (k) = f (k)


(2) f (f (2)) = 0

Solve yourself:

144. In how many years would Bane be exactly thrice as old as his grandson?

(1) In ten years, Bane would be 30 years more than twice his grandsons age.
(2) Ten years back, Bane was 40 years more than thrice his grandsons age.

Solve yourself:

145. If pqr s is a four-digit number, where p, q, r and s are the digits, is the sum of digits of
the number a multiple of 12?

(1) p + q = 2 (r + s)

(2) p+r =4 q+s

Solve yourself:

www.manhattanreview.com 19992016 Manhattan Review


Questions 109

146. Four friends, P, Q, R and S have $1400 with them. Does P have the highest amount among
the four friends?

(1) The amount with R is one-fifth the amount with Q.


(2) R has $210 greater than what S has.

Solve yourself:

147. If n can be expressed as p 2 q3 , where p and q are distinct positive integers greater than
1, what is value of the LCM of p and q?

(1) n is divisible by 9 and 8.


(2) 400 < n < 500

Solve yourself:

148. If a, b and c are distinct positive integers, what is the value of (a + b + c)?

(1) 22a+c + 3b = 91
c
!

(2) 2a+2b + 3 4 = 131

Solve yourself:

149. If a and b are positive integers such that a > b, what is the remainder when (a b) is
divided by 3?

(1) (2a b) and (a + b), when divided by 9, leave remainder 6 in each case.
(2) (a + b) is divisible by 3.

19992016 Manhattan Review www.manhattanreview.com


110 Questions

Solve yourself:

150. A few taps, numbered 1, 2, 3 . . . , are connected to a tank, each having a distinct rate of
filling the tank. What is time taken to fill the tank by the 4th tap alone?

(1) The rate of filling of each tap, starting from the second tap, is double the efficiency
of the previous tap.
(2) The time taken by the first five taps, working together, to fill the tank is 1 hour.

Solve yourself:

www.manhattanreview.com 19992016 Manhattan Review


Chapter 4

Answer-key

111
112 Answer Key

(1) A (22) A (43) D (64) D (85) A

(2) C (23) C (44) C (65) C (86) B

(3) C (24) C (45) C (66) A (87) C

(4) E (25) C (46) C (67) C (88) A

(5) C (26) D (47) A (68) E (89) C

(6) C (27) A (48) E (69) D (90) C

(7) C (28) B (49) C (70) A (91) C

(8) C (29) C (50) C (71) C (92) C

(9) C (30) B (51) E (72) C (93) C

(10) D (31) C (52) A (73) C (94) A

(11) C (32) C (53) D (74) D (95) A

(12) E (33) A (54) B (75) C (96) C

(13) A (34) E (55) C (76) C (97) D

(14) C (35) C (56) D (77) C (98) D

(15) E (36) C (57) E (78) A (99) A

(16) C (37) D (58) B (79) C (100) C

(17) B (38) C (59) C (80) C (101) C

(18) E (39) C (60) B (81) C (102) A

(19) C (40) C (61) C (82) C (103) C

(20) C (41) C (62) C (83) A (104) B

(21) A (42) B (63) C (84) C (105) C

www.manhattanreview.com 19992016 Manhattan Review


Answer Key 113

(106) C (115) C (124) C (133) A (142) A

(107) E (116) A (125) C (134) D (143) A

(108) C (117) A (126) C (135) E (144) B

(109) C (118) A (127) D (136) A (145) B

(110) E (119) C (128) E (137) A (146) C

(111) C (120) A (129) A (138) A (147) B

(112) C (121) C (130) D (139) E (148) A

(113) C (122) E (131) C (140) E (149) A

(114) C (123) E (132) E (141) D (150) C

19992016 Manhattan Review www.manhattanreview.com


114 Answer Key

www.manhattanreview.com 19992016 Manhattan Review


Chapter 5

Hints

115
116 Hints

1. To get the value of the changed proportion, we must know the value of old proportion.
Specific value of total quantity of the mixture is not needed.

2. Since it is a percent based problem, choosing a suitable value of the quantity and price
will help in simplifying the calculations.

3. Try to express the different variables in terms of a single variable.

4. First one needs to decide whether the sales fall short of $800, lie between $800 and
$1000, or exceed $1000.

5. Try to find the number of students who read only gossips or cartoons as a percent of the
total students in the class.

6. Both change in quantity and change in price will affect the revenue earned. Though price
per bottle is decreased, due to decreases in the quantity of soft drink, the revenue earned
now may not necessarily decrease.

7. Though regular price of store B is not mentioned in the question, we can infer that
since store B normally does not offer any discount on the manufacturers suggested
retail price, its regular price = manufacturers suggested retail price.

8. Profit percent is calculated on the cost price. New sales price and new profit percent will
give you a relationship between cost price and initial sales price.

9. Try to have an estimate of the initial and final profits and compare them.

10. Profit percent is always represented on the cost price, though it can be calculated on
selling price too.

11. The average age of the members would increase by one year with every passing year.

12. Work with different variables and establish a relation among them. You also need to use
hit-and-trial to check whether the number of fruits each girl gets is unique integer.

13. Make use of the condition: a, b, c and d are distinct numbers in decision-making. Place
the numbers on a number line after deciding their order.

14. Find the total marks obtained by the three students with the lowest marks.

15. Work with a few even number of consecutive integers and with a few odd number of
consecutive integers; establish a pattern.

www.manhattanreview.com 19992016 Manhattan Review


Hints 117

16. Logically decide which combination of samples can be used. Rule out improbable brand.
Rule out improbable combinations. Decide which brand is must.

17. To get the value of the final concentration, we must know the value of initial concentra-
tion and the proportion of mixture replaced. The value of total quantity of the mixture
is not needed.

18. When the average value of a mixture equals that of one of its constituent, the quantity of
that constituent cannot be determined.

19. Compare the cases when equal quantities of wine in the containers are mixed and when
unequal quantities of wine in the containers are mixed.

20. To get the value of the changed proportion, we must know the value of the old propor-
tion.

21. Using the equality given in the first statement, the ratio of the four numbers can be
determined.

22. Using the equality given in the first statement, the value of k can be determined.

23. The problem does not necessarily ask to solve for the variables. It simply asks whether
a particular value is possible. This is a Can be type of questions and not a Must be
type.

24. Assuming the number of voters as 100, the number of votes possible for the three can-
didates needs to be determined.

25. Using the ratio in the hypothetical case, we need to determine the actual value of the
contribution made by the friend who contributed the highest amount.

26. If the total work remains a constant, the time taken to complete the work is inversely
proportional to the number of workers present.

27. If the total work remains a constant, the time taken to complete the work is inversely
proportional to the number of workers present.

28. To simplify calculations, assume the total work to be the LCM of the number of days
each takes to complete the work.

29. Work completed by a person is directly proportional to the time for which the person
works.

19992016 Manhattan Review www.manhattanreview.com


118 Hints

30. The time taken to complete a work is inversely proportional to the number of people
working.

31. You need to know the value of exact time and speed to calculate distance.

32. Clearly a question with answer either C or E. Decide the distance travelled by X and Y in
the second case and use the concept that if time remains constant; distance is directly
proportional to speed.

33. Use the concept of relative speed between two bodies moving in the same direction.

34. Find a few palindrome numbers and calculate the speed in each case.

35. Use the relations between speed and time and equate the total distance.

36. The interest earned is proportional to both the time of investment and the rate of inter-
est.

37. The interest earned is proportional to both the time of investment and the rate of inter-
est. Thus, if the duration or rate of interest is increased, the interest (NOT the amount)
would increase in the same proportion.

38. The final value of the loan is the interest added to the amount taken as loan.

39. The interest earned is proportional to both the time of investment and the rate of inter-
est. Equate the total interest to the given value.

40. The interest earned is proportional to the sum of money invested, the time for which it
is invested and the rate of interest. The number of months after which the second sum
was invested is an integer, so the values of the variables need to be decided accordingly.

41. Express the simple and compound interests in terms of the sum of money invested and
solve the equation.

42. The difference between compound and simple interest after 2 years is equal to simple
interest for a year on the simple interest calculated for a year on the principal.

43. Under compound interest, the sum of money invested gets multiplied by the same factor
every successive year.

44. Determine the height of grass at the end of every day, in milimeter and solve the equa-
tion.

www.manhattanreview.com 19992016 Manhattan Review


Hints 119

45. Determine the expression for the interest accumulated in the second year and solve the
equation.

46. Draw the Venn-diagram and make equations according to the information in the state-
ments.

47. Draw the Venn-diagram and make equations according to the information in the state-
ments.

48. Draw the Venn-diagram and make equations according to the information in the state-
ments.

49. Draw the Venn-diagram and express the number of students in each region in terms of a
single variable.

50. Draw two Venn-diagrams, one for men and the other for women. Using both the dia-
grams, relate the information given in the statements and make equations.

51. A linear function is one where the highest degree of the variable x is 1. Try to define
the function first.

52. Using the relation for the function, try to deduce a pattern in the values of the function.

53. A function can also be a constant function, i.e. it returns the same value for any value of
the variable x. Try to define the function first.

54. A linear function is one where the highest degree of the variable x is 1. Try to see if the
answer can be obtained without determining the unknown constants of the function.

55. Using the given composite function, try to determine the function g (x).


56. A two-digit number is expressed in terms of its digits x and y as 10x + y .

57. Form the equations and try to solve for the variables. Every pair of two equations need
not be solvable!

58. A pair of linear equations has infinite solutions when they represent the same straight
line. Determine the values of a and b using the above information.

59. The question is a Yes or No type of question. The actual value need not be calculated.
Read the question carefully and infer what it truly asks!

19992016 Manhattan Review www.manhattanreview.com


120 Hints

60. Two equations can be formed using the statements. However, note that the question is a
Yes or No type of question. The actual value need not be calculated.

61. Note that the problem talks about distinct odd integers, not necessarily consecutive. The
possible integers, which result in the mean being 55, need to be determined.

62. Determine the possible forms of the numbers so that the standard deviation is mini-
mized. Standard deviation is minimized when the numbers are relatively close to the
mean.

63. Find the possibilities of the numbers in each group using information from the state-
ments.

64. Find the possibilities of the numbers in each group using information from the state-
ments.

65. Find the possibilities of the 6 numbers knowing that the smallest is 13, i.e. the other
integers are greater than 13 (distinct numbers). Knowing the total of the 6 numbers may
help in determining the value of the largest number.

66. Remove the radicals by repeated squaring of the equation.

 
67. Express p + 1 q + 1 in terms of the coefficients of the quadratic equation.

68. Form a quadratic equation by replacing 2x with some other variable.

69. Simplify the equation in the first statement and equate the coefficients assuming the
roots to be p and p.

70. Substitute x = 1 and x = 2 and check if the required value is determinable.

71. The vowels and digits need to be first selected and then arranged. Since the number of
letters is odd, the answer would depend on whether the starting character is a vowel or
a digit.

72. The letters need to be first selected and then arranged. Different cases need to be con-
sidered depending on the number of repetitions of any letter.

73. The condition in the seating arrangement is on the women. So it is easier to first seat the
men and then seat the women, satisfying the condition.

74. The fact that the men and women can sit in alternate positions, gives an idea about the
number of men and women. Also, for a handshake to happen, exactly two people are

www.manhattanreview.com 19992016 Manhattan Review


Hints 121

required.

n
75. Among all values of Crn , the value of Crn increases as r approaches and decreases
2
thereafter as r approaches n.

76. The number of apples and oranges must be integers. Finding out the possible numbers
of apples and oranges purchased will help finding the required probability.

77. It is necessary to know whether the cards are picked with or without replacement, since
the required probability is different in those two cases.

78. Using the relation between probability of an even face and the probability of an odd face,
find the probabilities of each face appearing. Remember that the sum of probabilities is
1.

79. The remainders obtained by dividing different integers by 5 follow a pattern. In every
set of 5 consecutive integers, the pattern repeats. In order to uniquely determine the
number of integers leaving a prime remainder, the first or the last number should be
known.

80. The expected mark is, in a way, the average of the positive and negative marks obtained
in the test.

81. Search for a 30-60-90 triangle and use the property of the ratio of the sides.

82. Search for two similar triangles and use the ratio of the sides.

83. Search for an equilateral triangle and a 30-60-90 triangle. Note that individual values of
the angles are not necessary.

84. Find all angles in terms of BAC and then solve for BAC.

85. Search for a 45-45-90 triangle and use the property of the ratio of the sides.

86. Check if triangle XYZ is equilateral.

87. Check if AXPY is a square. Join the center of the circle to P and use the property of a
45-45-90 triangle.

88. Assume one other angle as a variable and see if the variable cancels out while determining
the measure of PTQ.

89. Join the center of the circle to one of the two vertices and use Pythagoras theorem.

19992016 Manhattan Review www.manhattanreview.com


122 Hints

90. Use similar triangles to determine the lengths of the sides of ASBQ.

91. Draw the diagram and use Pythagoras theorem to determine the height of the cone.

92. Open the right circular cylinder and observe that the thread forms a straight line, whose
length can be determined using Pythagoras theorem.

93. Equate the volumes of the cube and cylinder, and express the lateral surface areas of
both solids in terms of the radius and height of the cylinder.

94. Express the length of the longest line that can be drawn inside a cube and a cylinder
in terms of the dimensions. Use the relation between the dimensions of the cube and
cylinder to compare the lengths of the longest lines in the cube and cylinder.

95. Express the height and the radius of the cylinder in terms of the side of the cube using
the conditions given. Both the diameter and height of the cylinder have to be less than
the side of the cube for the cylinder to fit inside the cube.

96. Divide the figure in two parts whose areas can be easily determined. Search for a 30-60-
90 triangle.

97. Trace out the shortest possible routes and express them in terms of the side of the
hexagon.

98. Form a 45-45-90 triangle, partly inside and partly outside the octagon.

99. The ratio of the area of two triangles having the same height is equal to the ratio of their
base.

100. Draw the diagram for the pathway. Since the stone slabs cannot be broken, the number
of stone slabs need not necessarily be the area of the pathway divided by the area of
each stone slab.

101. Identify the angles which are equal and use the given relations between the angles.

102. Identify the alternate and corresponding angles which are equal and use the given rela-
tions between the angles.

103. Identify the angles which are equal and use the given relations between the angles.

104. Construct a couple of lines parallel to the given lines and find the alternate angles that
are equal.

www.manhattanreview.com 19992016 Manhattan Review


Hints 123

105. Identify the alternate angles which are equal.

106. Use the idea that the absolute value of a number n, i.e. |n| is always non-negative.

107. Use the idea that |n| = n if n 0 and |n| = n if n < 0. Also use the fact that 729 = 36 .

108. Determine the value of z using different values of x and y satisfying the given condi-
tions.


109. Determine the possible conditions for which |x| > y . Note that the absolute value of
a number is always non-negative.

110. Determine the possible values of y and hence determine the values of x.

111. Parallel lines have the same slope and perpendicular lines have product of slopes 1.

112. The slope of PQ and AC is the same.

113. A line passing through the origin has 0 Y- intercept. The slope of Y-axis is undefined.

114. Use the relation for the distance between two points.

115. Make a rough sketch of the points A, B, C, D and possible positions of the point E.

116. Determine the maximum value of the hundreds digit obtained by adding 2 two-digit
numbers.

117. Determine the number, which when multiplied with it results in 5.

118. Simplify the expression and solve for D.

119. Determine the possible values of the carry obtained when B is multiplied with 3. Solve
for all possible situations and find the one which satisfies all given conditions.

120. Determine the digit A, which when multiplied with it results in the unit digit 6.

121. The GCD (Greatest Common Divisor) of the number of pens and the number of pencils
would give us the maximum number of students in the class.

122. If n is a factor of two numbers p and q, then n is a factor of the GCD of the numbers p
and q.

19992016 Manhattan Review www.manhattanreview.com


124 Hints

123. If a number n, when divided by d, leaves quotient q and remainder r , we have: n =


dq + r .

124. If a number n, when divided by d, leaves quotient q and remainder r , we have: n = dq+r .
Try few possible values for n for each statement and see if p < q.

n
125. If is an integer, where k is also an integer, then k is a factor of n, provided n is also
k
another integer.

126. Determine the different ways in which 64 and 36 can be expressed using exponents.

127. Determine the highest exponent of 2 and 3 possible in n.

128. Determine possible values of a and b using the divisibility rules of 4 and 9.

129. If a number n, when divided by d, leaves quotient q and remainder r , we have: n = dq+r .
Also, use the relation: n2 1 = (n 1) (n + 1).

130. Try for different values of p to determine the value such that (3p + 2) is not a prime
number.

131. A number can be a negative integer multiple of another number as well.

132. Use the divisibility rule for 9.

133. A fraction becomes a terminating decimal only if the denominator of the fraction can be
expressed only by the product of exponents of 2 and/or 5.

134. The remainder, when a number n is divided by 10, is simply the unit digit of n.

135. If a number n, when divided by d, leaves remainder r , we have: 0 r < d.

136. The greatest exponent of any prime would happen for the smallest possible prime.

137. Think of what happens when even and odd numbers are added, subtracted, or multi-
plied.

138. Break the quadratic expression in n so as to factor out (n + 4) and observe the remainder
obtained.

139. Think of 4 numbers, such that there is a constant difference of 5 between consecutive
numbers. Think of all possible scenarios.

www.manhattanreview.com 19992016 Manhattan Review


Hints 125

140. Think of the possible positions of 0 on the number line.

141. The wastage rate will be lowest if a plant uses the least quantity of plastic for manufac-
turing the same toy, or, if different toys are manufactured, the quantity of plastic used
as a percent of quantity of plastic received is the least.

142. Express a two-digit number in terms of its digits.

143. Substitute x = k, x = k and x = 2 to determine the values of f (k), f (k) and f (2),
respectively.

144. Frame the equations based on the information given and solve them.

145. Think of the factors of 12. Also, if a number is a multiple of m and n, it is also a multiple
of the LCM of m and n.

146. Frame the equations and find the maximum possible value of the amount with P.

147. Determine the numbers of the form p 2 q3 satisfying the given conditions.

148. Express the numbers 91 and 131 as sum of exponents of 2 and 3. Compare the exponents
of 2 and 3 to determine the values of a, b and c.

149. If a number n, when divided by d, leaves quotient q and remainder r , we have: n =


dq + r .

150. The time taken by a tap to fill the tank is inversely proportional to the rate of filling of
the tap. Assuming a suitable value of the rate of filling of the first tap, determine the
capacity of the tank.

19992016 Manhattan Review www.manhattanreview.com


126 Hints

www.manhattanreview.com 19992016 Manhattan Review


Chapter 6

Solutions

127
128 Solutions

1. From statement 1:

Let the total quantity of the mixture be 10 lb.

Original quantity of beans = 40% of 10 lb = 4 lb.

Original quantity of pulses = 60% of 10 lb = 6 lb.

Let x lb of the mixture is removed. This removed part will contain beans and pulses in

the same proportion, i.e. 40% and 60%, respectively.


2
Quantity of beans removed = 40% of x = x lb.
5
3
Quantity of pulses removed = 60% of x = x lb.
5
2
 
Thus, quantity of beans left = 4 x lb.
5
Since x lb of pulses are now added to the present mixture, the quantity of beans would

remain the same.

Final quantity of the mixture = 10 lb.

Final quantity of beans = 20% of 10 lb = 2 lb.

Thus, we have:
2
4 x = 2 => x = 5
5
5
Thus, percent of the mixture removed = 100 = 50%. Sufficient
10
From statement 2:

Since the question asks for a percent value, the actual volume of the mixture is not

necessary.

Also, the ratio of beans and pulses in the original mixture is not known. Insufficient

The correct answer is option A.

Note: When a part of the mixture is removed, both the constituents will be removed in

the same proportion as originally present in the mixture.

2. From statement 1:

There is no information about the cost of the nut-mix or the individual constituents.

Insufficient

www.manhattanreview.com 19992016 Manhattan Review


Solutions 129

From statement 2:

There is no information about the ratio of peanuts and cashews in the nut-mix. Insuf-

ficient

Thus, from both statements together:

Let the cost of 10 lb of pure peanuts be $10 (since the question asks for a percent value,

the actual volume or the actual cost of the mixture can be assumed to be any suitable

value).

Thus, the cost of 10 lb of the nut-mix = (100 + 20)% of $10 = $12.

Quantity of peanuts in the mix = 10% of 10 lb = 1 lb.

Cost of 1 lb of peanuts = $1 . . . (i)

Quantity of cashews present in the mix = 10 lb 1 lb = 9 lb.

Cost of 9 lb of cashews = $(12 1) = $11.


11
Thus, cost of 1 lb of cashews = $ = $1.22 . . . (ii)
9
Thus, from (i) and (ii):
1.22 1
Required percent = 100 = 22%. Sufficient
1
The correct answer is option C.

( ! )
r +s
3. We need to determine the value of 100 .
p
From statement 1:
20 
r = p + s => p + s = 5r . . . (i)
100
However, the required value cannot be determined. Insufficient

From statement 2:
10
s= r => r = 10s . . . (ii)
100
However, the required value cannot be determined. Insufficient

Thus, from both statements together:

Substituting the value of r from (ii) in (i):

19992016 Manhattan Review www.manhattanreview.com


130 Solutions

p + s = 50s => p = 49s . . . (iii)

Thus, from (ii) and (iii):


!
r +s 10s + s 1100
 
100 = 100 = %. Sufficient
p 49s 49
The correct answer is option C.

4. Let the sales in March be $x.

As per the earlier pay structure:

Case 1: x 1000: Amount received by the salesman = $500.


5
 
Case 2: x > 1000: Amount received by the salesman = $ 500 + (x 1000) .
100

As per the new pay structure:

Case 1: x 800: Amount received by the salesman = $600.


4
 
Case 2: x > 800: Amount received by the salesman = $ 600 + (x 800) .
100

From statement 1:

We only know the new pay structure of the salesman.

There is no information about the amount he actually received. Insufficient

From statement 2:

We have no information about the new pay structure of the salesman. Insufficient

Thus, from both statements together:

(1) Case 1: x 800:

Amount the salesman would have received (earlier pay structure) = $500.

Amount he should have received (new pay structure) = $600.

However, the difference is not $104.

Hence, this case is not possible.

(2) Case 2: 800 < x 1000:

Amount the salesman would have received (earlier pay structure) = $500.
4
 
Amount he should have received (new pay structure) = $ 600 + (x 800) .
100

www.manhattanreview.com 19992016 Manhattan Review


Solutions 131

Since the difference is $104, we have:


4
600 + (x 800) 500 = 104
100
x 800
=> = 4 => x = $900
25
Thus, this case is possible.

(3) Case 3: x > 1000:

Amount the salesman would have received (earlier pay structure)


5
 
= $ 500 + (x 1000) .
100
4
 
Amount he should have received (new pay structure) = $ 600 + (x 800) .
100
Since the difference is $104, we have:
4 5
   
600 + (x 800) 500 + (x 1000) = 104
100 100
x 800 x 1000
=> = 4 => (4x 3200) (5x 5000) = 400
25 20
=> x = $1400

Thus, this case is also possible.

Thus, the sales in March cannot be uniquely determined. Insufficient

The correct answer is option E.

5. Since the question asks for a percent value, we can assume any suitable value of the

number of students.

Let the number of students be 1000.

Number of boys = 60% of 100 = 600.

Number of girls = 100 60 = 400.

Number of boys who read The Telegraph (TT) = 30% of 600 = 180.

Total number of students who read TT = 40% of 1000 = 400.

Thus, the number of girls who read TT = 400 180 = 220.

From statement 1:

Number of boys who read cartoons = 40% of 180 = 72.

19992016 Manhattan Review www.manhattanreview.com


132 Solutions

However, we have no information about the students who read the gossip pages.

Insufficient

From statement 2:

Number of girls who read gossip pages = 60% of 220 = 132.

However, we have no information about the students who read cartoons. Insufficient

Thus, from both statements together:

Number of students who read only gossip pages or only cartoons = 72 + 132 = 204.

We can be assured that none of the boys read gossip pages as the statement 1 says that

rest [60%] read only the sports pages., eliminating any possibility of any boy reading

gossip pages.

Similarly, we can be assured that none of the girls read cartoon as the statement 2 says

that rest [40%] read only the fashion pages., eliminating any possibility of any girl

reading cartoons.
204
Thus, the required percent = 100 = 20.4%. Sufficient
1000
The correct answer is option C.

6. From statement 1:

There is no information about the change in quantity. Insufficient

From statement 2:

There is no information about the change in price. Insufficient

Thus, from both statements together:

Initially, the price of 500 ml was $5, i.e. the price of 100 ml was $1.00
96
 
Finally, the price of 400 ml was (100 4)% of $5 = $ 5 = $4.80
100
4.80
Thus, finally, the price of 100 ml was $ = $1.20
4
Thus, the percent change (increase) in revenue earned for every 100 ml
1.20 1
 
= 100%
1

www.manhattanreview.com 19992016 Manhattan Review


Solutions 133

= 20%. Sufficient

The correct answer is option C.

7. Let the manufacturers suggested retail price be $x.


4
 
Regular price at store A = 80% of $x = $ x .
5
Regular price at store B = $x.

From statement 1:
4
 
Selling price at store A after special discount of 20% = (100 20)% of $ x
5
4 4 16
    
=$ x =$ x .
5 5 25
Since the final selling price is $64, we have:
16
x = 64 => x = 100 . . . (i)
25
However, the discount offered by store B is not known. Insufficient

From statement 2:

The discount offered by store A is not known.

Also, the price of the toy in store B is not known since x is not known. Insufficient

Thus, from both statements together:

We know that: x = 100.

Thus, price of the toy in store B before the special discount = $100.

Price at store B after special discount of 40% = (100 40)% of $100 = $60.

Thus, difference in price of the toy at the two stores

= $64 $60 = $4. Sufficient

The correct answer is option C.

8. From statement 1:

The cost price of the article is not known. Insufficient

From statement 2:

19992016 Manhattan Review www.manhattanreview.com


134 Solutions

Let the cost price be $x.

Let the initial selling price be $y.



New selling price = $ y + 150 .

New profit made = 12%.

Thus, we have:

y + 150 = (100 + 12) % of x


112
=> y + 150 = x
100
Since there are two unknowns, we cannot determine the value of y. Insufficient

Thus, from both statements together:

Let the cost price be $x.


110
 
Initial selling price, when the profit was 10% = (100 + 10)% of $x = $ x .
100
110
 
New selling price = $ x + 150 .
100
Since the profit is 12%, we have:
110
x + 150 = (100 + 12) % of x
100
110 112 2x
=> x + 150 = x => = 150
100 100 100
=> x = 7500

Initial selling price = (100 + 10) % of $7500

= $8250. Sufficient

Alternately, one can simply say that when the percent profit changes from 10% to 12%,

i.e. by 2% of the cost price, the price difference is $150.

Thus: 2% of cost price = $150


100
 
=> Cost price = $ 150 = $7500.
2
Since the article was actually sold at 10% profit, its actual selling price

= (100 + 10) % of $7500

= $8250. Sufficient

The correct answer is option C.

www.manhattanreview.com 19992016 Manhattan Review


Solutions 135

9. From statement 1:

Profit or loss scenario after the manager was changed is not known. Insufficient

From statement 2:

Profit or loss scenario before the manager was changed is not known. Insufficient

Thus, from both statements together:

Let the cost price of each article be $100.

Initial profit (old manager) = 10% of $100 = $10.

New profit (new manager) = 80% of $100 = $80.


80 10
Thus, percent increase in profit = 100 = 700%. The answer to the question is
10
Yes. Sufficient

The correct answer is option C.

10. From statement 1:

Let the selling price of the article be $100.

Profit = 20% of $100 = $20.

Thus, cost price = $(100 20) = $80.

Thus, actual percent profit


Profit 20
= 100 = 100
Cost Price 80
= 25%. Sufficient

From statement 2:

Let the cost price of the article be $100.

After increasing the price by 50%, the price obtained = (100 + 50) % of $100 = $150.

After reducing this price by 16.67%, the final selling price


1
 
= (100 16.67) % of $150 = $ 1 150
6
5
 
=$ 150 = $125.
6
Thus, actual percent profit

19992016 Manhattan Review www.manhattanreview.com


136 Solutions

Profit 25
= 100 = 100
Cost Price 100
= 25%. Sufficient

The correct answer is option D.

11. From statement 1:

There is no information about the ages of children. Insufficient

From statement 2:

There is no information about the average age of the family. Insufficient

Thus, from both statements together:

Since the average age in 2003 was 25 years, the average age in 2013, i.e. 10 years later,

for the four members would be 25 + 10 = 35 years.

Thus, the total age of the four adult members in 2013 = 4 35 = 140 years.

However, in 2013, there are six members (including two children) having an average age

of 25 years.

Thus, the total age of the six members = 6 25 = 150 years.

Thus, total age of the two children in 2013 = 150 140 = 10 years.
10
Thus, the average age of the two children = = 5 years. Sufficient
2
The correct answer is option C.

12. Let the number of fruits received by each boy and by each girl be x and y, respectively.

Let the number of boys and number of girls be m and n, respectively.

Thus, total fruits received by the boys and by the girls are mx and ny, respectively.

We need to determine the value of y.

From statement 1:

y = x + 2 . . . (i)

However, we cannot determine the value of y. Insufficient

www.manhattanreview.com 19992016 Manhattan Review


Solutions 137

From statement 2:

mx + ny =74 . . . (ii)

m = n + 6 . . . (iii)

However, we cannot determine the value of y. Insufficient

Thus, from both statements together:

Using (i) and (iii) and substituting in (ii), we have:

(n + 6) x + n (x + 2) = 74

=> nx + 3x + n = 37

=> x (n + 3) = 37 n
37 n
=> x =
n+3
Since x and n are integers, we have following possible value for x.

If n = 1 => x = 9 => y = x + 2 = 11

If n = 2 => x = 7 => y = x + 2 = 9

If n = 5 => x = 4 => y = x + 2 = 6

Thus, there is no unique value of y. Insufficient

The correct answer is option E.

13. From statement 1:

a + c = 2d
a+b+c+d 2d + b + d 3d + b
Thus, the average of a, b, c and d = = =
4 4 4
3d + b
If we assume that the average of the four numbers is b, then, =b
4
3d + b = 4b

=> d = b

However, it is mentioned in the question that, a, b, c and d are all distinct numbers.
3d + b
Thus, 6= b
4

19992016 Manhattan Review www.manhattanreview.com


138 Solutions

=> Average of a, b, c and d cannot be b.

Thus, the answer to the question is No. Sufficient

From statement 2:

b + d = 2c (i.e., c is the average of b and d)


a+b+c+d 2c + a + c 3c + a
Average of a, b, c and d = = =
4 4 4
Since we have no information about a, c or b, we cannot conclude whether the above

average can be equal to b.

For example:
OR

cant be the average of , , and can be the average of ,, and

Thus, the answer cannot be determined. Insufficient

The correct answer is option A.

14. Since the average marks of x students was m, the total marks = mx.

Let the average marks obtained by three students with the lowest marks be a.

Thus, total marks obtained by three students with the lowest marks = 3a.

From statement 1:

If the lowest three marks were excluded, the total marks of the remaining (x 3) stu-

dents

= mx 3a
mx 3a
 
Thus, the average marks of (x 3) students =
x3
Since this average is 3 greater than the earlier average, we have:
mx 3a
=m+3
x3
=> mx 3a = mx 3m + 3x 9

=> a = 3 + m x

If, according to the problem statement, a = (x + 3), we have:

www.manhattanreview.com 19992016 Manhattan Review


Solutions 139

x+3=3+mx

=> 2x = m . . . (i)

However, such a condition has not been given in the first statement. Insufficient

From statement 2:

There is no information about the marks of the three students having the lowest marks.

Insufficient

Thus, from both statements together:

We can see that the condition (i) as derived from the first statement exactly conforms the

information given in the second statement.

The answer to the question is Yes. Sufficient

The correct answer is option C.

15. From statement 1:

Let us take a few examples to check the values of |a b|.

Let P = {1, 2, 3, 4, 5}.


1+3+5
Thus, the average of the odd integers = a = = 3.
3
2+4
The average of the even integers = b = = 3.
2
Thus: |a b| = 0.

Let P = {1, 2, 3, 4, 5, 6}.


1+3+5
Thus, the average of the odd integers = a = = 3.
3
2+4+6
The average of the even integers = b = = 4.
3
Thus: |a b| = 1.

Thus, depending on the number of elements in the set P, the value of |a b| varies.

Insufficient

Note: If the number of terms is odd, then |a b| = 0, else 1.

From statement 2:

19992016 Manhattan Review www.manhattanreview.com


140 Solutions

The integers present in set P are not known. Insufficient

Thus, from both statements together:

Even after combining both statements, we have no information about the numbers in set

P. Insufficient

The correct answer is option E.

16. From statement 1:

There is no information about the prices. Insufficient

From statement 2:

There is no information about the concentrations. Insufficient

Thus, from both statements together:

We observe that sample R has exactly 60% concentration. Hence, if it is mixed with any

other sample (none of which have 60% concentration), the resulting mixture can never

have 60% concentration. Also, both Q and S have concentration below 60%, and hence,

they together cannot be used to prepare a mixture having a concentration of 60%. This

follows that P is must as one of the samples.

Thus, the only way of mixing samples would be:

Case 1: P and Q: Let x ml of P and y ml of Q be used.


66x + 48y x 2
Thus, we have: = 60 => 6x = 12y => =
x+y y 1
Thus, to minimize cost, we should take 2 ml of P and 1 ml of Q.

Thus, total cost = $(2 1 + 1 4) = $6.

Case 2: P and S: Let a ml of P and b ml of S be used.


66a + 54b a 1
Thus, we have: = 60 => 6a = 6b => =
a+b b 1
Thus, to minimize cost, we should take 1 ml of P and 1 ml of S.

Thus, total cost = $(1 1 + 1 4) = $5.

Thus, the least total cost of the mixture was obtained by mixing P and S, not by mixing P

and Q.

www.manhattanreview.com 19992016 Manhattan Review


Solutions 141

(Note: The least average cost per milliliter of the mixture was obtained by mixing P and

Q)

Thus, the answer to the question is No. Sufficient

Alternate Approach 1:

The problem does not need to be solved at all.

It can be easily seen that from the first statements, the ratio of quantity of the samples

can be determined.

Using the above ratios, the cost of the mixture can be determined from the second state-

ment.

Irrespective of the result obtained in the calculation, the answer can only be either Yes

or No.

Thus, the answer must be C.

17. From statement 1:

We have no information about the quantity of chemical kept aside from the bottle.

Insufficient

From statement 2:

Let the quantity of the mixture present in each bottle be 100 ml.

Since 60% of the contents, i.e. 60 ml was kept aside, 40% of the content, i.e. 40 ml

remained.

Concentration of the agent present in 40 ml = 50% of 40 ml = 20 ml.

Since an equal quantity of water was added, the quantity of the agent remained 20 ml in

100 ml of the mixture.


20
Thus, required concentration percent = 100 = 20%. Sufficient
100
The correct answer is option B.

18. Let the quantity of tea priced at $120 per pound, $135 per pound and $160 per pound

used be a pounds, b pounds and c pounds, respectively.

19992016 Manhattan Review www.manhattanreview.com


142 Solutions

Thus, total cost of the mixture = $(120a + 135b + 160c).

From statement 1:

Since the price of the mix containing (a + b + c) pounds was $135 per pound, the total

cost of the mix = ${135 (a + b + c)}.

Thus, we have:

135 (a + b + c) = 120a + 135b + 160c

=> 15a = 25c


a 5
=> =
c 3
However, the proportionate quantity of b used is not known. Insufficient

Note: Since the average price of the mix is the same as one of its constituents, the quan-

tity of that particular constituent could be any value since it wont change the average

price.

From statement 2:

We only know the quantity of one variety. Insufficient

Thus, from both statements together:

From the first statement, we know that: a : c = 5 : 3

Thus, the values of a and c could be respectively, 5 and 3 OR 10 and 6 OR 15 and 9, etc.

From the second statement, we know that: b = 3

Since the exact quantities of a and c are not known, the answer cannot be determined.

Some possible ratios of a : b : c could be 5 : 3 : 3 OR 10 : 3 : 6 OR 15 : 3 : 9, etc.

Thus, there is no unique answer. Insufficient

The correct answer is option E.

19. Apparently, this seems a very complicated problem having six unknown terms. However,

the reasoning is very simple. Let us see how.

From statement 1:

www.manhattanreview.com 19992016 Manhattan Review


Solutions 143

We have no information about the quantities of wine taken from the containers. Insuf-

ficient

From statement 2:

We know the order of the concentrations and quantities of wine used to form the mixture:

1st container 2nd container 3rd container

Concentration a b c a>b>c

Quantity p q r p>q>r

Thus, we find that the concentration of the wine whose maximum quantity was taken is

also the maximum. Similarly, the concentration of the wine whose minimum quantity

was taken is also the minimum.

Let us assume that we have taken equal quantities of the three varieties of wine.
a+b+c
 
In that case, the average concentration of the mixture would be simply . . . (i)
3
However, since we have actually taken higher quantity of the wine with the highest con-

centration AND least quantity of the wine with the least concentration, the overall con-

centration of the mixture would be pushed higher up than that obtained in (i).

However, we cannot say for certain whether that concentration would be greater than b.

For example:

Let a = 100, b = 90, c = 10, p = 10, q = 9, c = 8


100 10 + 90 9 + 10 8
Thus, average concentration of the mixture = = 70 < b
10 + 9 + 8
Let a = 100, b = 90, c = 80, p = 3, q = 2, c = 1
100 3 + 90 2 + 80 1
Thus, average concentration of the mixture = = 93.3 > b
3+2+1

Thus, there is no unique answer. Insufficient

Thus, from both statements together:

From the first statement, we have:

19992016 Manhattan Review www.manhattanreview.com


144 Solutions

ab =bc

=> a + c = 2b
a+c
=> b =
2
Thus, b is the average of a and c.

Let us again assume that we have taken equal quantities of the three varieties of wine i.e.

p = q = r.

In that case, the average concentration of the mixture would be simply


a+b+c 2b + b
 
= = (Since a + c = 2b)
3 3
= b . . . (ii)

Thus, the concentration of the mixture would be exactly the same as b.

However, since we have actually taken higher quantity of the wine with the highest con-

centration AND least quantity of the wine with the least concentration, the overall con-

centration of the mixture would be pushed higher up than that obtained in (ii), i.e. higher

than b. a p  c r .

Thus, the answer to the question is Yes. Sufficient

The correct answer is option C.

20. From statement 1:

There is no information regarding the final ratio of A and B in the mixture. Insufficient

From statement 2:

There is no information regarding the initial ratio of A and B in the mixture. Insufficient

Thus, from both statements together:

Initial ratio of A and B in the mixture was 4 : 5.

Thus, let the quantity of A and B in the mixture be 4x ml and 5x ml, respectively.

Thus, total volume initially = 9x ml.

We know that: A and B evaporate at 2 ml per minute and 3 ml per minute, respectively.

www.manhattanreview.com 19992016 Manhattan Review


Solutions 145

Thus, quantity of A and B evaporated in 10 minutes = 20 ml and 30 ml, respectively.

Thus, quantity of A and B remaining in the mixture = (4x 20) ml and (5x 30) ml,

respectively.

Thus, total volume finally = (9x 50) ml.

We know that the final ratio of A and B in the mixture was 5 : 4.

Thus, we have:
4x 20 5
=
5x 30 4
=> 16x 80 = 25x 150

=> 9x = 70

Thus, total volume finally = (9x 50) ml

= (70 50) ml = 20ml. Sufficient

The correct answer is option C.

21. From statement 1:


3 y
We know that: w = x= = 3z
4 2
4 w
=> x = w, y = 2w and z =
3 3
4
  
w w
wx
=> = 3  = 2
yz w
2w
3
Thus, the answer to the question is Yes. Sufficient

From statement 2:

We cannot determine the answer only knowing the sum of the four numbers. Insuffi-

cient

The correct answer is option A.

22. From statement 1:


a 2b 3a 4b
Let = = =r
4 5 k

19992016 Manhattan Review www.manhattanreview.com


146 Solutions

=> a = 4r , 2b = 5r and 3a 4b = kr

Substituting the values of a and b in the third equation, we have: 3 (4r ) 2 (5r ) = kr

=> k = 2. Sufficient

From statement 2:

We cannot determine the value of k from the single equation with two unknowns.

Insufficient

The correct answer is option A.

23. Let the number of pens, pencils and erasers purchased be a, b and c, respectively.

We need to determine whether the value of c can be 3.

From statement 1:
a b
We know that: =
b c
=> b2 = a c

However, the value of c cannot be determined. Insufficient

From statement 2:

a + b + c = 21

However, the value of b cannot be determined. Insufficient

Thus, from both statements together:


a b
Let = = r => b = cr and a = br = cr 2
b c
a + b + c = 21

=> cr 2 + cr + c = 21

=> c r 2 + r + 1 = 21


Here, c must be an integer. However, the value of r can be a fraction or an integer.

According to the problem, we need to determine whether the value of c can be 3.

If c = 3, r 2 + r + 1 = 7


We can easily observe that for r = 2, r 2 + r + 1 = 4 + 2 + 1 = 7 (satisfies).

www.manhattanreview.com 19992016 Manhattan Review


Solutions 147

Thus, the value of c can be 3.

The answer to the question is Yes. Sufficient

The correct answer is option C.

24. Let the number of voters be 100.

We know that 75 percent of the voters decided to cast their votes in the ratio 3 : 2 for A

and B, respectively.
3
 
Number of voters who had decided to vote for A = 75 = 45.
3+2
2
 
Number of voters who had decided to vote for B = 75 = 30.
3+2
Number of voters who had decided not to vote before C decided to contest = 100 75 =

25.

From statement 1:

We know that, among the voters who had initially decided not to vote, 40 percent voted

for candidate C.

Thus, number of votes for C from the undecided voters

= 40% of 25 = 10 . . . (i)

However, we have no information about 60% of 25 = 15 of the voters, and the number

of voters who had initially decided to vote for A, but finally changed their opinion.

Insufficient

From statement 2:

We know that, among the voters who had decided to vote for candidate A, 80 percent

voted for candidate C.

Thus, number of votes for C from the voters who had initially decided to vote for A

= 80% of 45 = 36 . . . (ii)

Thus, the number of votes for A = 45 36 = 9.

However, we have no information about the number of voters who had initially decided

not to vote (25), and how many out of those finally voted for C.

19992016 Manhattan Review www.manhattanreview.com


148 Solutions

Assuming all 25 voted for C, then the vote tally would be: A gets 45 36 = 9; B gets 30;

C gets 36 + 25 = 61 (Winner).

Moreover, assuming all 25 voted for A, then the vote tally would be: A gets 45 36 + 25

= 34; B gets 30; C gets 36 (Winner).

However, assuming all 25 voted for B, then the vote tally would be: A gets 45 36 = 9; B

gets 30 + 25 = 45 (Winner); C gets 36. Insufficient

Thus, from both statements together:

Thus, the final tally of votes:

Candidate A = 9

Candidate B = 30

Candidate C = 10 + 36 = 46 . . . Using (i) and (ii)

However, we still have no information about the voters who had not decided to vote

(From the first statement we know that 40% of the undecided voters (25) voted for C.

However, we have no information about the remaining 60% of the undecided voters (25),

i.e. 60% of 25 = 15 voters, who may have voted for any one of A, B or C).

Let us take cases:

The remaining 15 voters vote for A:

A gets 9 + 15 = 24 votes => C remains the winner

The remaining 15 voters vote for B:

B gets 30 + 15 = 45 votes => C remains the winner

Thus, irrespective of the result of the 15 voters, C remains the winner.

Thus, the answer to the question is Yes. Sufficient

The correct answer is option C.

25. From statement 1:

We cannot determine the amount contributed by the friend who contributed the highest

amount only knowing the total amount contributed. Insufficient

www.manhattanreview.com 19992016 Manhattan Review


Solutions 149

From statement 2:

We have no information about the actual amount contributed by the three friends to-

gether. Insufficient

Thus, from both statements together:

Total amount contributed = $510.

Had each contributed $10 extra, the total contribution would have been

= $510 + $(10 3) = $540.

Since, in this case, the ratio of the contributions is 2 : 3 : 4, the contribution made by the

friend who contributed the highest amount


4
 
= $540 = $240.
2+3+4
Thus, the actual contribution made by the friend who contributed the highest amount

= $(240 10) = $230. Sufficient

The correct answer is option C.

26. From statement 1:


1
We know that in 30 days, 50 people completed only 25% = of the work.
4
Thus, to complete the entire work, they would have taken a total time of 30 4 = 120

days. Sufficient

From statement 2:

Let us assume that each person employed can do one unit of work per day.

Thus, the initial 50 people in 30 days would have done 50 30 = 1500 units.

Thereafter, the 75 people (since 25 more workers joined) in the remaining 90 30 = 60

days, would have done 75 60 = 4500 units.

Thus, the total work of construction involved 1500 + 4500 = 6000 units.

Thus, if the extra people were not employed, time taken by the 50 workers
6000
= = 120 days to complete the entire work. Sufficient
50
The correct answer is option D.

19992016 Manhattan Review www.manhattanreview.com


150 Solutions

27. From statement 1:

After 13 days, number of soldiers doubled from 100 to 100 + 100 = 200.

Since thereafter, the rations last 12 days, had the additional soldiers not joined, the food

would have lasted for double the time, i.e. 12 2 = 24 days.

Since 13 days have already passed, the total number of days the rations would have

lasted in a camp of 100 soldiers

= 24 + 13 = 37 days. Sufficient

From statement 2:

If 20 soldiers had left the camp in the very beginning, the strength of the camp would
80 4
have been 80, i.e. = of the initial strength.
100 5
5 1
 
Thus, the rations would have lasted for of the initially expected time, i.e. 1 + =
4 4
25% more number of days.

Thus, the data provided in the statement is an obvious fact.

Hence, the statement is not sufficient to determine the answer. Insufficient

The correct answer is option A.

28. We know that A, B and C can complete a piece of work in 24 days, 40 days and 60 days,

respectively.

Let the total work = LCM of (24, 40 and 60) = 120 units.
120 120 120
Thus, A, B and C, per day complete = 5 units, = 3 units and = 2 units of
24 40 60
work, respectively.

Let the total number of days in which the work was completed be x.

From statement 1:

We have no information about the number of days B and C had worked. Insufficient

From statement 2:

Since B and C left the work a few days before it was completed, it is implied that A

worked till the end.

www.manhattanreview.com 19992016 Manhattan Review


Solutions 151

Thus, A, B and C had worked for x days, (x 2) days and (x 7) days, respectively.

Thus, total units of work completed by A, B and C

= 5x + 3 (x 2) + 2 (x 7) = (10x 20) units.

Thus, we have: 10x 20 = 120 => x = 14. Sufficient

The correct answer is option B.

29. From statement 1:

We know that: 11 hours after A and B had started working, 45 percent of the work was

still remaining, and thus, 55% of the work was completed.


11
Thus, time required to complete the remaining 45% of the work = 45 = 9 hours.
55
Thus, the total work was completed in 11 + 9 = 20 hours.

However, we cannot determine the time taken by A to complete the work alone. Insuf-

ficient

From statement 2:

We have no information about the time taken to complete the work. Insufficient

Thus, from both statements together:

Since A had done 60% of the work and the work lasted for 20 hours, we can say that A,

in 20 hours, can do 60% of the work.


20
Thus, time taken by A to complete the entire work = 100 = 33.33 hours. Sufficient
60
The correct answer is option C.

30. Let each man and each woman can complete m units and w units of work per day.

Let the total work be t units.

From statement 1:
1
We know that: 4 men and 10 women employed initially completed of the work in less
3
than 6 days.

Thus, they would have completed the entire work in less than 3 6 = 18 days.

19992016 Manhattan Review www.manhattanreview.com


152 Solutions

However, we cannot determine whether they can complete the work within 15 days.

Insufficient

From statement 2:

With 2 additional men and 2 additional women, we have:

1
6 men and 12 women would have completed of the work in 5 days.
2
Thus, 6 men and 12 women would have completed the entire work in 2 5 = 10 days.

Thus, 1 man and 2 women would have completed the entire work in 10 6 = 60 days

1
(The number of men and the number of women have become of their initial values).
6
60
Thus, 4 men and 8 women would have completed the entire work in = 15 days
4
(The number of men and the number of women have become 4 times of their values).

Since 4 men and 8 women can complete the work in 15 days, with 4 men and 10 women

(i.e. 2 more women), the work would be completed in less than 15 days. Sufficient

The correct answer is option B.

31. From statement 1:

There is no information about the time taken to travel. Insufficient

From statement 2:

There is no information what part of the total distance was travelled at 15 miles per

hour. Insufficient

Thus, from both statements together:

Let the time for which he travelled at 15 miles per hour be x hours.

Thus, distance travelled = 15x miles.

1
Since this is of the total distance, the total distance between two cities = 4 15x
4
The remaining distance = 4 15x 15x = 45x miles.

45x
 
Time taken to travel this distance at 20 miles per hour = hours.
20

www.manhattanreview.com 19992016 Manhattan Review


Solutions 153

We know that: The time for which he travelled at 20 miles per hour was 5 hours greater
45x
than the time for which he travelled at 15 miles per hour. => =5+x
20
5x
=> =5
4
=> x = 4 Thus, the total distance = 15x + 45x

= 60x = 240. Sufficient

The correct answer is option C.

32. From statement 1:

We only know the speeds of X and Y.

Using only the speeds, we cannot determine the distance. Insufficient

From statement 2:

We have no information about the speeds of X and Y. Insufficient

Thus, from both statements together:

In the first case, X travelled distance MP and Y travelled distance NP.

Since their time of travel is the same, the distances travelled is proportional to their
Distance travelled by X Speed of X
speeds => =
Distance travelled by Y Speed of Y
MP 30 3 3
=> = = => MP = NP . . . (i)
NP 20 2 2
In the second case, Y travelled at a higher speed that in the previous situation. Thus, the

distance travelled by Y should be greater than in the previous situation.

We know that they would meet at a point which is 10 miles away from P.

Thus, X travelled distance (MP 10) and Y travelled distance (NP + 10).

Since their time of travel is the same, the distances travelled is proportional to their

speeds
Distance travelled by X Speed of X
=> =
Distance travelled by Y Speed of Y
MP 10 30 30
=> = =
NP + 10 (100 + 20) % of 20 24
MP 10 5
=> =
NP + 10 4

19992016 Manhattan Review www.manhattanreview.com


154 Solutions

=> 4MP 40 = 5NP + 50


3
 
=> 4 NP 5NP = 90
2
=> NP = 90. Sufficient

The correct answer is option C.

33. From statement 1:

Let the speeds of X and Y be s miles per hour and 2s miles per hour, respectively.
45 3
We know that: X started 45 minutes i.e. = hours ahead of Y.
60 4
3
 
Thus, when Y started, X was already ahead by s miles.
4
At this point, both X and Y are moving the same direction with Y chasing X at a speed of

2s miles per hour, while X is moving at s miles per hour.

Thus, relative speed of Y with respect to X = 2s s = s miles per hour.

Thus, Y is gaining on X at a speed of s miles per hour.


3
 
Thus, time Y will take to overtake X, i.e. gain a distance of s miles
4
3
 
s
4 3
= = hours, i.e. 45 minutes.
s 4
Thus, X and Y would meet 45 minutes past 7:45 am, i.e. at 8:30 am. Sufficient

From Statement 2:

We have no information about the relation between the speeds of X and Y. Insufficient

The correct answer is option A.

34. From statement 1:

Te odometer reading is 18992. A possible palindrome before this was 18981. However,

since we need a number greater than 18992, the number must start with 19_ _ _.

Possible five-digit palindromes after 18992 are:

99
19091: Distance = 19091 18992 = 99 => Speed = = 19.8 miles per hour
5
199
19191: Distance = 19191 18992 = 199 => Speed = = 39.8 miles per hour
5

www.manhattanreview.com 19992016 Manhattan Review


Solutions 155

299
19291: Distance = 19291 18992 = 299 => Speed = = 59.8 miles per hour, and
5
so on.

Thus, the speed cannot be uniquely determined. Insufficient

From statement 2:
80
Since in 2 hours, he travelled less than 80 miles, his speed must be less than = 40
2
miles per hour.

However, the speed cannot be uniquely determined. Insufficient

Thus, from both statements together:

Since the speed is less than 40 miles per hour, it can be either 19.8 miles per hour or

39.8 miles per hour, as observed from the first statement.

Thus, the speed cannot be uniquely determined. Insufficient

The correct answer is option E.

35. We know that the buses start from A and B at 7:00 am and 8:30 am, respectively.

From statement 1:

Let the speed of the first bus be x miles per hour.


4
 
Thus, the speed of the second bus = x miles per hour.
7
The speed of the first bus cannot be determined since no other information is given.

Insufficient

From statement 2:

Since the buses meet at 7:00 pm, the bus from A travelled for 12 hours and the bus from
1 21
B travelled for 10 = hours.
2 2
However, there is no information about the relation between the speeds of the two buses.

Insufficient

Thus, from both statements together:

Since the total distance is 900 miles, we have:

19992016 Manhattan Review www.manhattanreview.com


156 Solutions

Distance travelled by the bus from A + Distance travelled by the bus from B = 900
21 4
 
=> 12x + x = 900
2 7
=> x = 50. Sufficient

The correct answer is option C.

36. Let the sum of money invested in each bank be $P .

From statement 1:

Let the rates of interest in the two banks be x% and y%, respectively.
P x2
 
Interest obtained from the first bank after 2 years = $ .
100
P y 3
 
Interest obtained from the second bank after 3 years = $ .
100
Since the ratio of the interests are 4 : 5, we have:
P x2 P y 3
: =4 :5
100 100
=> 2x : 3y = 4 : 5
x 6
=> =
y 5
Since the value of x is not known, the value of y cannot be determined. Insufficient

From statement 2:

We cannot determine the rate of interest in the second bank only knowing the rate of

interest in the first bank. Insufficient

Thus, from both statements together:

We know that x = 6
6 6
=> =
y 5
=> y = 5. Sufficient

The correct answer is option C.

37. From statement 1:

Amount invested = $1400.

www.manhattanreview.com 19992016 Manhattan Review


Solutions 157

Final value of the investment after 2 years = $1600.

Thus, interest accumulated in 2 years = $(1600 1400) = $200.


200
 
Thus, interest accumulated in 3 years = $ 3 = $300.
2
Thus, final value of the investment after 3 years = $(1400 + 300) = $1700. Sufficient

From statement 2:

Let the rate of interest be x%.

Let the interest obtained after 3 years be $N.

Thus, the final value of the investment after 3 years = $(1400 + N)

If the rate of interest was doubled to 2x%, the interest obtained after 3 years would also

be doubled, i.e. the interest would be $(2N).

Thus, the final value of the investment after 3 years = $(1400 + 2N)

Thus, we have:

(1400 + 2N) (1400 + N) = 300 => N = 300

Thus, the final value of the investment after 3 years with normal interest rate

= $(1400 + N) = $(1400 + 300) = $1700. Sufficient

The correct answer is option D.

38. The interest earned is proportional to both the time of investment and the rate of inter-

est. Equate the interests from each sum of money to the total value given.

From statement 1:

We only know that the number of years for which the loan was taken was the same as

the rate of interest. Insufficient

From statement 2:

Let the loan taken be $L, at x% rate of interest, for n years.


Lnx
 
Thus, the interest accumulated = $(n x% of L) = $ .
100
Lnx nx
    
Thus, the final value of the loan = $ L + = $ L 1+ .
100 100

19992016 Manhattan Review www.manhattanreview.com


158 Solutions

Since the final value of the loan after the duration of loan was 9 percent higher than the

amount taken as loan, we have:


nx 109
 
L 1+ = (100 + 9) % of L = L
100 100
nx 109
=> 1 + =
100 100
=> nx = 9

However, the value of x cannot be determined since the value of n is not known.

Insufficient

Thus, from both statements together:

We know that: n = x

=> x 2 = 9

Since x cannot be negative, we have:

x = 3. Sufficient

The correct answer is option C.

39. From statement 1:

There is no information about the duration for which the sums are invested. Insufficient

From statement 2:

There is no information about the sums of money invested. Insufficient

Thus, from both statements together:

The man invested $12000 for 3 years and $9000 for 5 years.

Let the interest obtained from the first sum be $N.

Thus, for $12000 invested for 3 years, the interest is $N.


N N
   
Thus, for $1000 invested for 1 year, the interest is $ =$ .
12 3 36
N 5N
   
Thus, for $9000 invested for 5 years, the interest is $ 95 = $ .
36 4
5N 9N
   
Thus, total interest obtained = $ N + =$ .
4 4
9N
Thus, we have: = 2250
4

www.manhattanreview.com 19992016 Manhattan Review


Solutions 159

=> N = 1000 Thus, for $12000 invested for 3 years, the interest is $1000.
1000 25
   
Thus, the rate of interest = 100 = %. Sufficient
3 12000 9
The correct answer is option C.

Alternate Approach:

Let the rate of interest be x%.

We know that the sum of interests on $12000 for 3 years and $9000 for 5 years is $2250
12000 3 x 9000 5 x
=> + = 2250
100 100
=> 810x = 2250
2250 25
=> x = = %.
810 9

40. Let the second sum of money be invested after m months (where m is an integer) from

the beginning of the year.

From statement 1:

Let the sums of money invested in the two banks be $(x) and $(6x), respectively.
 
Also, let the rates of interest applicable in the two banks be 3y % and 4y %, respec-

tively.

However, there is no information, using which, the value of m may be determined.

Insufficient

From statement 2:

We know that the total interest accumulated after one year was 4 percent of the total

investment made in the year.

However, there is no information, using which, the value of m may be determined.

Insufficient

Thus, from both statements together:

Interest accumulated on the first sum of money


x 3y 1
 
=$
100

19992016 Manhattan Review www.manhattanreview.com


160 Solutions

3xy
 
=$ .
100
Since the second sum of money was invested after m months from the beginning of the
12 m th
 
year, it remained invested only for (12 m) months, i.e. part of the year.
12
Thus, the interest accumulated on the second sum of money
12 m
 
6x 4y 12
= $
100

2xy (12 m)
 
=$ .
100
Thus, total interest
3xy 2xy (12 m)
 
=$ + .
100 100
Since the interest is 4% of the total investment, we have:
3xy 2xy (12 m) 4
+ = (x + 6x)
100 100 100
xy 28x
=> {3 + 2 (12 m)} =
100 100
=> y (27 2m) = 28
28
=> 27 2m =
y
( )
1 28
=> m = 27
2 y
28
Since m is an integer and also less than 12, must be an odd integer (only then,
( ) y
28
27 will be even and hence, m would be divisible by 2) and also should be greater
y ( )
44
than 3 (only then 27 will be less than 24 and hence m would be less than 12).
y
28
Since should be odd, y must be 4. Hence, we have:
y
1 28
 
m= 27 = 10. Sufficient
2 4
The correct answer is option C.

It is important to extract the juice out of the information given in the question. Had

we missed the analysis of the given information after an integer number of months,

we might have concluded that the answer is E. Every piece of information given in the

question or statements is of use, so do read judiciously.

www.manhattanreview.com 19992016 Manhattan Review


Solutions 161

41. From statement 1:

There is no information about the rate of interest under compound interest. Insufficient

From statement 2:

Let the rate of interest under simple interest be x% per annum.

Thus, the rate of interest under compound interest = (x + 3) % annum.

Let the sum of money invested in either case = $P .


2P x
 
Simple interest in 2 years = $ .
100
P (x + 3)
 
Compound interest in 1 year = $ .
100
(In 1 year, the compound interest is the same as simple interest)

Thus, we have:
2P x P (x + 3) P (x 3)
= 111 => = 111 . . . (i)
100 100 100
However, the value of x is not known. Insufficient

Thus, from both statements together:

We know that: x = 5.

Thus, from (i), we have:


P (5 3)
= 111
100
=> P = $5550. Sufficient

The correct answer is option C.

42. Let the sum of money invested in each case be $x.

Let the rate of interest in each case be r % per annum.

From statement 1:
2xr
 
Simple interest after 2 years = $ .
100
(  2 )
r
Compound interest after 2 years = $ x 1+ x
100
( !)
r2 2r
= $ x 1+ 2
+ 1
100 100

19992016 Manhattan Review www.manhattanreview.com


162 Solutions

r 2r
  
=$ x +
1002 100
xr r
  
=$ 2+ .
100 100
Thus, we have:
xr r 2xr
 
2+ = 110
100 100 100
r 2
 
=> x = 110
100
However, the value of x is not known. Insufficient

From statement 2:
2xr
 
Simple interest after 2 years = $ .
100
!
2xr 2
If the rate of interest is squared, simple interest after 2 years = $ .
100
Thus, we have:
2xr 2 2xr
=4
100 100
=> r = 4. Sufficient

The correct answer is option B.

43. Let the sum of money be $x.

Let the rate of interest be r % per annum.

Let the number of years after which the sum of money become nine times its present

value be t years.

Thus, we have:
r t
 
x 1+ = 9x
100
r t
 
=> 1 + = 9 . . . (i)
100
We need to determine the value of t.

From statement 1:

Since the sum of money became thrice its value in 6 years, we have:
r 6
 
x 1+ = 3x
100

www.manhattanreview.com 19992016 Manhattan Review


Solutions 163

r 6
 
=> 1 + =3
100
r
   
1
=> 1 + = 3 6 . . . (ii)
100
Substituting the above in (i), we have:
  1  t
3 6 = 9 = 32

t
=> = 2 => t = 12. Sufficient
6
The correct answer is option D.

Alternate Approach:

In compounding, the sum of money becomes multiplied by the same factor ever year.

We know that the sum of money was multiplied by a factor of 3 in 6 years.

Thus, in another 6 years, the sum of money would become multiplied again by a factor

of 3, i.e. the sum of money would become 3 3 = 9 times.

Thus, total time = 6 + 6 = 12 years.

From statement 2:

We know that the sum of money was multiplied by a factor of 27 in 9 years.

Working by the above logic, and back-tracking, we can say that in every 3 years, the sum

of money must have been tripled.

(Observe: If in 3 years, the sum becomes thrice, then in another 3 years, i.e. 6 years,

the sum becomes 3 3 = 9 times, and hence, in another 3 years, i.e. 9 years, the sum

becomes 9 3 = 27 times.)

How to spot the above pattern?

Observe that 27 is a cube, i.e. 33 . That means, that sum of money must have been

multiplied by 3 thrice (33 = 3 3 3). Since the total time was 9 years, divide 9 by 3 (this
9
3 represents how many times the sum of money was multiplied by a factor) to get = 3,
3
i.e. in every 3 years the sum of money was tripled.

The result obtained from the second statement is the same as obtained from the first

statement. Sufficient

19992016 Manhattan Review www.manhattanreview.com


164 Solutions

44. Let the initial height of the grass be x mm.

Cows reduce the height of the grass every day by 1 mm.

From statement 1:

We only know that grass grows by 2% of the height on the previous day.

Using this information alone, we cannot determine the value of x. Insufficient

From statement 2:

We know that the grass is completely grazed in 3 days.

However, we have no information about the percent by which grass grows every day.

Insufficient

Thus, from both statements together:

First day: Height of grass = x mm.

Height reduced by cows = 1 mm.

Final height = (x 1) mm.


102 (x 1)
Second day: Height of grass = 102% of (x 1) mm = mm.
100
Height reduced by cows = 1 mm.
102 (x 1)
 
Final height = 1 mm.
100
102 (x 1)
 
Third day: Height of grass = 102% of 1 mm =
100
102 102 (x 1)
 
1 mm.
100 100
Height reduced by cows = 1 mm.
102 102 (x 1)
   
Final height = 1 1 mm.
100 100

Since there is no grass left after the third day, we have:


102 102 (x 1)
 
1 1=0
100 100
We can see that this is a linear equation, and hence, can be easily solved for x. Sufficient

(The above calculation gives us x = 2.94, however, one may require a calculator to do it.

Note that in DS problems, the actual answer is not required!)

The correct answer is option C.

www.manhattanreview.com 19992016 Manhattan Review


Solutions 165

45. Let the sum of money invested be $x.

Let the rate of interest be r % per annum.

From statement 1:
xr
 
Interest in the first year = $ .
100
Thus, we have:
xr
= 120 . . . (i)
100
Since the value of x is not known, the value of r cannot be determined. Insufficient

From statement 2:

Interest accumulated in the 2nd year

= Interest accumulated in 2 years - Interest accumulated in the 1st year


r 2
( ! )
xr
 
= $ x 1+ x .
100 100
Thus, we have:
r 2
!
xr
 
x 1+ x = 144
100 100
r 2
( )
r

=> x 1+ 1 = 144
100 100
r 2
( )
r
 
=> x 1+ 1+ = 144
100 100
r r
  
=> x 1 + 1+ 1 = 144
100 100
xr r
 
=> 1+ = 144 . . . (ii)
100 100
Since the value of x is not known, the value of r cannot be determined. Insufficient

Thus, from both statements together:

Dividing (ii) by (i):


r 144 r 1
1+ = => =
100 120 100 5
=> r = 20. Sufficient

The correct answer is option C.

Alternate Approach:

19992016 Manhattan Review www.manhattanreview.com


166 Solutions

Since the value of x is not known, the value of r cannot be determined from either

statement alone.

Combining both statements, we see that:


144 120
Percent increase in the interest in the 2nd year over the 1st year = 100 = 20%.
120
This is because in compound interest, for any year, there is additional interest applicable

on the interest of the previous year.

Since the interest grew by 20%, the rate of interest must have been 20%.

46.
German French

Japanese

Total 100

We need to determine the value of d.

From statement 1:

We know that 51 students study German.

Also, 31 students study German but not French

=> a + e = 31

=> d + g = 51 31 = 20 . . . (i)

Since the value of g is not known, the value of d cannot be determined. Insufficient

From statement 2:

We know that:

g = 9 . . . (ii)

However, from this alone, the value of d cannot be determined. Insufficient

www.manhattanreview.com 19992016 Manhattan Review


Solutions 167

Thus, from both statements together:

From (i) and (ii):

d = 20 g = 20 9 = 11. Sufficient

The correct answer is option C.

47.
Baseball Basketball

=0 Football

Total 50

We need to determine the value of a.

From statement 1:

We know that 60 percent of the students do not play basketball or football.

Thus, we have:

a + n = 60% of 50

Since n = 0, we have:

a = 30. Sufficient

From statement 2:

We know that: g = 4% of 50 = 2.

However, this information cannot be used to determine the value of a. Insufficient

The correct answer is option A.

48.

19992016 Manhattan Review www.manhattanreview.com


168 Solutions

English Mathematics


Total 120

We need to determine the value of (a + b).

Since there are 120 students, we have:

a + b + c + n = 120 . . . (i); where n is the number of students who failed in both the

subjects.

From statement 1:

a + c = 70 . . . (ii)

=> b + n = 120 70 = 50 . . . (iii)

The value of (a + b) cannot be determined. Insufficient

From statement 2:

b + c = 80 . . . (iv)

=> a + n = 120 80 = 40 . . . (v)

The value of (a + b) cannot be determined. Insufficient

Thus, from both statements together:

Even after combining both statements, the value of (a + b) cannot be determined using

that above equations. Insufficient

The correct answer is option E.

49.

www.manhattanreview.com 19992016 Manhattan Review


Solutions 169

Economics Accountancy

=0
Total 100%

We need to determine the value of a.

From statement 1:

We know that:

Out of every seven students taking Accountancy, three take Economics as well
b+c 7 b 7
=> = => + 1 =
c 3 c 3
b 4
=> =
c 3
4
=> b = c . . . (i)
3
However, we cannot determine the value of a from the above equation alone. Insuffi-

cient

From statement 2:

We know that:

For every three students taking at least one of the two courses, there is one student who

takes up both
a+b+c 3 a+b
=> = => +1=3
c 1 c
a+b
=> =2
c
=> a + b = 2c . . . (ii)

However, we cannot determine the value of a from the above equation alone. Insuffi-

cient

Thus, from both statements together:

19992016 Manhattan Review www.manhattanreview.com


170 Solutions

From (i) and (ii):


4 2
a+ c = 2c => a = c . . . (iii)
3 3
Thus, the required percent value
a
= 100
a+b+c
2
 
c
3  100
=
2 4
c+ c+c
3 3
200
= = 22.22% Sufficient
9
The correct answer is option C.

50. Total people = 108.

Since the ratio of the number of men and women is 5 : 4, we have:


5
 
Number of men = 108 = 60
5+4
4
 
Number of women = 108 = 48
5+4
We draw the Venn-diagrams separately for men and women:

Morning Evening Morning Evening

= 0 = 0
Men = 60 Women = 48

We know that:
a+b a+b 1
 
100 = 50 => = => a + b = 30
a+b+c 60 2
=> a + b = c = 30 . . . (i)

We need to determine the value of (b + e).

From statement 1:

www.manhattanreview.com 19992016 Manhattan Review


Solutions 171

We know that:

The number of men who participate in the safari only in the morning is 80 percent of the

women who participate in the safari only in the morning


80 5
=> a = d => d = a . . . (ii)
100 4
Also, the number of men who participate in the safari only in the evening is double the

number of women who participate in the safari only in the evening

=> c = 2f . . . (iii)

However, using (i), (ii) and (iii), the value of (b + e) cannot be determined. Insufficient

From statement 2:

We know that:

The number of women who participate in the safari only in the evening is 60 percent of

the girls who participate in the safari only in the morning


60 3
=> f = d => f = d . . . (iv)
100 5
However, using (i) and (iv), the value of (b + e) cannot be determined. Insufficient

Thus, from both statements together:

From (i) and (iii): c = 2f = 30 => f = 15 . . . (v)


3
From (iv) and (v): d = 15 => d = 25 . . . (vi)
5
5
From (ii) and (vi): a = 25 => a = 20 . . . (vii)
4
From (i) and (vii): 20 + b = 30 => b = 10 . . . (viii)

For women, we have: d + e + f = 48 => e = 48 25 15 = 8 . . . (ix)

Thus, from (viii) and (ix):

b + e = 18. Sufficient

The correct answer is option C.

Alternate Approach:

We can fill up the Venn-diagrams as shown:

19992016 Manhattan Review www.manhattanreview.com


172 Solutions

Morning Evening Morning Evening

$ * /
= = 2 = =
% % %

= 0 = 0
Men = 60 Women = 48

60 6
Since a + b = c = = 30, we have: d = 30 => d = 25.
2 5
Thus, the completed Venn-diagram is:

Morning Evening Morning Evening

= 20 = 30 = 25 = 15

= 0 = 0
Men = 60 Women = 48

Thus: b = 60 (20 + 30) = 10 and e = 48 (25 + 15) = 8

=> b + e = 18.

51. We know that f (x) is a linear function

=> f (x) = ax + b, where a and b are constants.

From statement 1:

f (f (x)) = f (ax + b)

= a (ax + b) + b

= a2 x + (ab + b)

Thus, we have:

a2 x + (ab + b) = 9x + 8

Comparing the coefficients on either side:

www.manhattanreview.com 19992016 Manhattan Review


Solutions 173

a2 = 9 => a = 3

ab + b = 8

=> 3b + b = 8 => b = 2

OR

3b + b = 8 => b = 4

Thus, we have:

f (x) = 3x + 2 => f (2) = 8

OR

f (x) = 3x 4 => f (2) = 10

Thus, there is no unique answer. Insufficient

From statement 2:

Since we have no information on the function f (x), we cannot determine the value of

f (x). Insufficient

Thus, from both statements together:

If f (x) = 3x + 2 => f (2) = 8 => f (f (2)) = 3 8 + 2 = 26

If f (x) = 3x 4 => f (2) = 10 => f (f (2)) = 3 (10) 4 = 26

Thus, we do not have a unique function f (x). Insufficient

The correct answer is option E.

52. From statement 1:

We know that:

f (a + b) = f (a) + f (b)

=> f (2) = f (1 + 1) = f (1) + f (1) = 2f (1) = 4

=> f (1) = 2

Again: f (4) = f (2 + 2) = f (2) + f (2) = 4 + 4 = 8

19992016 Manhattan Review www.manhattanreview.com


174 Solutions

Thus: f (5) = f (1 + 4) = f (1) + f (4) = 2 + 8 = 10. Sufficient

From statement 2:

We know that:

f (a + b) = f (a) + f (b)

=> f (2a) = f (a + a) = f (a) + f (a) = 2f (a)

Thus, the statement does not provide any additional information using which the value

of f (5) can be determined. Insufficient

The correct answer is option A.

53. From statement 1:

We have:
 2
f (x) = 2f (x) 1
 2
=> f (x) 2f (x) + 1 = 0
 2
=> f (x) 1 = 0

=> f (x) = 1

Thus, f (x) is a constant function having value 1

=> f (2) = 1. Sufficient

From statement 2:

We have:

f (4x) = 8x 2 1
1
To find the value of f (2), we substitute x = :
2
 2
1 1
 
f 4 =8 1
2 2
=> f (2) = 2 1 = 1. Sufficient

The correct answer is option D.

54. Since f (x) is a linear function, let us have: f (x) = ax + b

www.manhattanreview.com 19992016 Manhattan Review


Solutions 175

From statement 1:

f (x 3) = a (x 3) + b

Since f (x 3) = f (x) + 1, we have:

a (x 3) + b = (ax + b) + 1

=> ax 3a + b = ax + b + 1
1
=> a =
3
However, the value of b cannot be determined.
1
 
Thus, f (1) = a + b = + b
3
Since the value of b is not known, the value of f (1) cannot be determined. Insufficient

From statement 2:

Since f (5) = 2f (3), we have:

5a + b = 2 (3a + b)

=> a + b = 0

Thus, we have:

f (1) = a 1 + b = a + b = 0. Sufficient

The correct answer is option B.

55. From statement 1:

We have:

Let us express the RHS in terms of (x 1):

=> f (x 1) = 2x + 4 = 2 (x 1) + 6

=> f (x) = 2x + 6

However, there is no information about g (x). Insufficient

From statement 2:

There is no information about f (x). Insufficient

Thus, from both statements together:

19992016 Manhattan Review www.manhattanreview.com


176 Solutions

Since f (x) = 2x + 6, we have:


 
f g (x) = 2 g (x) + 6

=> 4x = 2 g (x) + 6

=> g (x) = 2x 3

=> g (2) = 2 2 3 = 1. Sufficient

The correct answer is option C.

56. Let the two-digit number N be 10x + y, where x and y are the digits.

We know that 1 x 9 and 0 y 9. If no constraint given for S and for P , the

largest number would be 99.

The sum of the digits of the number = S = x + y.

The product of the digits = P = xy.

From statement 1:

We know that:

N + S = 103

=> 10x + y + x + y = 103

=> 11x + 2y = 103

Since we need to determine the largest possible value of N, we should assign x = 9.

Thus, we have:

99 + 2y = 103

=> y = 2 (integer)

Thus, the largest possible value of N is 92. Sufficient

From statement 2:

We know that:

2N = 2S + 9P
 
=> 2 10x + y = 12 x + y + 9xy

www.manhattanreview.com 19992016 Manhattan Review


Solutions 177

=> 20x + 2y = 2x + 2y + 9xy

=> 18x = 9xy

=> y = 2

Thus, the two-digit number N has 2 as the units digit.

Since there is no constraint over the value of x, we can assume it to be 9, thus the largest

possible value of N is 92. Sufficient

The correct answer is option D.

57. Let the number of marbles with P and Q be p and q, respectively.

From statement 1:

p = q + 10 . . . (i)

However, the exact number of marbles with P cannot be determined.

From statement 2:

If P gives 10 marbles to Q, then:



Number of marbles with P = p 10 .

Number of marbles with Q = q + 10 .

Since P now has 10 marbles less than what Q has, we have:



p 10 = q + 10 10

=> p = q + 10

The above is the same condition as obtained from the first statement. Insufficient

Thus, from both statements together:

Since both statements result in the same equation, we cannot solve for p. Insufficient

The correct answer is option E.

58. We know that the equation px + qy = r represents the equation of a straight line.

The solution of a pair of linear equation represents the x and y coordinates of the

point where the lines intersect.

19992016 Manhattan Review www.manhattanreview.com


178 Solutions

If two equations have infinite solutions, it implies that the lines corresponding to those

equations intersect at infinite points, which is only possible if the two lines are the same,

i.e. the equations are the same.

Thus, ax + 2y = 6 and bx + cy = 9 represent the same line.

Thus, the coefficients of the two equations are in the same ratio
a 2 6
=> = =
b c 9
a 2 2
=> = = . . . (i)
b c 3
From statement 1:

We have: c = 3
2 2
However, from (i), we can see that it is obvious, since: = => c = 3.
c 3
Though we get a = 2 and b = 3 => a + b = 5, but these are ratio values and not the
6 2
actual values. Had we not reduced to , we might have a + b = 6 + 9 = 15 as well.
9 3
Thus, the first statement does not offer any new information. Insufficient

From statement 2:

We have: b = a + 1

Thus, from (i), we have:


a 2
=
a+1 3
=> a = 2

=> b = a + 1 = 3

=> a + b = 5. Sufficient

The correct answer is option B.

59. Let the number of marbles with A, B and C be a, b and c, respectively.

From statement 1:

We have:
1
a= (b + c)
3

www.manhattanreview.com 19992016 Manhattan Review


Solutions 179

=> b + c = 3a . . . (i)

From this information alone, the answer cannot be determined. Insufficient

From statement 2:

We have:
1
b= (a + c)
4
=> a + c = 4b . . . (ii)

From this information alone, the answer cannot be determined. Insufficient

Thus, from both statements together:

Adding a to both sides in (i):


1
a + b + c = 4a => a = (a + b + c)
4
Assuming (a + b + c) = t, we have:
t
a= . . . (iii)
4
Adding b to both sides in (ii):
1
a + b + c = 5b => b = (a + b + c)
5
t
=> b = . . . (iv)
5
Thus, we have:
t t
 
c = t (a + b) = t +
4 5
11t
=> c = . . . (v)
20
11t t
=> c a =
20 4
3t
=> c a =
10
We need to determine if c a = 10.

If that were true, we would have:


3t 1
= 10 => t = 33
10 3
However, the number of marbles cannot be a fractional number.

Thus, the answer to the problem is No. Sufficient

19992016 Manhattan Review www.manhattanreview.com


180 Solutions

The correct answer is option C.

C can have more number of marbles than what A has by a number that is a multiple of

3: 3, 6, 9 (less than 10), 12, 15, . . . .. any number (more than 10), but not EXACTLY 10!

60. Let the price of an apple and an orange be $a and $b, respectively.

From statement 1:

We have:

10a + 15b = 8 . . . (i)


8 15
=> a = b
10 10
8 b
 
=> a = b+
12 2
From this equation alone, we cannot compare the price of an apple and an orange.

Insufficient

From statement 2:

We have:

5a = 6b + 1.30
6 1.30
=> a = b+
5 5
b
 
=> a = b + + 0.26
5
=> a > b. Sufficient

The correct answer is option B.

Alternatively, a fewer number of apples (5) cost more ($1.30) than relatively more number

of oranges (6), thus an apple is priced higher than an orange.

61. Since the average of 10 odd integers is 55, their total = 10 55 = 550.

From statement 1:

We know that the largest integer is 65.

Thus, the sum of the remaining 9 integers = 550 65 = 485.

www.manhattanreview.com 19992016 Manhattan Review


Solutions 181

However, the median cannot be determined. Insufficient

From statement 2:

We know that the smallest integer is 41.

Thus, the sum of the remaining 9 integers = 550 41 = 509.

However, the median cannot be determined. Insufficient

Thus, from both statements together:

Apart from the smallest and the largest integers, the sum of the remaining 8 integers

= 550 41 65 = 444.

The odd integers between 41 and 65 are:

43, 45, 47, 49, 51, 53, 55, 57, 59, 61 and 63

Sum of the above integers = 43+45+47+49+51+53+55+57+59+61+63 = 1153 = 583.

We need to select 8 integers from the above 11 integers which will add up to 444.

Thus, we need to leave out 3 integers which add up to 583 444 = 139.

If we select 43 as the LEAST such integer, the other 2 integers add up to 139 43 =

96, i.e. they are: 47, 49 OR 45, 51.

If the 3 integers to be left out are: 43, 47, 49, the integers in the set are:

41, 43, 45, 47, 49, 51, 53, 55, 57, 59, 61, 63, 65
55 + 57
=> Median of the 10 integers = Average of 5th and 6th numbers = = 56.
2
If the 3 integers to be left out are: 43, 45, 51, the integers in the set are:

41, 43, 45, 47, 49, 51, 53, 55, 57, 59, 61, 63, 65
55 + 57
=> Median of the 10 integers = Average of 5th and 6th numbers = = 56.
2

If we select 45 as the LEAST such integer, the other 2 integers add up to 139 45

= 94, which is not possible using distinct odd integers greater than 45 (since both

integers would have to be 47, which is not possible).

Thus, for any number greater than 43 as the LEAST such number, there is no possi-

ble solution.

19992016 Manhattan Review www.manhattanreview.com


182 Solutions

Thus, the median is 56, a unique value. Sufficient

The correct answer is option C.

62. Let the smallest number be x.

Since the range is 12, the largest integer = (x + 12).

From statement 1:

Since the average of 6 integers is 50, their total = 6 50 = 300.

However, there is no information about the remaining numbers. Insufficient

Note: One may be tempted to think that since we need the smallest number, the other

numbers should have the maximum value.

Thus, the numbers are x, (x + 12) , (x + 12) , (x + 12) , (x + 12) and (x + 12).

Thus, the sum of the numbers = 6x + 60 = 300 => x = 40.

However, this only gives us the least possible value of the smallest number and NOT

necessarily the value of the smallest number in the particular problem.

From statement 2:

We know that the numbers have the least possible standard deviation.

In order to have the least standard deviation, the numbers should be as close as possible

to the mean.

However, for a given range and minimum value of the standard deviation, there can be

infinitely possible values of the numbers, since the mean is not known.

For example: We may have the set of integers as 1, 7, 7, 7, 7, 13 could be a set with

range 12, mean 7 and having minimum value of the standard deviation. However, we

may also have the set of integers as 10, 16, 16, 16, 16, 22, with range 12, mean 7 and

having minimum value of the standard deviation.

Note: How do we generate the integers?

Since the standard deviation needs to be minimized, we put 4 of the integers equal to the

mean and keep the last 2 integers at equal distances from the mean, so that the range

www.manhattanreview.com 19992016 Manhattan Review


Solutions 183

12
comes out as 12. Thus, the smallest integer should be = 6 less than the mean and
2
the largest integer should be 6 greater than the mean.

Thus, the value of the smallest integer cannot be uniquely determined. Insufficient

Thus, from both statements together:

We know that the mean is 50.

Thus, in order to minimize the standard deviation, we put 4 of the integers as 50 and the

remaining 2 integers can be used to satisfy the range as 12.

Since the standard deviation needs to be minimized, we should keep the last 2 integers

at equal distances from the mean.


12
Thus, the smallest integer should be = 6 less than the mean, i.e. 50 6 = 44, and the
2
largest integer should be 6 greater than the mean, i.e. 50 + 6 = 56.

Thus, the integers are: 44, 50, 50, 50, 50 and 56.

Thus, the value of the smallest number is 44. Sufficient

The correct answer is option C.

63. From statement 1:

Since the mean of each group is the same, it must be the same as the mean of all 8

numbers.
1+2+3+4+5+6+7+8 1+8 1
The mean of the numbers 1 to 8 = = =4 .
8 2 2
1
Thus, the sum of the numbers in each group = 4 4 = 18.
2
Thus, randomly picking 2 numbers for the first group would automatically give us the

other numbers in the two groups. Two of the possible ways of forming the groups are:

(1, 2, 7, 8) and (3, 4, 5, 6):


2+7 1 4+5 1
Medians of the groups are = 4 and = 4 , respectively
2 2 2 2
=> Difference in median is 0.

(2, 3, 5, 8) and (1, 4, 6, 7):

19992016 Manhattan Review www.manhattanreview.com


184 Solutions

3+5 4+6
Medians of the groups are = 4 and = 5, respectively
2 2
=> Difference in median is 1.

Thus, the required answer cannot be uniquely determined. Insufficient

From statement 2:

We need to form the groups such that 1 and 8 do not fall in the same group.

There are many such possibilities, for example:

(1, 2, 3, 4) and (5, 6, 7, 8)

(1, 3, 5, 7) and (2, 4, 6, 8)

Thus, the required answer cannot be uniquely determined. Insufficient

Thus, from both statements together:


1
We need to form groups having the same mean, which is 4 , i.e. the sum of each group
2
is 18 and also ensure that 1 and 8 do not fall in the same group.

Let the groups be: (1, __, __, __) and (__, __, __, 8).

Sum of the remaining three numbers in the second group = 18 8 = 10.

The numbers remaining are: 2, 3, 4, 5, 6 and 7.

The only possibility is: 2, 3 and 5.

Thus, the other three numbers, i.e. 4, 6 and 7 must belong to the first group.

Thus, the groups are: (1, 4, 6, 7) and (2, 3, 5, 8).


4+6 3+5
Medians of the groups are = 5 and = 4, respectively
2 2
=> Difference in median is 1. Sufficient

The correct answer is option C.

64. From statement 1:

The numbers having a constant difference of 2 forms two series:

1, 3, 5, 7, 9, 11, 13, 15

www.manhattanreview.com 19992016 Manhattan Review


Solutions 185

2, 4, 6, 8, 10, 12, 14, 16

Thus, the groups are: (1, 3, 5, 7), (9, 11, 13, 15), (2, 4, 6, 8) and (10, 12, 14, 16)
3+5 11 + 13 4+6
The respective medians of the groups are: = 4, = 12, = 5 and
2 2 2
12 + 14
= 13
2
Thus, the sum of the medians = 4 + 12 + 5 + 13 = 34. Sufficient

From statement 2:

The numbers having a constant difference of 4 forms four series:

1, 5, 9, 13

2, 6, 10, 14

3, 7, 11, 15

4, 8, 12, 16

Thus, the groups are: (1, 5, 9, 13), (2, 6, 10, 14), (3, 7, 11, 15) and (4, 8, 12, 16)
5+9 6 + 10 7 + 11
The respective medians of the groups are: = 7, = 8, = 9 and
2 2 2
8 + 12
= 10
2
Thus, the sum of the medians = 7 + 8 + 9 + 10 = 34. Sufficient

The correct answer is option D.

1
65. Sum of the 6 integers = 6 15 = 93.
2
From statement 1:

We only know that the numbers are distinct.

The largest number cannot be uniquely determined. Insufficient

From statement 2:

We know that the smallest number is 13.

Thus, the possibilities of the other numbers are:

13, 13, 13, 13, and 93 5 13 = 28 (largest)

OR

19992016 Manhattan Review www.manhattanreview.com


186 Solutions

14, 15, 16, 17 and 93 (13 + 14 + 15 + 16 + 17) = 18 (largest)

Thus, the largest number cannot be uniquely determined. Insufficient

Thus, from both statements together:

Since the numbers are distinct, and the smallest number is 13, the smallest possible

values of the other numbers are: 14, 15, 16, 17 and 18

Sum of the 6 numbers = 13 + 14 + 15 + 16 + 17 + 18 = 93, which matches with the actual

total of the 6 numbers.

Thus, the 6 numbers must be 13, 14, 15, 16, 17 and 18.

Thus, the largest number is 18. Sufficient

The correct answer is option C.

66. From statement 1:



2x + 2 x3=2

=> 2x + 2 = x 3 + 2

Squaring both sides of the above equation, we have:


2
2x + 2 = x3+2

=> 2x + 2 = (x 3) + 22 + 2 2 x 3

=> x + 1 = 4 x 3

Squaring both sides of the above equation, we have:

x 2 + 2x + 1 = 16 (x 3)

=> x 2 14x + 49 = 0

=> (x 7)2 = 0

=> x = 7. Sufficient

From statement 2:

The value of x cannot be determined only from x > 0. Insufficient

The correct answer is option A.

www.manhattanreview.com 19992016 Manhattan Review


Solutions 187

67. The roots of ax 2 + bx + c = 0 are p and q.

Thus, p and q satisfy the given quadratic equation.

=> ap 2 + bp + c = 0 . . . (i)

aq2 + bq + c = 0 . . . (ii)

Thus, we have:

ap 2 + bp + c = aq2 + bq + c

=> ap 2 aq2 + bp bq = 0

=> a p 2 q2 + b p q = 0
 

  
=> a p + q pq +b pq =0
 
=> p q a p+q +b =0

Since p 6= q, i.e. (p q) 6= 0, we have:



a p+q +b =0
b
=> p + q = . . . (iii)
a
Adding (i) and (ii), we have:

a p 2 + q2 + b p + q + 2c = 0
 

n 2 o 
=> a p + q 2pq + b p + q + 2c = 0
2 
=> a p + q + b p + q 2pqa + 2c = 0
 
=> p + q a p + q + b 2pqa + 2c = 0

=> 2pqa + 2c = 0 (Since, from (iii), we have: a p + q + b = 0)
c
=> pq = . . . (iv)
a
Note: In the quadratic equation ax 2 + bx + c, with roots are p and q, we have:

b
Sum of the roots = p + q =
a
c
Product of the roots = pq =
a

We need to determine the value of:


 
p+1 q+1

19992016 Manhattan Review www.manhattanreview.com


188 Solutions


= pq + p + q + 1
c b
= +1
a a
bc
 
= +1
a
From statement 1:

The value of a is not known. Insufficient

From statement 2:

The value of (b c) is not known. Insufficient

Thus, from both statements together:


 
p+1 q+1
bc
 
= +1
a
2 1
= + 1 = . Sufficient
3 3
The correct answer is option C.

68. From statement 1:

Let us assume 2x = k.

Thus, we have: 4x = (2x )2 = k2 .

Hence, our equation is modified to:

k2 20k + 64 = 0

=> k2 16k 4k + 64 = 0

=> (k 16) (k 4) = 0

=> k = 4 OR 16.

Thus, we have:

2x = 4 = 22 OR 2x = 16 = 24

=> x = 2 OR 4.

Thus, there is no unique answer. Insufficient

www.manhattanreview.com 19992016 Manhattan Review


Solutions 189

From statement 2:

4x = x 4

The solutions of the above are x = 2 (42 = 16 = 24 ) and x = 4 (44 = 44 ).

Thus, there is no unique answer. Insufficient

Thus, from both statements together:

Even after combining both statements, we still have: x = 2 OR 4. Insufficient

The correct answer is option E.

69. From statement 1:


a b
+ =1
xa xb
=> a (x b) + b (x a) = (x a) (x b)

=> x (a + b) 2ab = x 2 x (a + b) + ab

=> x 2 2x (a + b) + 3ab = 0

Since the roots of the above quadratic equation are equal in magnitude, but opposite in

sign, let the roots be p and p.

Thus, we have:

x + p x 2 2x (a + b) + 3ab
 
xp

=> x 2 (0) x p 2 x 2 2x (a + b) + 3ab

Comparing the coefficient of x, we have:

0 = 2 (a + b)

=> a + b = 0.

Thus, the answer is Yes. Sufficient

From statement 2:

x (x a) + x (x b) = 0

=> x {(x a) + (x b)} = 0

=> x (2x a b) = 0

19992016 Manhattan Review www.manhattanreview.com


190 Solutions

a+b
 
=> x = 0 OR .
2
Since the roots of the quadratic equation are equal, we have:
a+b
=0
2
=> a + b = 0.

Thus, the answer is Yes. Sufficient

The correct answer is option D.

70. From statement 1:

Since 1 and 2 are roots of f (x) = 0, we have:

f (1) = 0 => 1 p + q r = 0 . . . (i)

f (2) = 0 => 8 4p + 2q r = 0 . . . (ii)

Subtracting (i) from (ii):

7 3p + q = 0

=> 3p q = 7. Sufficient

From statement 2:

Since 3 is a root of f (x) = 0, we have:

f (3) = 0 => 27 9p + 3q r = 0

=> 9p 3q = 27 r
1
=> 3p q = (27 r )
3

Since the value of r is unknown, the value of 3p q cannot be determined. Insuffi-

cient

The correct answer is option A.

71. From statement 1:

Since the password contains vowels (V) and digits (D) in alternate positions, there are

two possibilities:

www.manhattanreview.com 19992016 Manhattan Review


Solutions 191

VDVDVDV:

There are 4 distinct vowels, which can be chosen from 5 vowels (A, E, I, O, U) in

C45 = 5 ways and 3 distinct digits, which can be chosen from 10 digits (0, 1, 2, 3, 4,
10 9 8
5, 6, 7, 8, 9) in C310 = = 120 ways.
3!
The digits may be arranged in 3! = 6 ways and the vowels in 4! = 24 ways.

Thus, the total number of ways = 5 120 6 24 = 86400 . . . (i)

DVDVDVD:

There are 3 distinct vowels, which can be chosen from 5 vowels (A, E, I, O, U) in
543
C35 = = 10 ways and 4 distinct digits, which can be chosen from 10 digits
3!
10 9 8 7
(0, 1, 2, 3, 4, 5, 6, 7, 8, 9) in C410 = = 210 ways.
4!
Note: 0can be placed at first place as here we are talking about a password, not a

number.

The digits may be arranged in 4! = 24 ways and the vowels in 3! = 6 ways.

Thus, the total number of ways = 10 210 6 24 = 302400 . . . (ii)

Thus, there is no distinct answer. Insufficient

From statement 2:

There is no information about the remaining characters to be used. Insufficient

Thus, from both statements together:

Since the password must start with a digit, and should only have vowels and digits, the

only possible situation is as given by the second case for the first statement, i.e. total

number of ways is 302400. Sufficient

The correct answer is option C.

72. From statement 1:

We have no information whether a code-word can have the same letter occurring multiple

times. Insufficient

From statement 2:

19992016 Manhattan Review www.manhattanreview.com


192 Solutions

We have no information about the number of letters to be used to form a code-word.

Insufficient

Thus, from both statements together:

We need to form code-words having 4 letters such that only letter S can be used at most

3 number of times and all others only once.

Each letter of SYSTEMS is used only once:

Thus, there are 5 distinct letters (S, Y, T, E, M), from which 4 letters can be selected

in C45 = 5 ways.

Each of the above words can be arranged in 4! = 24 ways.

Thus, the number of ways = 5 24 = 120.

The letter S is used twice:

Thus, there are 4 remaining letters (Y, T, E, M), from which 2 letters can be selected

in C24 = 6 ways.
4!
Each of the above words can be arranged in = 12 ways.
2!
Thus, the number of ways = 6 12 = 72.

The letter S is used thrice:

Thus, there are 4 remaining letters (Y, T, E, M), from which 1 letter can be selected

in C14 = 4 ways.
4!
Each of the above words can be arranged in = 4 ways.
3!
Thus, the number of ways = 4 4 = 16.

Thus, the total number of code-words = 120 + 72 + 16 = 208. Sufficient

The correct answer is option C.

73. Let the number of women be x.

Thus, the number of men = (x + 1).

From statement 1:

Let us first arrange the men. Then, we place the women in the gaps created between the

men or at the two ends, so that no two women sit beside one another.

www.manhattanreview.com 19992016 Manhattan Review


Solutions 193

The men may be arranged in (x + 1)! ways.

As a result, there are (x + 2) positions (including the two ends) created for the women

to be placed.

(x+2)
For x women, we select x out of the (x + 2) positions in Cx ways.

The women may now be arranged in x! ways.

Thus, total number of ways

(x+2)
= (x + 1)! Cx x!
(x + 2)!
= (x + 1)! x!
x! 2!
1
= (x + 1)! (x + 2)!
2
However, we have no information about the number of ways in which the people can be

seated. Insufficient

From statement 2:

There is no information about the pattern followed in deciding the seating arrangement

of the men and women. Insufficient

Thus, from both statements together:

We have:
1
(x + 1)! (x + 2)! = 3 (5!) (7!)
2
1 1 1
=> (x + 1)! (x + 2)! = 6 (5!) (7!) = 6! 7!
2 2 2
Comparing the above equation, we have: x = 5.

Thus, the number of men and women are (x + 1) = 6 and x = 5, respectively.

Thus, the total number of people = 6 + 5 = 11. Sufficient

The correct answer is option C.

74. From statement 1:

Since the men and women sit in alternate positions along a circle, it is implied that the

number of men and women is the same.

19992016 Manhattan Review www.manhattanreview.com


194 Solutions

Let the number of men and the number of women be x each.

In order to make them sit along a circle, first the men can be seated in (x 1)! ways and

the women can sit in between the men in x! ways.

Thus, number of sitting arrangements along a circle = (x 1)! x!

In order to make them sit in a row, first the men can be seated in x! ways and the women

can sit in between the men in x! ways.

However, the first position can be assigned to a man or a woman, i.e. in 2 ways.

Thus, number of sitting arrangements in a row = 2 x! x!

(Note: For example, if there are 5 men (M) and 5 women (W), the possibilities are

MWMWMWMWMW or WMWMWMWMWM)

Thus, the required ratio:


(x 1)! x! 1
=
2 x! x! 10
1 1
=> =
2 (x 1) 10
=> x = 6

Thus, the number of man and women are 6 each, hence the total number of people = 12.

Sufficient

From statement 2:

Let there be a men and b women.

Number of handshakes among the men = C2a .

Number of handshakes among the women = C2b .

Since we know that: C2a = C2b , it follows that a = b.

Thus, number of handshakes between the men and women, i.e. a man shakes hands only

with women = C1a C1a = a2 .

Thus, we have:
5 2
C2a = a
12
a (a 1) 5 2
=> = a
2 12

www.manhattanreview.com 19992016 Manhattan Review


Solutions 195

=> 6 (a 1) = 5a

=> a = 6

Thus, the number of man and women are 6 each, hence the total number of people = 12.

Sufficient

The correct answer is option D.

75. Let the number of students selected in the group be x.

From statement 1:

Number of ways of selecting x students from 25 students = Cx25 .

We know that the number of possible selections of the x students to form the group is

the maximum possible number.

Thus, the value of x is such that Cx25 has the largest possible value among all possible

values of Cx25 .

We know that Cxn has the largest possible value if:

n
x= , if n is an even number
2
n1
x= , if n is an odd number
2
25 1
Since 25 is an odd number, Cx25 will have the largest possible value if x = = 12
2
OR 13.

Thus, the value of x cannot be uniquely determined. Insufficient

From statement 2:
25
We only know that: x < . Insufficient
2
Thus, from both statements together:

x = 12. Sufficient

The correct answer is option C.

76. From statement 1:

19992016 Manhattan Review www.manhattanreview.com


196 Solutions

We cannot determine the number of apples and oranges purchased. Insufficient

From statement 2:

We cannot determine the number of apples and oranges purchased. Insufficient

Thus, from both statements together:

Let the number of apples be p and the number of oranges be r .

Thus, we have: 5p + 2r = 27.

We know that p and r must be positive integers.

Thus, we need to find positive integer solutions of the equation: 5p + 2r = 27.

A starting solution is p = 5 and e = 1.

Other acceptable solutions are shown below:

p r

Reduce the value of 5 1 Increase the value of

p by the coefficient r by the coefficient


(= 5 2) 6 (= 1 + 5)
of r i.e. 2, to generate of p i.e. 5, to generate

the next value of p the next value of r


1 (= 3 2) 11 (= 6 + 5)

Hence, there are three possible solutions, among which, the number of apples is greater

than the number of oranges (i.e. p > r ) is valid in only the first solution (5 apples and 1

orange).
# of possible cases in which # of apples > # of or anges
Hence, the required probability = =
total # of cases
1
. Sufficient
3
The correct answer is option C.

77. From statement 1:

Since the cards are drawn in succession with replacement, there are n n = n2 ways of

drawing two cards.

www.manhattanreview.com 19992016 Manhattan Review


Solutions 197

Of these n2 cases, there would be n cases where the same number would appear on both

draws viz. (1, 1); (2, 2); (3, 3) . . . (n, n).

Thus, there are n2 n cases left.




n2 n

Of these, in exactly half of them i.e. cases, the first card would have a larger
2
n2 n

number than the second card and in the remaining cases, the second card
2
would have a larger number than the first card.
n2 n

Thus, there are favorable cases out of a total of n2 cases.
2
!
n2 n
2 n1
 
Hence, required probability = = .
n2 2n
However, the value of n is not known. Insufficient

From statement 2:

We know that n = 10.

However, whether the cards are drawn with replacement or without replacement is not

known:

If the cards are drawn with replacement, we have already seen above that the prob-
n1 10 1 9
 
ability is = = .
2n 2 10 20
If the cards are drawn without replacement, then there is no case where the same

number would be drawn in both cases. Thus, in exactly 50% of the cases, the number

on the first card will be less than the number on the second card, i.e. the required
1
probability is .
2

Thus, there is no unique answer. Insufficient

Thus, from both statements together:


n1 10 1 9
The required probability = = = . Sufficient
2n 2 10 20
The correct answer is option C.

78. From statement 1:

Let the probability of the dice showing an odd number is x.

19992016 Manhattan Review www.manhattanreview.com


198 Solutions

Thus, the probability of the dice showing an even number = 2x.

Thus, the total probability that the dice shows an odd number or an even number =

x + 2x = 3x.
1
Thus, we have: 3x = 1 => x = .
3
1
Thus, the probability of the dice showing an odd number is
3
1
=> The probability of the dice showing 1 or 3 or 5 is
3
Since the faces 1, 3 and 5 are equally likely, we have:

Probability of the dice showing 1 = Probability of the dice showing 3 = Probability of the
1 1 1
dice showing 5 = = .
3 3 9
2
Similarly, the probability of the dice showing an even number is 2x =
3
2
=> The probability of the dice showing 2 or 4 or 6 is
3
Since the faces 2, 4 and 6 are equally likely, we have:

Probability of the dice showing 2 = Probability of the dice showing 4 = Probability of the
1 2 2
dice showing 6 = = .
3 3 9
Thus, the probability that the man goes ahead with the course of action

= Probability that the dice shows a prime number

= Probability that the dice shows 2 or 3 or 5


2 1 1 4
= + + = . Sufficient
9 9 9 9
From statement 2:
1
We only know that the probability of the dice showing 1 is less than .
6
This is not sufficient to answer the question. Insufficient

The correct answer is option A.

79. From statement 1:

We know that any integer, when divided by 5, leaves remainders: 1, 2, 3, 4 or 0.

We need to find the integers which leave a prime remainder, i.e. 2 or 3.

www.manhattanreview.com 19992016 Manhattan Review


Solutions 199

We know that the numbers in set P are consecutive positive integers.

Since the range is 10, the difference between the largest and the smallest integers in set

P is 10.

Thus, we may have different situations:

The integers in set P are: 1, 2, 3, 4, . . . 10, 11:

The integers which leave a prime remainder are obviously 2, 3, 7 and 8.

Thus, there are 4 such integers from 1 to 11 which leave a prime remainder when

divided by 5.
4
Thus, required probability = .
11
The integers in set P are: 2, 3, 4, 5, . . . 11, 12:

The integers which leave a prime remainder are obviously 2, 3, 7, 8 and 12.

Thus, there are 5 such integers from 2 to 12 which leave a prime remainder when

divided by 5.
5
Thus, required probability = .
11

Thus, the probability cannot be uniquely determined. Insufficient

Note: The reason why the probability changes is that we do not know the nature of the

first number or the last number in the set P.

From statement 2:

We know that the largest integer in the set P is 35.

However, we have no information about the number of integers in the set P. Insufficient

Thus, from both statements together:

We know that the last number is 35. Also, the range is 10. Thus, the set is:

25, 26, 27, 28, 29, 30, 31, 32, 33, 34 and 35

The integers which leave a prime remainder are obviously 27, 28, 32 and 33.
4
Thus, required probability = . Sufficient
11
The correct answer is option C.

19992016 Manhattan Review www.manhattanreview.com


200 Solutions

80. From statement 1:

We do not know the marks assigned for correct or wrong responses to a question.

Insufficient

From statement 2:

We only know the marks assigned for correct or wrong responses to a question.

There is no other information with which the value of n may be determined. Insufficient

Thus, from both statements together:


1
 
Since there are n options, the probability of randomly choosing a correct answer = .
n
1 n1
   
Thus, probability of choosing a wrong answer = 1 = .
n n
Thus, it implies that:

Out of every n attempts, 1 attempt would be correct and (n 1) attempts would be

wrong.

We know that for every correct response, 1 mark is assigned and for every wrong re-
1
sponse, marks are assigned.
3
Thus, out of n attempts:

1 correct attempt results in 1 1 = +1 mark


1 n1
   
(n 1) wrong attempts result in (n 1) = marks.
3 3

Thus, average marks obtained


n1
 
1
3 3 (n 1)
= =
n 3n
4n
 
=
3n
Since the expected i.e. average mark is 0, we have:
4n
= 0 => n = 4. Sufficient
3n
The correct answer is option C.

Alternate Approach:
1
 
Since there are n options, the probability of randomly choosing a correct answer = .
n

www.manhattanreview.com 19992016 Manhattan Review


Solutions 201

1 n1
   
Thus, probability of choosing a wrong answer = 1 = .
n n
We know that for every correct response, 1 mark is assigned and for every wrong re-
1
sponse, marks are assigned.
3
Since the candidate expects 0 marks, we have:
1 n1 1
     
(1) + =0
n n 3
1 n1
=> =0
n 3n
=> 3 (n 1) = 0

=> n = 4.

81. From statement 1:

The location of point D is not known. Sufficient

From statement 2:

The measure of ACB is not known. Insufficient

Thus, from both statements together:


A

D
6
60#
60#
C B
6

Triangle ACB is isosceles, since AC = BC.


(180 ACB)
Thus, ABC = BAC = = 30
2
Thus, in triangle BCD, CDB = 180 (60 + 30 ) = 90

Thus, triangle BCD is a 30-60-90 triangle



=> CD : BD : CB = 1 : 3:2

Since BC = 6, we have: CD = 3. Sufficient

The correct answer is option C.

19992016 Manhattan Review www.manhattanreview.com


202 Solutions

82.
P T

Q S U

From statement 1:

We have no information about the orientations of the lines PQ, RS and TU, i.e. whether

they are parallel, etc. Insufficient

Note: From the diagram it appears as if PQ, RS and TU are perpendicular to QU and so

they are parallel, but such an assumption should NOT be made.

From statement 2:

We have no information about the dimensions of any of the lines. Insufficient

Thus, from both statements together:

Since PQ is parallel to RS, triangles PQU and RSU are similar

PQ QU
=> = . . . (i)
RS SU
Since TU is parallel to RS, triangles TQU and RQS are similar

TU QU
=> = . . . (ii)
RS SQ
Dividing (i) and (ii):

PQ SQ 28 4
= = =
TU SU 21 3
SQ 4
=> +1 = +1
SU 3
SQ + SU 7
=> =
SU 3
QU 7
=> =
SU 3
Thus, from (i):

PQ QU 7 28 7
= = => = => RS = 12. Sufficient
RS SU 3 RS 3
The correct answer is option C.

www.manhattanreview.com 19992016 Manhattan Review


Solutions 203

83.
A




C D E

From statement 1:

Since in triangle BCD, BC = CD = DB, we have:

BCD = BDC = CBD = 60

=> g = c = 60 . . . (i)

=> BDE = 180 60 = 120

Since in triangle BDE, BD = DE, i.e. the triangle is isosceles, we have:

180 120
DEB = DBE = = 30
2

=> e = 30 . . . (ii)

=> P = g + c e = 60 + 60 30 = 90 . . . (iii)

=> CBE = CBD + DBE = 60 + 30 = 90

=> In triangle ABE, ABE = 90

=> Q = a + f = 180 90 = 90 . . . (iv)

=> P + Q = 90 + 90 = 180 . Sufficient

From statement 2:

1
Only from AB = AE, we cannot determine the values of any of the required angles.
2
Insufficient

The correct answer is option A.

84.

19992016 Manhattan Review www.manhattanreview.com


204 Solutions

D
E

B C

From statement 1:

Let BAC = x

=> BCA = 90 x

Since BC = BD, we have: BCD = BDC = 90 x

=> DBC = 180 2 (90 x) = 2x

=> DBA = 90 2x . . . (i)

In triangle AED, AE = AD
180 x x
=> AED = ADE = = 90
2 2
x 3
 
=> EDB = 180 ADEBDC =180 90 (90 x) = x . . . (ii)
2 2
However, there is no information using which the measure of x can be determined.

Insufficient

From statement 2:

Only knowing BE = DE, the value of x cannot be determined. Insufficient

Thus, from both statements together:

Since BE = DE, we have EBD = EDB.

Thus, using (i) and (ii):


3
=> 90 2x = x
2
180
 
=> x = . Sufficient
7
The correct answer is option C.

www.manhattanreview.com 19992016 Manhattan Review


Solutions 205

85.
A

E
D
P

B C

From statement 1:

In triangle EBC, BE = BC and EBC = 90


180 90
=> ECB = CEB = = 45 .
2
90
Since BD bisects ABC, we have: DBC = = 45 .
2
Thus, in triangle BPC:

PBC = PCB = 45 => BPC = 180 (45 + 45 ) = 90

Thus, triangle BPC is an isosceles triangle, right-angled at P



=> BP : PC : BC = 1 : 1 : 2
6
Since BC = 6, we have: BP = PC = = 3 2
2
1 1  2
=> Area of triangle BPC = (BP) (PC) = 3 2 = 9. Sufficient
2 2
From statement 2:

We have seen that the length of AB is neither necessary nor sufficient to determine the

area of triangle BPC. Insufficient

The correct answer is option A.

86. Let us join ZP, ZQ, VP, PQ and VQ, as shown below:
Y Z

X
P Q

19992016 Manhattan Review www.manhattanreview.com


206 Solutions

We have:

Since the circles pass through the centers of each other, they must have the same radius;

PQ being the common radius.

PZ = PV = PQ = ZQ = VQ = radius of the circle

=> ZPQ = VPQ = 60

=> ZPV = 120

For the circle with center P, we have: ZPV = 2 ZYV

(Since angle subtended by an arc at the center is double the angle subtended by the same

arc at any point on the circumference)


120
=> ZYV = = 60
2
For the circle with center Q, we have: ZPV = ZXV

(Since angles subtended by an arc at any point on the circumference are always equal)

=> ZXV = 120

=> ZXY = 180 120 = 60

Thus, in triangle XYZ, we have ZYX = ZXY = 60

=> Triangle XYZ is an equilateral triangle



3
=> Area of triangle XYZ = (XY)2
4
From statement 1:

The location of the point X is not known. Also, the dimensions of the sides of triangle

XYZ are not known. Insufficient

From statement 2:

Since XY = 2, we have:

3
Area of triangle XYZ = 22 = 3. Sufficient
4
The correct answer is option B.

87. From statement 1:

www.manhattanreview.com 19992016 Manhattan Review


Solutions 207


We know that AP = 2 2.

However, we cannot determine the lengths of AX or AY or the side of the square.

Insufficient

From statement 2:

We have no information about the dimensions of either the rectangle AXPY or the circle.

Insufficient

Thus, from both statements together:


90
Since AP coincides with AC, we have: PAX = PAY = = 45
2
=> Triangle APY is a 45-45-90 triangle

=> AY : YP : AP = 1 : 1 : 2

2 2
Since AP = 2 2, we have: AY = YP = = 2.
2
Let us construct the lines as shown:

A Y
B
P 2
X Q
2

D C

Here, O is the center of the circle.

Let the radius of the circle be r .

Since we have PY = PX = 2:

OQ = PQ = r 2

Thus, triangle PQO is a right-angled isosceles triangle



=> PQ : OQ : OP = 1 : 1 : 2
r 2 1
=> =
r 2

2 2
=> r =
2 1

19992016 Manhattan Review www.manhattanreview.com


208 Solutions

We can see that the side of the square is twice the radius of the circle.

4 2
Thus, the side of the square = 2r = . Sufficient
2 1
The correct answer is option C.

88.
P

T
O

From statement 1:

Let RPO = x => PRO = x (Since in triangle RPO, PO = RO = radius)

=> ROP = 180 2x

Since ROS = 40 , we have: SOQ = 180 (180 2x) 40 = 2x 40


180 SOQ
=> SQO = QSO = = 110 x (Since in triangle SOQ, SO = QO = radius)
2
Thus, in triangle PTQ:

PTQ = 180 (TPQ+TQP) = 180 (RPO+SQO)

= 180 x (110 x) = 70 . Sufficient

From statement 2:

We have: RPO = 55 => PRO = 55 (Since in triangle RPO, PO = RO = radius)

=> ROP = 180 2 55 = 70

Let ROS = y

=> SOQ = 180 70 y = 110 y


180 SOQ y
=> SQO = QSO = = 35 + (Since in triangle SOQ, SO = QO = radius)
2 2
Thus, in triangle PTQ:

PTQ = 180 (TPQ+TQP) = 180 (RPO+SQO)

www.manhattanreview.com 19992016 Manhattan Review


Solutions 209

y y
 

= 180 55 35 + = 90 . Since the measure of y is not known, the answer
2 2
cannot be determined. Insufficient

The correct answer is option A.

89.
A B

C O D

Let us join OB.

Thus, in right-angled triangle OBD:

OB2 = OD2 + BD2


2
CD AB AB

Since O is the center of the circle, OD = OC = = => OB2 = + BD2
2 2 2
From statement 1:

We know that OB = 10.

However, there is no information about BD or the relation between AB and BD. Insuffi-

cient

From statement 2:

We know that AB : BD = 3 : 2

However, there is no information about OB. Insufficient

Thus, from both statements together:

Let AB = 3x and BD = 2x
3x 2
 
=> 10 =2
+ (2x)2
2
25 2
=> x = 100 => x 2 = 16 => x = 4
4
As the side cannot be negative, we have: x = 4

=> AB = 3x = 12. Sufficient

19992016 Manhattan Review www.manhattanreview.com


210 Solutions

The correct answer is option C.

90.
P

A S

Q B R

Let AS = QB = x and BS = AQ = y.

Since triangles PAS and SBR are similar, we have:


PA AS
=
SB BR
PQ AQ AS
=> =
SB QR QB
PQ y x
=> = . . . (i)
y QR x
We need to determine the area of SABQ as a fraction of the area of triangle PQR, which

is:
AS SB
1
PQ QR
2
xy
= . . . (ii)
1
PQ QR
2
From statement 1:

We know that: PQ = 24 and QR = 7

Thus, from (i):


24 y x
= => 168 24x 7y + xy = xy
y 7 x
=> 24x + 7y = 168

Since the values of x and y cannot be determined, the required ratio cannot be deter-

mined either. Insufficient

From statement 2:

We know that: SAQB is a square

www.manhattanreview.com 19992016 Manhattan Review


Solutions 211

=> AS = BS => x = y

Thus, from (i):


PQ x x
=
x QR x
=> PQ QR x QR x PQ + x 2 = x 2

=> x (PQ + QR) = PQ QR


PQ QR
=> x =
PQ + QR
Thus, from (ii), the required ratio
PQ QR 2
 
x2 PQ + QR
= =
1 1
PQ QR PQ QR
2 2
PQ QR
= . . . (iii)
2(PQ + QR)2
Since the lengths of PQ and QR are not known, the answer cannot be determined.

Thus, from both statements together:

From (iii), we have:

The required ratio


PQ QR 24 7
= 2
=
2(PQ + QR) 2(24 + 7)2
84 84
= 2
= Sufficient
31 961
The correct answer is option C.

91.
R

P
Q

The diagram for the above situation is shown:

Let the radius of the sphere be R => OQ = OR = R

19992016 Manhattan Review www.manhattanreview.com


212 Solutions

Let the radius of the cone be r => PQ = r

Let the height of the come be h => PR = h

From Pythagoras theorem in triangle OPQ:

OP2 + PQ2 = OQ2 => (h R)2 + r 2 = R 2 . . . (i)

Thus, to calculate h, we need information about both R and r .

From statement 1:

We have R = 6, however, the value of r is not known. Insufficient

From statement 2:

We have r = 4, however, the value of R is not known. Insufficient

Thus, from both statements together:

From (i), we have:

(h 6)2 + 42 = 62 => (h 6)2 = 20



=> h 6 = 20 = 2 5 (since radius cannot be negative).

=> h = 6 + 2 5 Sufficient

The correct answer is option C.

92. We know that the thread is uniformly strung around a cylinder so as to form one com-

plete loop.

Let us try to open the cylinder. If we cut the cylinder open along the line AB, we have the

following diagram:
A A

B B

Note that A and A coincide, and similarly, B and B coincide in the original cylinder.

The line AB is the thread, which, when the cylinder is opened, appears as a straight line.

www.manhattanreview.com 19992016 Manhattan Review


Solutions 213

The length of AA is equal to the length of the circumference of the base circle of the

cylinder.

If the cylinder has radius r , then we have: AA = 2 r .

Also, AB is the height of the cylinder, say h.


q 
2 2
Thus, from Pythagoras theorem, the length of the thread AB = (2 r ) + h .

From statement 1:
2
We have r = , however, the value of h is not known. Insufficient

From statement 2:

We have h = 3, however, the value of r is not known. Insufficient

Thus, from both statements together:

The length of the thread


s
2 2
q 
2 2
= (2 r ) + h = 2 + 32


= 42 + 32 = 5. Sufficient

The correct answer is option C.

93. From statement 1:

There is no information about the cube. Insufficient

From statement 2:

Let each side of the cube be s.

Also, let the radius and height of the cylinder be r and h, respectively.

Since the volumes of cube and cylinder are equal, we have: s 3 = r 2 h


 3
3
=> s 3 = r 2h
2
 
3 3
=> s = r 2h
2
Lateral surface area of the cube = Area of its 4 faces
  2
3 3
= 4s 2 = 4 r 2h
2

19992016 Manhattan Review www.manhattanreview.com


214 Solutions

 2 q
3 3 2
=4 r 2h
2

3
= 9 r 4 h2 . . . (i)

Lateral surface area of the cylinder


 3
3 27
= 2 r h = 2 rh = rh
2 4
3
 
=9 r h . . . (ii)
4

Comparing (i) and (ii):

Let the lateral surface area of the cube be greater than that of the cylinder:

3 3
=> r 4 h2 > rh
4

Cubing both sides:

27 3 3
=> r 4 h2 > r h
64
27
=> r > h
64
27
Thus, the cube has a greater lateral surface area if r > h; the cube has a smaller lateral
64
27
surface area if r < h . . . (iii)
64

However, such a condition is not mentioned. Insufficient

Thus, from both statements together:

From the first statement, we know that:

h = 2r

1 32
=> r = h= h
2 64
27
=> r > h
64

Thus, from (iii): The cube has a greater lateral surface area than that of the cylinder.

Thus, the answer to the question is Yes. Sufficient

The correct answer is option C.

94. The length of the longest line in a cube and in a cylinder is shown in the diagram below:

www.manhattanreview.com 19992016 Manhattan Review


Solutions 215

R
A

C
P Q
B

In the cube, let each side be s.


p
r  2
Thus, the length of the longest line = AC = AB2 + BC2 = s 2 + s 2 = s 3 . . . (i)

In the cylinder, let the radius and height be r and h, respectively.


q q
Thus, the length of the longest line = PR = PQ2 + QR2 = (2r )2 + h2 = 4r 2 + h2 . . . (ii)

We need to compare AC and PR.

From statement 1:
h
s= = 2r
2
s
=> h = 2s and r =
2
Thus, from (ii):
s  
s 2
PR = 4r 2 + h2 = 4 + (2s)2 = s 2 + 4s 2 = s 5
2

Since s 5 > s 3, we have: PR > AC.

Thus, the length of the longest line in the cylinder is greater than that in the cube.

The answer to the question is No. Sufficient

From statement 2:

We only know that the sum of the lengths of the radius of cylinder, the height of the

cylinder and each side of the cube is 14.

Since the individual lengths are not known, the answer cannot be determined. Insuffi-

cient

The correct answer is option A.

95. The cylinder would fit inside the cube only if the base circles of the cylinder have diame-

ters less than the side of the cube AND the height of the cylinder is less than the side of

19992016 Manhattan Review www.manhattanreview.com


216 Solutions

the cube.

In the cube, let each side be s.

In the cylinder, let the radius and height be r and h, respectively.

Volume of the cube = s 3 .

Volume of the cylinder = r 2 h.


1 3
Since the volume of a cylinder equals half the volume of a cube, we have: r 2 h = s
2
=> s 3 = 2 r 2 h . . . (i)

From statement 1:

We know that the ratio of height to radius of the cylinder is 2 : 1


h
=> = 2 => h = 2r
r
Using the above in (i):

s 3 = 2 r 2 (2r )

=> s 3 = 4 r 3

3

3

3

3
=> s = r 4 = r 4 3.14 = r 12.56 > r 8

=> s > 2r

Thus, the side of the cube is greater than the diameter of the cylinder.

Since h = 2r , the side of the cube is also greater than the height of the cylinder.

Thus, the cylinder would fit inside the cube.

The answer to the question is Yes. Sufficient

From statement 2:

We know that each side of the cube is less than twice the height of the cylinder, and also

that the volume of a cylinder equals half the volume of a cube.

Let us take two cases:

Let h = 2, s < 2h => s = 3:


s s s
1 27 3 3 3 3 3
Thus: r 2 2 = 33 => r 2 = => r = < (Since = < 1)
2 4 2 2 3.14

www.manhattanreview.com 19992016 Manhattan Review


Solutions 217

=> Diameter < 3

Thus, the diameter as well as the height of the cylinder is less than the side of the

cube. Thus, the cylinder fits inside the cube.

Let h = 2, s < 2h => s = 1: This could also be a possible case. Since the height of

the cylinder is greater than the side of the cube, the cylinder does not fit inside the

cube.

Thus, there is no unique answer. Insufficient

The correct answer is option A.

96.
A B

D
X

The above figure can be broken in two triangles as shown below:

The required area

= Area of triangle ABD + Area of triangle DBC

= Area of rectangle ABXD Area of triangle DCX


1
= (AD)(AB) (DX)(CX)
2
1
= (AD)(AB) (AB)(CX) . . . (i)
2
From statement 1:

There is no information about CX. Insufficient

From statement 2:

We know that ADC = 60

=> CDX = 90 60 = 30

Thus, triangle DXC is a 30-90-60 triangle



=> DX : CX : CD = 3 : 1 : 2 . . . (ii)

19992016 Manhattan Review www.manhattanreview.com


218 Solutions

However, none of the sides are known.

Thus, there is no information about AD, AB or CX. Insufficient

Thus, from both statements together:


6
From (ii), using DX = AB = 6, we have: CX = = 2 3
3
Thus, from (i):
1
Required area = (AD)(AB)
(AB)(CX)
2
1 
= (6) (6) (6) 2 3 = 36 6 3. Sufficient
2
The correct answer is option C.

97. Let each side of the hexagons be x.

From statement 1:

The shortest distance between A and B is the line AB as shown in the diagram below:
R

B
O

P
A Q

Thus, we have:

AB = AP + PB

= AP + PO + OP

= AP + PQ + RB

(Since a regular hexagon can be broken in 6 equal equilateral triangles, triangles OPQ and

ORB are equilateral, thus, OP = PQ and OB = RB) = 3x

=> 3x = 6 => x = 2. Sufficient

From statement 2:

The shortest straight-line distance between A and B such that the line does not enter

inside either hexagon is shown in the diagram below:

www.manhattanreview.com 19992016 Manhattan Review


Solutions 219

P
S
A X Q T

While going from A to B, we break the route as follows:

AQ (AQ < AP + PQ)

QS (QS < QT + TS)

SB

In triangle APX: APQ = 120 (internal angle of a regular hexagon is 120 )


120
=> APX = = 60
2

3 3
=> AX = AP = x
2 2

=> AQ = 2 AX = 3x

Thus, the length of the route = AQ + QS + SB



= 3x + 2x = x 2 + 3

=> x 2 + 3 =4+2 3

=> x = 2. Sufficient

The correct answer is option D.

98. From statement 1:

Let us join AC. Also, let us extend AB and DC which meet at X, as shown in the diagram

below:
A B X

H C

G D

F E

19992016 Manhattan Review www.manhattanreview.com


220 Solutions

(8 2) 180
In a regular octagon, each interior angle = = 135
8
=> ABC = 135 => CBX = 45

Similarly, BCX = 45

Thus, triangle BCX is a 45-45-90 triangle


BC AB 2
=> BX = CX = = = = 2
2 2 2
Thus, area of triangle ABC

= Area of triangle ACX Area of triangle BCX


1 1
= (AX)(CX) (BX)(CX)
2 2
1    1    
= 2+ 2 2 2 2
2 2
1 
= 2 2 + 2 2 = 2. Sufficient
2
From statement 2:

Let each side of the regular octagon be x.

Let us join AD and drop perpendiculars BX and CY on AD, as shown in the diagram

below:
A B

H X C

Y
G D

F E

We know that ABC = 135 (as discussed above)

Since XBC = YCB = 90 , we have:

ABX = DCY = 45

Thus, triangles ABX and CDY are 45-45-90 triangles


AB x
=> AX = =
2 2
x
Similarly, DY =
2

www.manhattanreview.com 19992016 Manhattan Review


Solutions 221

Also, XY = BC = x
x x  
=> AD = AX + XY + YD = + x + = x 2+1 .
2 2
Thus, we have:
   
x 2+1 =2 2+1

=> x = 2

This is the same information as mentioned in the first statement. Sufficient

The correct answer is option D.

99.
A B

D
C

From statement 1:

Let the areas of triangles BOC and AOD be x and y, respectively.

Since triangles AOB and BOC share the same vertex, they have the same height. Thus, we

have:
Area AOB AO 16
= = . . . (i)
Area BOC CO x
Similarly, we have:
Area AOD AO y
= = . . . (ii)
Area COD CO 36
Thus, from (i) and (ii):
16 y
=> =
x 36
=> xy = 16 36 = 242 . . . (iii)

We need to minimize the sum x + y .

The sum is minimized only when we have x = y in (iii):

=> x = y = 24

19992016 Manhattan Review www.manhattanreview.com


222 Solutions

=> x + y = 48. Sufficient

Note: For a given product of two quantities, the sum of the two quantities is minimized

when they are equal.

For example: If p + q = 16, possible values of p and q are:

p = 1, q = 16 => p + q = 17

p = 2, q = 8 => p + q = 10

p = 4, q = 4 => p + q = 8 (minimum value)

From statement 2:

We cannot determine the sum of areas of triangles AOD and BOC only knowing that their

areas are equal. Insufficient

The correct answer is option A.

100. From statement 1:

There is no information about the dimensions of the stone slabs. Insufficient

From statement 2:

There is no information about the dimensions of the lawn. Insufficient

Thus, from both statements together:

We have the following diagram:

62 6 56
6 6
6 50 50
56
50 Lawn 50 50 Lawn
56
6
6 6
62 56 6

It can be seen that if the pathway is designed as shown in the diagram on the left, the

stone slabs cannot be used to make the pathway since in the figure, the pathway consists

www.manhattanreview.com 19992016 Manhattan Review


Solutions 223

of two rectangles of 6 by 50 and two rectangles of 6 by 62, none of the sides being a

multiple of 4, the length of a side of the stone slabs.

Similarly, if the pathway is designed as shown in the diagram on the right, the pathway

consists of four rectangles of 6 by 56, and 56 is a multiple of 4. However, 6 is not a

multiple of 4. Thus, the stone slabs cannot be used to make the pathway.

Thus, the answer to the question is No. Sufficient

The correct answer is option C.

101.

X
A B

K L
C D
Y

From statement 1:

Let LXB = AXK = x . . . (i)

=> YXL = (x 6 ) . . . (ii)

There is no information using which the value of x can be determined.

Hence, the measure of KXY cannot be determined. Insufficient

From statement 2:

We only know that XY is perpendicular to AB, i.e. AXY = 90 . . . (iii)

The measure of KXY cannot be determined. Insufficient

Thus, from both statements together:

Since AXY = BXY = 90 and LXB = AXK, we have:

AXY AXK = BXY LXB

19992016 Manhattan Review www.manhattanreview.com


224 Solutions

=> KXY = YXL

Also, from (i), (ii) and (iii):

x + (x 6 ) = 90

=> x = 48

=> KXY = YXL = x 6 = 42 . Sufficient

The correct answer is option C.

102.

X
A B

K L
C D
Y

From statement 1:

We know that: YLM = 3 LXB

Let LXB = x

=> YLM = 3x

Also, XLY = LXB = x (alternate angles)

YLM + XLY = 180

=> 3x + x = 180

=> x = 45

=> AXL = YLM (corresponding angles) = 3x = 135 . Sufficient

From statement 2:

We only know that BXL = AXK.

This alone cannot help us determine the measure of AXL. Insufficient

The correct answer is option A.

www.manhattanreview.com 19992016 Manhattan Review


Solutions 225

103.
A X B

D E

Let BCD = x.

From statement 1:

The measure of BCD cannot be determined only knowing that AB and DE are parallel.

Insufficient

From statement 2:

DC is extended to intersect AB at X, as shown:

Thus, in triangle BCX:

BCX = 180 x and CBX = 55 .. (CBX is the same as ABC)

=> BXC = 180 (180 x + 55 ) = x 55

There is no information using which the value of x can be determined. Insufficient

Thus, from both statements together:

Since AB and DE are parallel to each other, BXC = CDE = 90

=> x 55 = 90

=> x = 145 . Sufficient

The correct answer is option C.

104. Let the lines BD and CE, when extended meet at X, as shown below (the points D and E

have not been shown):


A

60#


X
B C

19992016 Manhattan Review www.manhattanreview.com


226 Solutions

We need to determine BXC.

From statement 1:

Only knowing that x = y, we cannot determine BXC. Insufficient

From statement 2:
A

60#


X
B C
P Q

Through X, let us draw two lines XP parallel to AB and XQ parallel to AC, as shown:

Thus, we have:

BXP = ABX = x (alternate angles)

CXQ = ACX = y (alternate angles)

PXQ = BAC = 60 (angles bounded by parallel lines)

=> BXC = x + y + 60 = 40 + 60 = 100 . Sufficient

The correct answer is option B.

Alternate Approach:

Let the lines BD and CE, when extended meet at X.

Thus, we have a quadrilateral ABXC

=> BAC + ABX + ACX + = 360

=> 60 + x + y + = 360

=> = 360 60 x + y = 360 60 40 = 260




www.manhattanreview.com 19992016 Manhattan Review


Solutions 227

Here, is the reflex angle.

Thus, the required non-reflex angle = BXC = 360 260 = 100 .

105.
A B

C D

F
E

From statement 1:

ABC = BCD (alternate angles, since AB is parallel to CD)

BCD = CDF (alternate angles since CB is parallel to DF)

=> CDF = ABC = 45

However, EDF cannot be determined. Insufficient

From statement 2:

EDF = 40

However, CDF cannot be determined. Insufficient

Thus, from both statements together:

CDE = CDF + EDF = 45 + 40 = 85 . Sufficient

The correct answer is option C.

106. From statement 1:



y 3x


Since y 0, we have:

3x 0

=> x < 0

However, the value of x cannot be determined. Insufficient

19992016 Manhattan Review www.manhattanreview.com


228 Solutions

From statement 2:

|5x 1| = x + 7

=> 5x 1 = (x + 7)

=> 5x 1 = x + 7

=> 4x = 8

=> x = 2

OR

5x 1 = x 7

=> 6x = 6

=> x = 1

Thus, the value of x cannot be uniquely determined. Insufficient

Thus, from both statements together:

Since x < 0, the only possible value of x = 1. Sufficient

The correct answer is option C.

107. From statement 1:

Let us look at two possible situations:

x 0:

|x| = x

=> x |x| = x 2

Thus: x 2 9

=> 0 x 3 . . . (i)

x < 0:

|x| = x

=> x |x| = x 2

Thus: x 2 9

www.manhattanreview.com 19992016 Manhattan Review


Solutions 229

=> x 2 9, which is true for all real values of x

=> < x < 0 . . . (ii)

Thus, the value of x cannot be uniquely determined. Insufficient

From statement 2:

x 2|x| = 729

Let us look at two possible situations:

x 0:

|x| = x

=> x 2|x| = x 2x

Thus: x 2x = 729 = 36

=> (x x )2 = 36

=> x x = 33

=> x = 3 (The only possible equation is x x = 33 since x 0)

x < 0:

|x| = x

=> x 2|x| = x 2x

Thus: x 2x = 729 = 36
2
=> (x x ) = 36

=> x x = 33

=> x = 3 (The only possible equation is x x = 33 since x < 0)

Thus, the value of x cannot be uniquely determined. Insufficient

Thus, from both statements together:

Even after combining both statements, we have x = 3 (from (i)) and x = 3 (from (ii)).

Thus, the value of x cannot be uniquely determined. Insufficient

The correct answer is option E.

19992016 Manhattan Review www.manhattanreview.com


230 Solutions

108. x+y +z <1



=> z < 1 x + y . . . (i)

From statement 1:

We know that x and y are positive numbers.

Thus:

1 1 1 1 4
 
If x = and y = => z < 1 + => z <
10 10 10 10 5
Thus, z may or may not be less than 1: The answer is No

If x = 1 and y = 1 => z < 1 (1 + 1) => z < 1: The answer is Yes

Thus, there is no unique answer. Insufficient

From statement 2:

We know that xy = 1.

Thus:

1 1 7
 
If x = and y = 2 => z < 1 2 => z < .
2 2 2
Thus, z may or may not be less than 1: The answer is No

If x = 1 and y = 1 => z < 1 (1 + 1) => z < 1: The answer is Yes

Thus, there is no unique answer. Insufficient

Thus, from both statements together:

x and y are positive numbers and xy = 1.

Thus, trying with a few values:

1 1 5
 
If x = and y = 2 => z < 1 + 2 => z < : The answer is Yes
2 2 2
If x = 1 and y = 1 => z < 1 (1 + 1) => z < 1: The answer is Yes

Thus, we see that z is always less than 1. Sufficient

The correct answer is option C.

Alternate Approach:

We know that, for any two positive numbers a and b:

www.manhattanreview.com 19992016 Manhattan Review


Solutions 231

(a b)2 0

=> a2 + b2 2ab 0


=> a2 + b2 2ab
a2 + b 2
=> ab
2

Replacing a = x and b = y:
x+y
xy
2
x+y
=> 1 (since xy = 1)
2
=> x + y 2

=> x + y 2

=> 1 x + y 1

Thus, from (i): z < 1 x + y

=> z < 1.

Thus, the answer to the question is Yes. Sufficient

109. From statement 1:



|x| > y

=> x > y . . . (i): The answer is Yes

OR

x < y . . . (ii): The answer is No

Thus, the answer cannot be uniquely determined. Insufficient

From statement 2:

x+y >0

=> x > y . . . (iii)

Thus:

If y < 0: y > 0 => x > y > 0

Thus, x is positive, hence is greater than y, which is negative: The answer is Yes

19992016 Manhattan Review www.manhattanreview.com


232 Solutions

If y > 0: y < 0

Thus, x is greater than the negative number y .

However, we cannot determine whether x is also greater than the positive number

y: The answer may be Yes or No.

Thus, we cannot uniquely determine whether x > y. Insufficient

Thus, from both statements together:

Conditions (ii) and (iii) contradict each other.

Thus, (ii) is not a possible scenario since we know that (iii) is true.

Thus, we have:

x > y . . . (i)

AND

x > y . . . (iii)

The above two conditions are possible simultaneously, for example, if

x = 2 and y = 1

x = 2 and y = 1

Thus, x > y.

The answer to the question is Yes. Sufficient

The correct answer is option C.

110. From statement 1:

|x| y < 9

There is no information about y. Insufficient

From statement 2:

There is no information about x. Insufficient

Thus, from both statements together:



y 1

www.manhattanreview.com 19992016 Manhattan Review


Solutions 233

=> 1 y 1

Since y is an integer, we have:

y = 1, 0, 1

Also: |x| y < 9

Thus, if:

y = 1 => |x| (1) < 9 => |x| > 9

x > 9 OR x < 9

y = 0 => |x| (0) < 9 => 0 < 9, which is not possible

y = 1 => |x| 1 < 9 => |x| < 9, which is not possible

Thus, we have: x > 9 OR x < 9

Thus, the answer to the question may be Yes or No. Insufficient

The correct answer is option E.

111. From statement 1:

There is no information about line l. Insufficient

From statement 2:

There is no information about line m. Insufficient

Thus, from both statements together:

Since lines m and n are parallel, they have the same slope, say p.

Thus, we have:
1 1
p+p < => p <
2 4
Let the slope of line l be q.

Since the lines m and l are perpendicular to each other, the product of their slopes is

1.

Thus, we have:
1
p q = 1 => q =
p

19992016 Manhattan Review www.manhattanreview.com


234 Solutions

1
We know that: p <
4
1
If p is positive, i.e. 0 < p < :
4
1 1
4< < => < < 4
p p
=> < q < 4
1
Thus, the slope of line l cannot be . The answer is No.
5
If p is negative, i.e. < p < 0:
1 1
is also negative => is positive.
p p
1
Thus, the slope of line l cannot be . The answer is No.
5

Thus, the answer to the question is No in both cases. Sufficient

The correct answer is option C.

112.
Y
A B
P

O Q X

From statement 1:

We know that the coordinates of P and Q are (0, 6) and (2, 0), respectively.

60
Thus, slope of PQ = = 3
02
=> Slope of AC = 3.

However, the length of AC cannot be determined. Insufficient

From statement 2:

The slope of AC is not known. Insufficient

Thus, from both statements together:



Let the coordinates of A and C be p, q and (r , s), respectively.

www.manhattanreview.com 19992016 Manhattan Review


Solutions 235

Points A and B have the same Y-coordinate value. Also, points B and C have the same

X-coordinate value.


Thus, the coordinates of B are r , q .

Thus we have A (p, q), B (r , q), and C (r , s).

 
Thus, length of AB = r p and length of BC = q s .

qs qs
Slope of AC = = 3 => =3
pr r p

Length of BC
=> =3
Length of AB

Since the length of AB is given as 2, we have:

Length of BC
=> =3
2

=> Length of BC = 6.

Thus, in triangle ABC, from Pythagoras theorem, we have:

p
AC = AB2 + BC2 = 22 + 62


= 40 = 2 10. Sufficient

The correct answer is option C.

113. From statement 1:

The possible positions of the lines m and n are shown in the diagram below:

19992016 Manhattan Review www.manhattanreview.com


236 Solutions

Y Y



O X O X

CASE I: CASE II:


Since the lines and are perpendicular, Since the lines and are perpendicular,
the product of their slopes is 1. the product of their slopes is 1.
Since the Y-intercept of line is 0, product Since the Y-intercept of line is 0, product
of their Y-intercepts is 0. of their Y-intercepts is 0.

Thus, product of the slopes is less then the Thus, product of the slopes is less then the
product of the Y-intercepts. Answer is Yes. product of the Y-intercepts. Answer is Yes.

Y CASE III:
Line has undefined slope while line has
a 0 slope. Thus, product of their slopes is
undefined.
Since the Y-intercept of line is 0, product
of their Y-intercepts is 0.
O X
Thus, product of the slopes and the product
of the Y-intercepts cannot be compared.
Answer is No.

Thus, the answer cannot be uniquely determined. Insufficient

From statement 2:

We know that none of the lines m and n are parallel to either axis.

Since the lines m and n are perpendicular, the product of their slopes is 1.

However, we may have different scenarios for the Y-intercepts:

Both lines have positive Y-intercepts:

Product of their Y-intercepts is positive, and hence is greater than the product of

their slopes. Answer is Yes.

Both lines have negative Y-intercepts:

Product of their Y-intercepts is positive, and hence is greater than the product of

their slopes. Answer is Yes.

www.manhattanreview.com 19992016 Manhattan Review


Solutions 237

One line has positive Y-intercept, while the other has negative Y-intercept:

Product of their Y-intercepts is negative, which may be greater than or less than 1

i.e. the product of their slopes. Answer maybe Yes or No.

Thus, the answer cannot be uniquely determined. Insufficient

Thus, from both statements together:

Combining both statements, we see that only Case I and Case II, as discussed under

statement 1 are possible, and the answer is unique.

Thus, the answer to the question is Yes. Sufficient

The correct answer is option C.

114. From statement 1:

Since the points A and B are equidistant from the origin (0, 0), we have:
q 2 2 q
p 0 + q 0 = (r 0)2 + (s 0)2

=> p 2 + q2 = r 2 + s 2 . . . (i)

However, the values of q and s cannot be compared. Insufficient

From statement 2:

We know that: p > |r |
2
=> p > |r |2

=> p 2 > r 2 . . . (ii)

However, the values of q and s cannot be compared. Insufficient

Thus, from both statements together:

From (i): p 2 r 2 = s 2 q2

Also, from (ii): p 2 r 2 > 0

Thus, we have: s 2 q2 > 0

=> s 2 > q2
2
=> |s|2 > q

19992016 Manhattan Review www.manhattanreview.com


238 Solutions


=> |s| > q .

Thus, the answer to the question is No. Sufficient

The correct answer is option C.

115. From statement 1:

There is no information about point E. Insufficient

From statement 2:

There is no information about the points A, B, C and D. Insufficient

Thus, from both statements together:

The points A, B, C, D and one possible position of point E are shown in the diagram

below:
Y
=6
A 0, 5
E 6,

O B 6, 0
D 2, 0 X

C 0,3

Triangles AEC and ABC lie on the same base AC and also have the same height 6

=> Area of triangle ABC = Area of triangle AEC

Thus, we have:

Area of ABCD = Area of triangle ABC + Area of triangle ACD

= Area of triangle AEC + Area of triangle ACD

= Area of AECD.

The answer to the question is Yes. Sufficient

The correct answer is option C.

116. From statement 1:

www.manhattanreview.com 19992016 Manhattan Review


Solutions 239

Since the two-digit numbers PQ and QP add to form a three-digit number RSR, the value

of the digit in the hundreds place must be 1

=> R = 1

Thus, P and Q should be such digits which add up to give 1 as unit digit (so that the unit

digit of R = 1 is obtained in the sum).

Thus, possible values of P and Q are:

P = 2, Q = 9 => PQ + QP = 29 + 92 = 121 => P + Q + R + S = 2 + 9 + 1 + 2 = 14

P = 3, Q = 8 => PQ + QP = 38 + 83 = 121 => P + Q + R + S = 3 + 8 + 1 + 2 = 14

P = 4, Q = 7 => PQ + QP = 47 + 74 = 121 => P + Q + R + S = 4 + 7 + 1 + 2 = 14

P = 5, Q = 6 => PQ + QP = 56 + 65 = 121 => P + Q + R + S = 5 + 6 + 1 + 2 = 14

Thus, the unique answer is 14. Sufficient

From statement 2:

We know that P, Q, R and S are distinct non-zero digits.

However, their values cannot be determined. Insufficient

The correct answer is option A.

117. From statement 1:

In the multiplication, the unit digit is 5.

Thus, the product P P has 5 in the unit place => P = 5

Thus, the number PQP 5Q5

=> PQP P 5Q5 5

Thus, 5Q5 represents a number between 505 and 595; we can ballpark between 500 and

600.

Since 500 5 = 2500 and 600 5 = 3000, we have:

2500 < RQS5 < 3000

Thus, we have: R = 2. Sufficient

19992016 Manhattan Review www.manhattanreview.com


240 Solutions

From statement 2:

We know that P, Q, R and S are distinct non-zero digits.

However, their values cannot be determined. Insufficient

The correct answer is option A.

118. From statement 1:

D.D + DD = D.D DD

=> (D 1.1) + (10D + D) = (D 1.1) (10D + D)

=> D (1.1 + 11) = D2 (1.1 11)

=> D 12.1 =D2 12.1

=> D =D2

Since it is known that D 6= 0, we have:

D = 1. Sufficient

From statement 2:

The information that D is not a prime number cannot alone be used to determine the

value of D. Insufficient

The correct answer is option A.

119. In the multiplication, the product of the unit digits of the numbers results in 3, i.e. A

3 results in the unit digit 3.

Thus, we have: A = 1.

Thus, we have: 1B1 3 = BC3

If, in the tens digit multiplication, i.e. B 3, there is no carry, then, in the hundreds

digit multiplication, i.e. 1 3, we have B = 3.

Thus: 131 3 = 393 Satisfies.

If, in the tens digit multiplication, i.e. B 3, there is a carry 1, then, in the hundreds

digit multiplication, i.e. 1 3 + 1 (carry), we have B = 4.

www.manhattanreview.com 19992016 Manhattan Review


Solutions 241

Thus: 141 3 = 423 Satisfies

If, in the tens digit multiplication, i.e. B 3, there is a carry 2, then, in the hundreds

digit multiplication, i.e. 1 3 + 2 (carry), we have B = 5.

However, for B 3 to have a carry 2, B must be at least 7.

Thus, such a situation is not possible.

Note: In the tens digit multiplication, i.e. B 3, the maximum carry can only be 2.

Thus, the value of B can be either 3 or 4. Insufficient

From statement 2:

The information that A, B, C and 3 are distinct non-zero digits cannot alone be used to

determine the value of B. Insufficient

Thus, from both statements together:

We have the following situations:

131 3 = 393 => A = 1, B = 3, C = 9 Not valid, since it is known that B 6= 3.

141 3 = 423 => A = 1, B = 4, C = 2 Valid, since A, B, C and 3 are all distinct.

Thus, we have: B = 4. Sufficient

The correct answer is option C.

120. From statement 1:

In the unit digit multiplication, i.e. A A, the unit digit of the product is 6.

Thus, we have: A = 4 OR 6:

A = 4: 4444 4 = 17776 => C = 7, B = 1 => C B = 6

A = 6: 6666 6 = 39996 => C = 9, B = 3 => C B = 6

Thus, the unique answer is 6. Sufficient

From statement 2:

The information that A < 6 cannot alone be used to determine the value of (C B).

Insufficient

19992016 Manhattan Review www.manhattanreview.com


242 Solutions

The correct answer is option A.

121. From statement 1:

We know that student received an equal number of pens and an equal number of pencils.

Thus, the GCD of the number of pens and the number of pencils is the largest possible

number of students in the class.

180 = 22 32 5

40 = 23 5

Thus, the terms common to both = 22 5 = 20.

=> GCD of 40 and 180 = 20

Thus, the number of students could be 20 or a factor of 20:

Number of students = 20:


180
Number of pens received by each student = =9
20
40
Number of pencils received by each student = =2
20
Number of students = 10:
180
Number of pens received by each student = = 18
10
40
Number of pencils received by each student = =4
10
Number of students = 5:
180
Number of pens received by each student = = 36
5
40
Number of pencils received by each student = =8
5
There are three more possible cases for the number of students, i.e. 10 OR 4 OR 2

Thus, there is no unique answer. Insufficient

From statement 2:

There is no information about the number of pens and pencils distributed. Insufficient

Thus, from both statement s together:

Combining both statements, we find that, since the total number of items received by

each student is less than 15, the only possible scenario (scenario #1 mentioned above) is

www.manhattanreview.com 19992016 Manhattan Review


Solutions 243

when the number of students is 20. Total number of items received by each student = 9

+ 2 = 11 < 15. Sufficient

The correct answer is option C.

122. Since 2135, when divided by n, leaves remainder r , we have, assuming the quotient to

be a:

2135 = na + r . . . (i)

From statement 1:

2315 when divided by n, leaves remainder r , and assuming the quotient to be b, we have:

2315 = nb + r . . . (ii)

Subtracting (i) from (ii):

2315 2135 = n (b a)

=> n (b a) = 180 . . . (iv)

Thus, the value of n could be any factor of 180. Insufficient

From statement 2:

2585 when divided by n, leaves remainder r , and assuming the quotient to be c, we have:

2585 = nc + r . . . (iii)

Subtracting (i) from (iii):

2585 2135 = n (c a)

=> n (c a) = 450 . . . (iv)

Thus, the value of n could be any factor of 450. Insufficient

Thus, from both statements together:

The value of n could be any factor of 180 as well as 450, i.e. the common factors of 180

and 450.

180 = 22 32 5

450 = 2 32 52

19992016 Manhattan Review www.manhattanreview.com


244 Solutions

Thus, the terms common to both = 2 32 5 = 90.

=> GCD of 180 and 450 = 90

Thus, the value of n could be any factor of 90, for example: 90, 45, 30, etc. Insufficient

The correct answer is option E.

123. From statement 1:

n when divided by 6, leaves remainder 5, and assuming the quotient to be a , we have:

n = 6a + 5 . . . (i)

=> 2n = 12a + 10

Thus, the remainder when 2n is divided by 8 is the same as the remainder when

(12a + 10) is divided by 8.

12a + 10 = (8a + 8) + (4a + 2)

Since (8a + 8) is a multiple of 8, the remainder when (8a + 8) is divided by 8 is 0.

Thus, the required remainder is the same as the remainder when (4a + 2) is divided by

8:

If a = 1: 4a + 2 = 6 => Required remainder is 6; with n = 11

If a = 2: 4a + 2 = 10 => Required remainder is 2; with n = 17

If a = 3: 4a + 2 = 14 => Required remainder is 6; with n = 23

If a = 4: 4a + 2 = 18 => Required remainder is 2; with n = 27

Thus, the remainder can be either 2 or 6. Insufficient

From statement 2:

Let the remainder when n is divided by 6 be r (where 0 r < 6) and the quotient be b

=> n = 6b + r

=> 3n = 18b + 3r

Thus, the remainder when 3n is divided by 6 is the same as the remainder when 3r is

divided by 6, as 18b is completely divisible by 6.

www.manhattanreview.com 19992016 Manhattan Review


Solutions 245

Thus, 3r , when divided by 6, leaves remainder 3 (given in the question).

Thus, possible values of 3r could be 9, 15, 21, etc.

=> The possible values of r could be 3, 5, 7, etc.

Since 0 r < 6, we have: r = 3 OR 5

=> n = 6b + 3 OR n = 6b + 5

=> 2n = 12b + 6 OR 2n = 12b + 10

We already have n = 6b + 5 as a solution from statement 1, from which the required

remainder is either 6 or 2, i.e. the answer cannot be uniquely determined.

Here, we additionally have n = 6b + 3, which could possibly result in additional values

of the required remainder.

Thus, the answer cannot be uniquely determined. Insufficient

Thus, from both statements together:

After combining both statements, we have, n as 5 greater than a multiple of 6, as a

common scenario.

Thus, the remainder can be either 6 or 2.

Thus, the answer cannot be uniquely determined. Insufficient

The correct answer is option E.

124. Since n, when divided by 12, leaves remainder q (where 0 q < 12), assuming the

quotient to be a, we have:

n = 12a + q . . . (i)

The remainder obtained by dividing n by 6 is the same as the remainder obtained by



dividing 12a + q by 6

Since 12a is divisible by 6, the remainder obtained is simply the same as the remainder

obtained by dividing q by 6.

Thus, q, when divided by 6, leaves a remainder p (where 0 p < 6).

The possible scenarios are:

19992016 Manhattan Review www.manhattanreview.com


246 Solutions

0 q < 6: Since q is less than 6, the remainder when q is divided by 6 would remain

the same, i.e. p = q.

Example, if q = 5, the value of p, i.e. the remainder when 5 is divided by 6, is also 5.

6 q < 12: Since q is greater than 6, the remainder when q is divided by 6 would

become less than 6, i.e. p < q.

Example, if q = 6, the value of p, i.e. the remainder when 6 is divided by 6, is 0.

Note: p is either less than or equal to q; p can NEVER exceed q.

From statement 1:

We know that n is a multiple of 8, i.e. 8, 16, 24, 32, etc.:

n = 8 => q = 8 => p < q The answer is Yes.

n = 16 => q = 4 => p = q The answer is No.

Thus, there is no unique answer. Insufficient

From statement 2:

We know that n is a multiple of 6, i.e. 6, 12, 18, 24, etc.:

n = 6 => q = 6 => p < q The answer is Yes.

n = 12 => q = 0 => p = q The answer is No.

Thus, there is no unique answer. Insufficient

Thus, from both statements together:

Since n is a multiple of 8 and 6, it must be a multiple of the LCM of 8 and 6, i.e. 24.

Thus, possible values of n are: 24, 48, 72, etc.

Since n is always a multiple of 12, q = 0 => p = q. The answer is No.

Thus, the unique answer to the question is No. Sufficient

The correct answer is option C.

125. From statement 1:


n2
We know that is an integer, i.e. n2 is divisible by 144.
144

www.manhattanreview.com 19992016 Manhattan Review


Solutions 247

However, it is not known whether n is an integer.

Thus, we may have:



n= 288 => n2 = 288, which is divisible by 144.

However, n is not divisible by 12. The answer is No.

n = 24 => n2 = 242 , which is divisible by 144.

Also, n is divisible by 12. The answer is Yes.

Thus, there is no unique answer. Insufficient

From statement 2:
n
We know that is an integer, i.e. n is divisible by 6, hence n must be an integer.
6
Thus, we may have:

n = 18, which is divisible by 6.

However, n is not divisible by 12. The answer is No.

n = 24, which is divisible by 6.

Also, n is divisible by 12. The answer is Yes.

Thus, there is no unique answer. Insufficient

Thus, from both statements together:

From the second statement we know that n is an integer.

Thus, from the first statement, we have: n2 is an integer, which is divisible by 144 = 122

=> n is divisible by 12. Sufficient

The correct answer is option C.

126. From statement 1:

Different ways of expressing 64 are:

(2)6 => x = 2, y = 6 => x + y = 4 OR 8

43 => x = 4, y = 3 => x + y = 7

(8)2 => x = 8, y = 2 => x + y = 6 OR 10

19992016 Manhattan Review www.manhattanreview.com


248 Solutions

641 => x = 64, y = 1 => x + y = 65

Thus, there is no unique answer. Insufficient

From statement 2:

Different ways of expressing 36 are:

(6)2 => x = 6, y = 2 => x + y = 4 OR 8

361 => x = 36, y = 1 => x + y = 37

Thus, there is no unique answer. Insufficient

Thus, from both statements together:

The only common solution is: x = 2, y = 6 => x + y = 8. Sufficient

The correct answer is option C.

127. 192 = 26 3
n
Thus, can be an integer only if n is divisible by 26 and also by 3.
192
From statement 1:

n is a multiple of 24, i.e. 23 3 => n is divisible by 23 and also by 3.

n is not a multiple of 16, i.e. 24 => n is not divisible by 24

=> n is not divisible by 192.


n
Thus, cannot be an integer. Sufficient
192
From statement 2:

n is a multiple of 8, i.e. 23 => n is divisible by 23 .

n is not a multiple of 48, i.e. 24 3

Thus, there might be two possibilities:

n is not a multiple of 48 since it is not a multiple 3, hence, it is definitely not a

multiple of 192.

The highest exponent of 2 by which n is divisible is 3, i.e. n is divisible by 23 , but

not by 24 , hence, it is definitely not a multiple of 192.

www.manhattanreview.com 19992016 Manhattan Review


Solutions 249

n
Thus, cannot be an integer. Sufficient
192
The correct answer is option D.

128. From statement 1:

Since 25a7b is divisible by 36, it is divisible by both 4 and 9.

Since the number is divisible by 4, the number formed by the last two digits of 25a7b,

i.e. 7b must be divisible by 4.

Thus, the possible values of b = 2 OR 6 (Since 72 and 76 are divisible by 4)

Since 25a7b is divisible by 9, the sum of digits, i.e. 2 + 5 + a + 7 + b = (14 + a + b) must

be divisible by 9:

If b = 2 : 14 + a + b = 16 + a is divisible by 9 => a = 2 => a + b = 4

If b = 6 : 14 + a + b = 20 + a is divisible by 9 => a = 7 => a + b = 13

Thus, there is no unique answer. Insufficient

From statement 2:

Since a1b is divisible by 4, the number formed by the last two digits of a1b, i.e. 1b must

be divisible by 4.

Thus, the possible values of b = 2 OR 6 (Since 12 and 16 are divisible by 4)

However, the value of a cannot be determined. Insufficient

Thus, from both statements together:

Even after combining both statements, we still have:

b = 2, a = 2 => a + b = 4

b = 6, a = 7 => a + b = 13

Thus, there is no unique answer. Insufficient

The correct answer is option E.

129. From statement 1:

19992016 Manhattan Review www.manhattanreview.com


250 Solutions

We know that:

(2n + 2) is divisible by 14

Dividing by 2 throughout:

(n + 1) is divisible by 7

=> n + 1 = 7k, where k is the quotient obtained when (n + 1) is divided by 7 => n =

7k 1 = 7 (k 1) + 6

Thus, the remainder when n is divided by 7 is 6. Sufficient

From statement 2:

We know that:

n2 1 is divisible by 7


=> (n + 1) (n 1) is divisible by 7

=> Either (n + 1) or (n 1) or both are divisible by 7, but (n 1) and (n + 1) have

gap of 2 numbers, thus, both at the same time cannot be multiples of 7 (consecutive

multiples of 7 must have gap of 7)

Let (n + 1) = 7 => n = 6

Thus, the remainder when n is divided by 7 is 6.

Let (n 1) = 7 => n = 8

Thus, the remainder when n is divided by 7 is 1.

Thus, there is no unique answer. Insufficient

The correct answer is option A.

130. From statement 1:

We know that:

(3p + 2) is not a prime number.

Thus, working with different values of p:

p = 1 => 3p + 2 = 5, which is prime

www.manhattanreview.com 19992016 Manhattan Review


Solutions 251

p = 2 => 3p + 2 = 11, which is prime

p = 3 => 3p + 2 = 29, which is prime

p = 4 => 3p + 2 = 83, which is prime

p = 5 => 3p + 2 = 245, which is NOT prime (since it is divisible by 5).

Thus, the least possible value of p is 5.

Thus, the remainder when (3p + 2) = 245 is divided by p = 5, is 0. Sufficient

From statement 2:

We know that (3p + 2) is not a prime number.

Also, (3p + 2) < 500

As seen for the first statement, p = 5 is the first value which satisfies both the above

conditions.

Let us check for p = 6: 3p + 2 = 36 + 2 = 731.

Since 731 is greater than 500, we cannot take p = 6 even if 731 is NOT prime.

Note: We do not need to check whether 731 is a prime number, which, incidentally, it is

not, since 731 = 17 43.

Thus, we have: p = 5, and hence, the required remainder is 0, as obtained from the first

statement. Sufficient

The correct answer is option D.

131. We need to determine whether:

q 1 = kp, where k is an integer

From statement 1:

We know that p > q:

Let p = 2, q = 5 => q 1 = 6

=> q 1 = (3) p The answer is Yes.

19992016 Manhattan Review www.manhattanreview.com


252 Solutions

Let p = 4, q = 3 => q 1 = 2
1
 
=> q 1 = p The answer is No.
2

Thus, there is no unique answer. Insufficient

From statement 2:

There is no information about p. Insufficient

Thus, from both statements together:

We have: p > q > 1

Thus, p and q are positive integers greater than 1.


 
Thus, q 1 must be less than p, and both q 1 and p are positive.
q1
q 1 = kp => =k<1
p

=> q 1 cannot be expressed as an integer multiple of p. Sufficient

The correct answer is option C.

132. From statement 1:

Since 2ab3 is divisible by 9, the sum of digits, i.e. (2 + a + b + 3) = (5 + a + b) is divisi-

ble by 9

=> 5 + a + b = 9k, where k is a positive integer

=> a + b = 9k 5 . . . (i)

However, there is no information about c. Insufficient

From statement 2:

Since 4bc1 is divisible by 9, the sum of digits, i.e. (4 + b + c + 1) = (5 + b + c) is divisible

by 9

=> 5 + b + c = 9l, where l is a positive integer

=> b + c = 9l 5 . . . (ii)

However, there is no information about a. Insufficient

Thus, from both statements together:

www.manhattanreview.com 19992016 Manhattan Review


Solutions 253

Adding (i) and (ii):

a + 2b + c = 9(k + l) 10

=> a + b + c = 9(k + l) 10 b

However, the value of b is not known:

If b = 8: a + b + c = 9(k + l) 10 8 = 9(k + l 2), which is divisible by 9

If b = 1: a + b + c = 9 (k + l) 10 1 = 9 (k + l) 11, which is not divisible by 9

Thus, the answer cannot be uniquely determined. Insufficient

The correct answer is option E.

133. From statement 1:


a
We know that: = 2.1, i.e. a terminating decimal.
x+y

=> The denominator x + y must be expressed only by the product of exponents of 2

and/or 5, for example:


a 21 42 105
= or or , etc.
x+y 10 20 50

Thus, x + y is an even number.

=> Since x is even, y must also be even. Sufficient

From statement 2:

We know that: a x + y = 210

The scenarios possible are:

a = 21, x + y = 10 => Since xis even, y must be even. The answer is Yes

a = 42, x + y = 5 => Since xis even, y must be odd. The answer is No

Thus, there is no unique answer. Insufficient

The correct answer is option A.

134. From statement 1:

We know that: p 11 + 11p , when divided by 10, leaves remainder 4

19992016 Manhattan Review www.manhattanreview.com


254 Solutions

=> The unit digit of p 11 + 11p is 4




The unit digit of 11p The unit digit of 1p = 1

Thus, the unit digit of p 11 = 4 1 = 3

Since the unit digit of p 11 is odd, p must be an odd number.

Thus, possibilities for the unit digit of p are 1, 3, 5, 7 and 9:

Unit digit of p is 1: The unit digit of 111 = 1 Does not satisfy

Unit digit of p is 3: The unit digit of 311 The unit digit of 33 = 7 Does not satisfy

(The unit digit cycle for exponents of 3 is: 3, 9, 7, 1)

Unit digit of p is 5: The unit digit of 511 = 5 Does not satisfy

(The unit digit for exponents of 5 is always 5)

Unit digit of p is 7: The unit digit of 711 The unit digit of 73 = 3 Satisfies

(The unit digit cycle for exponents of 7 is: 7, 9, 3, 1)

Unit digit of p is 9: The unit digit of 911 The unit digit of 91 = 9 Does not satisfy

(The unit digit cycle for exponents of 9 is: 9, 1)

Thus, the unit digit of p is 7.

=> The required remainder is 7. Sufficient

From statement 2:

We know that: p 3 , when divided by 10, leaves remainder 3

=> The unit digit of p 3 is 3

Since the unit digit of p 3 is odd, p must be an odd number.

Thus, possibilities for the unit digit of p are 1, 3, 5, 7 and 9:

Unit digit of p is 1: The unit digit of 13 = 1 Does not satisfy

Unit digit of p is 3: The unit digit of 33 = 7 Does not satisfy

Unit digit of p is 5: The unit digit of 53 = 5 Does not satisfy

Unit digit of p is 7: The unit digit of 73 = 3 Satisfies

www.manhattanreview.com 19992016 Manhattan Review


Solutions 255

Unit digit of p is 9: The unit digit of 93 = 9 Does not satisfy

Thus, the unit digit of p is 7.

=> The required remainder is 7. Sufficient

The correct answer is option D.

135. We know that qs < 130 . . . (i)

From statement 1:

Since p, when divided by q, leaves remainder 9, we have:

q 10 . . . (ii)

However, the value of s cannot be determined. Insufficient

From statement 2:

Since r , when divided by s, leaves remainder 10, we have:

s 11 . . . (iii)

However, the value of q cannot be determined. Insufficient

Thus, from both statements together:

Multiplying (ii) and (iii):

qs 110 . . . (iv)

We are given that qs < 130

Thus, 130 > qs 110 . . . (v)

Thus, the possible scenarios satisfying (i), (ii), (iii), (iv) and (v) are:

19992016 Manhattan Review www.manhattanreview.com


256 Solutions

q s qs q+s

10 11 110 Satisfies 21

11 11 121 Satisfies 22

10 12 120 Satisfies 22


Thus, the value of q + s is either 21 or 22.

Thus, there is no unique answer. Insufficient

The correct answer is option E.

136. From statement 1:

We know that:

x represents the greatest possible value of the exponents of 3, such that 3x can divide

n!

Thus, the value of x, i.e. the greatest possible exponent of 3 can be determined by

dividing n by 3 successively and adding only the integer parts of the quotients obtained.

We also know that:

y represents the greatest possible value of the exponent among all possible prime num-

bers p, so that p y can divide n!

The greatest exponent of any prime would happen for the smallest possible prime.

Thus, the value of y would be the greatest possible if p = 2.

Thus, the value of y, i.e. the greatest possible exponent of p = 2 can be determined by

dividing n by 2 successively and adding only the integer parts of the quotients obtained.

Since the successive quotients obtained when n is divided by 2 is greater than that ob-

tained for 3, we have:

y >x

www.manhattanreview.com 19992016 Manhattan Review


Solutions 257

Note: Only for n = 1, i.e. n! = 1! = 1, we have x = y = 0, and for n = 3, i.e. n! = 3!, we

have x = y = 1.

Thus, the answer is No. Sufficient

From statement 2:

The value of n is neither necessary nor is alone sufficient to answer the question.

Insufficient

The correct answer is option A.

137. From statement 1:



We know that: 4x + 3y is even

Since 4x is even, 3y must be even => y is even

However, x may be either even or odd, i.e. its nature cannot be determined.

Thus, we have:

x 2 + 3x + y 2

= x 2 + x + 2x + y 2

= x (x + 1) + 2x + y 2

Here:

x (x + 1) denotes the product of two consecutive numbers, one of which must be

even and the other odd => x (x + 1) is even

2x is even

y 2 is even (since y is even)

Thus, we have: x 2 + 3x + y 2 is even. Sufficient




From statement 2:

We know that: x y is even

Thus, we may have:

x and y are both even

19992016 Manhattan Review www.manhattanreview.com


258 Solutions

x and y are both odd

Thus, proceeding in the same manner as in the first statement, we have: x 2 + 3x + y 2

= x (x + 1) + 2x + y 2

As before, x (x + 1) and 2x are even, however, the nature of y is not known. Insuffi-

cient

The correct answer is option A.

138. From statement 1:

Let us express n2 + 6n + 16 in terms of (n + 4): n2 + 6n + 16




= n2 + 4n + 2n + 8 + 8

= n (n + 4) + 2 (n + 4) + 8

= (n + 2) (n + 4) + 8

Since n2 + 6n + 16 is divisible by (n + 4):




=> {(n + 2) (n + 4) + 8} is divisible by (n + 4)

=> 8 is divisible by (n + 4)

Thus, (n + 4) must be the factors of 8

=> n + 4 = 8 or 4 or 2 or 1 or 1, etc.

=> n = 4 or 0 or 2 or 3 or 5, etc.

Since n is a positive integer, we only have: n = 4. Sufficient

From statement 2:

Only from n > 2, we cannot determine the value of n. Insufficient

The correct answer is option A.

139. From statement 1:

We only know that there were 4 students in the class.

The starting numbers of the 4 students cannot be determined. Insufficient

www.manhattanreview.com 19992016 Manhattan Review


Solutions 259

From statement 2:

We know that the starting number must not be a prime number nor should it be 1, so the

minimum number to choose would be 4. Also, we know that no number should exceed

25.

Thus, we have, starting with the smallest possible values:

Starting number = 4: The numbers are: 4, 9, 14, 19 Possible

Starting number = 6: The numbers are: 6, 11, 16, 21 Possible

Starting number = 8: The numbers are: 8, 13, 18, 23 Possible

Starting number = 9: The numbers are: 9, 14, 19, 24 Possible

Starting number = 10: The numbers are: 10, 15, 20, 25 Possible

Thus, we find that there are 5 different sets of four numbers possible.

However, since the number of students is not known, the answer cannot be determined.

Insufficient

Thus, from both statements together:

Since there are 4 students, the four sets of 4 numbers can be any of the five sets shown

for the second statement.

Note: Had there been 5 students, we could have definitely commented on the starting

numbers, since there are 5 options for the same.

Thus, the starting numbers chosen by the students cannot be uniquely determined.

Insufficient

The correct answer is option E.

140.

From statement 1:

We have: ab > 0

Thus, the possible scenarios are:

19992016 Manhattan Review www.manhattanreview.com


260 Solutions

a > 0, b > 0:

Since c and d are to the right of a and b: c > 0 and d > 0

Thus, both ab and cd are positive, with a < c and b < d

=> ab < cd. The answer is Yes.

a < 0, b < 0:

Since c and d are to the right of a and b, we may have:

I. c < 0 and d < 0, with c and d closer to 0, implying their absolute values are

smaller than that of a and b.

Since c and d are negative, cd > 0 => ab > cd. The answer is No.

II. c < 0 and d > 0, i.e. 0 lies somewhere in between c and d

=> cd < 0 => ab > cd. The answer is Yes.

III. c > 0 and d > 0, with 0 lying somewhere in between b and c.

Thus, both ab > 0 and cd > 0.

However, the values of ab and cd cannot be compared. The answer maybe

Yes or No.

Thus, there is no unique answer. Insufficient

From statement 2:

We have: bd < 0

Thus, the possible scenarios are:

b > 0, d < 0: This scenario is not possible since from the diagram, d > b.

b < 0, d > 0:

Thus, 0 lies somewhere in between b and d.

Note: Since a < b, we have: a < 0.

Thus, we have:

I. c < 0: ab > 0 and cd < 0 => ab > cd. The answer is Yes.

II. c > 0: Both ab > 0 and cd > 0.

www.manhattanreview.com 19992016 Manhattan Review


Solutions 261

However, the values of ab and cd cannot be compared. The answer maybe Yes

or No.

Thus, there is no unique answer. Insufficient

Thus, from both statements together:

After combining both statements, we find that the scenarios common to both is:

a < 0, b < 0, c < 0 and d > 0: ab > cd The answer is Yes.

a < 0, b < 0, c > 0 and d > 0: ab > 0 and cd > 0 - The answer may be Yes or No.

Thus, there is no unique answer. Insufficient

The correct answer is option E.

141. From statement 1:

Since each plant manufactures the same toy, the quantity of raw material required to

manufacture a toy in each plant is:

Plant Raw material (000 kg) Finished product (units) Raw material per toy (kg)

400, 000
A 400 100,000 =4
100, 000

360, 000
B 360 120,000 =3
120, 000

250, 000
C 250 50,000 =5
50, 000

180, 000
D 180 30,000 =6
30, 000

Thus, we find that Plant D uses the maximum quantity of raw material (6 kg) for each

toy, implying that it has the maximum wastage. Similarly, Plant B uses the minimum

quantity of raw material (3 kg) for each toy, implying that it has the minimum wastage.

Thus, the answer is Yes. Sufficient

From statement 2:

19992016 Manhattan Review www.manhattanreview.com


262 Solutions

We know that the weight of the toys manufactured in the plants A, B, C and D are in the

ratio 2 : 2 : 3 : 3.

Let us assume that the weight of the toys manufactured in the plants A, B, C and D are

2k, 2k, 3k and 3k, respectively.

Thus, we have:

Raw material Finished product Weight/toy Total weight of toys


Plant
(000 kg) (units) (kg) (000 kg)

A 400 100,000 2k 200k

B 360 120,000 2k 240k

C 250 50,000 3k 150k

D 180 30,000 3k 90k

Thus, we see that Plant A receives 400,000 kg raw material but uses 200, 000k kg to

manufacture toys, the rest being wasted

Raw material used 200k


=> Raw material utilization for Plant A = 100 = 100 =
Raw material received 400
50k%

Similarly, for the other plants, we have:

www.manhattanreview.com 19992016 Manhattan Review


Solutions 263

Plant Raw material utilization

200k
A 100 = 50k%
400

240k
B 100 = 66.7k%
360

150k
C 100 = 60k%
250

90k
D 100 = 50k%
180

Thus, we see that the raw material utilization rate is the highest for Plant B, implying

that it has the minimum wastage.

Thus, the answer is Yes. Sufficient

The correct answer is option D.

142. Let the two-digit number be 10x + y, where x and y are the digits.

Thus, x lies between 1 and 9, inclusive and y lies between 0 and 9, inclusive.

We need to determine the value of:


 !
10x + y x + y + 9x x
= =1+9
x+y x+y x+y
From statement 1:

We know that the ratio is the minimum possible.

Thus, to minimize the ratio, we take y = 9 and x = 1 (maximize the denominator and

minimize the numerator).


1
 
Thus, the required ratio = 1 + 9 = 1.9 Sufficient
1+9
The two-digit number would be 19. You may do a hit and trial with other numbers and

would find that the ratios of numbers to sum of their respective digits are greater than

1.9.

From statement 2:

19992016 Manhattan Review www.manhattanreview.com


264 Solutions

We have no information about the value of the tens digit, i.e. the value of x. Insufficient

The correct answer is option A.

143. From statement 1:

We have:

f (k) = k3 + k3 4k = 2k3 4k
 
f (k) = (k)3 + k (k)2 4 (k) = 4k

Since f (k) = f (k), we have:

2k3 4k = 4k

=> k3 = 0 => k = 0. Sufficient

From statement 2:

f (2) = 23 + k 22 4 2 = 4k

=> f (f (2)) = f (4k) = (4k)3 + k (4k)2 4 (4k) = 80k3 16k

Since f (f (2)) = 0, we have:

80k3 16k = 0 => 16k 5k2 1 = 0




1
=> k = 0 OR
5
Thus, there is no unique answer. Insufficient

The correct answer is option A.

144. Let the present ages of Bane and his grandson be x years and y years, respectively.

Let the number of ages after which Bane would be exactly thrice as old as his grandson

be t years.

Thus: x + t = 3 y + t => x + t = 3y + 3t

x 3y
=> t = . . . (i)
2
From statement 1:

www.manhattanreview.com 19992016 Manhattan Review


Solutions 265


The ages of Bane and his grandson after 10 years would be (x + 10) years and y + 10

years, respectively.

Thus, we have:

x + 10 = 30 + 2 y + 10

=> x = 2y + 40 . . . (ii)

Since the values of x and y cannot be determined from only one equation, the answer

cannot be determined. Insufficient

From statement 2:

The ages of Bane and his grandson 10 years ago were (x 10) years and y 10 years,

respectively.

Thus, we have:

x 10 = 40 + 3 y 10

=> x = 3y + 20

x 3y = 20. . . (iii)

If we plug in this value of (iii) in (i)


x 3y 20
t= = = 10
2 2
Since we are able to find the value of t, that is 10. Sufficient

The correct answer is option B.

145. From statement 1:

p + q = 2 (r + s)

Adding (r + s) to both sides:

p + q + r + s = 3 (r + s)

=> p + q + r + s is a multiple of 3.

In order to determine whether p + q + r + s is a multiple of 12, we need to determine

whether p + q + r + s is also a multiple of 4 (since the LCM of 3 and 4 is 12).

19992016 Manhattan Review www.manhattanreview.com


266 Solutions


However, we cannot determine whether p + q + r + s is a multiple of 4.

Thus, the answer may be Yes or No. Insufficient

From statement 2:

p+r =4 q+s

Adding q + s to both sides:

p+q+r +s =5 q+s

Thus, p + q + r + s is a multiple of 5.
 
Thus, if p + q + r + s has to be a multiple of 12, then p + q + r + s should be a

multiple of the LCM of 5 and 12, i.e. 60.



However, the largest possible value of p + q + r + s = 9 + 9 + 9 + 9 = 36, which is not

a multiple of 60.

Thus, p + q + r + s cannot be a multiple of 12.

The answer to the question is No. Sufficient

The correct answer is option B.

146. Let the amounts with P, Q, R and S be $p, $q, $r and $s, respectively.

Thus: p + q + r + s = 1400 . . . (i)

From statement 1:
1
r = q
5
=> q = 5r . . . (ii)

However, the amounts with P and S are not known. Insufficient

From statement 2:

r = s + 210

=> s = r 210 . . . (iii)

However, the amounts with P and Q are not known. Insufficient

Thus, from both statements together:

www.manhattanreview.com 19992016 Manhattan Review


Solutions 267

Substituting the values of q and s from (ii) and (iii) in (i):

p + 5r + r + r 210 = 1400

=> p + 7r = 1610 . . . (iv)

From (iii), it is clear that: r 210 (since no one can have negative quantity of money)

Thus, assuming the minimum possible value of r , i.e. $210, we have, from (iv):

p + 7 210 = 1610 => p = 1610 1470 = 140

Thus, if the value of r increases further, the value of p would decrease even further.

Since p = 140 is less than r = 210, P cannot have the highest amount.

The answer to the question is No. Sufficient

The correct answer is option C.

147. From statement 1:

We know that n is DIVISIBLE by 9 = 32 and 8 = 23 .


 

Thus, the exponents of 3 and 2 in n cannot be uniquely determined.

Since n = p 2 q3 , a few scenarios are shown below:

p q n = p 2q 3 LCM of p and q

3 2 32 23 = 72 6

9 2 92 23 = 648 18

3 4 32 43 = 576 12

9 4 92 43 = 5184 36

Thus, there is no unique answer.

From statement 2:

19992016 Manhattan Review www.manhattanreview.com


268 Solutions

We have: n is a number between 400 and 500, exclusive, which can be expressed as p 2 q3 ,

where p and q are integers.

Since p and q are greater than 1 and are distinct, the minimum values of p and q are 2

and 3, not necessarily respectively.

400 500
If q = 2 => q3 = 8 (minimum), the value of p 2 lies between = 50 and =
8 8
62.5, exclusive, i.e. no such p is possible. If we try with p = 7, p 2 = 49 < 50( not

qualified), similarly, we try with p = 8, p 2 = 64 > 62.5( not qualified)


400 500
If q = 3 => q3 = 27, the value of p 2 lies between 15 and 18, exclusive,
27 27
i.e. p = 4.
400 500
If q = 4 => q3 = 64, the value of p 2 lies between 6 and 8, exclusive,
64 64
i.e. no such p is possible.
400 500
If q = 5 => q3 = 125, the value of p 2 lies between 3 and = 4, exclusive,
125 125
i.e. no such p is possible.
400 500
If q = 6 => q3 = 216, the value of p 2 lies between < 2 and < 2, i.e. no
216 216
such p is possible.

Also, no other possibilities exist, since the minimum value of p = 2 => p 2 = 4.

Thus, the only possible situation is: p = 4 and q = 3 => LCM of p and q = 12.

Sufficient

The correct answer is option B.

148. From statement 1:

We have: 22a+c + 3b = 91

Let us express 91 as sum of exponents of 2 and 3:

91 = 31 + 88 (cannot be expressed as an exponent of 2)

91 = 32 + 82 (cannot be expressed as an exponent of 2)

91 = 33 + 64 (can be expressed as 26 )

91 = 34 + 10 (cannot be expressed as an exponent of 2)

www.manhattanreview.com 19992016 Manhattan Review


Solutions 269

Since 35 exceeds 91, there are no other possibilities.

Thus, we have:

22a+c = 26 => 2a + c = 6

Since a and c are positive integers, we have: a = 1, c = 4 or a = 2, c = 2.

However, since a and c are distinct, we have: a = 1 and c = 4.

We also have:

3b = 33 => b = 3

Thus: a + b + c = 1 + 3 + 4 = 8. Sufficient

From statement 2:
c
!

We have: 2a+2b +3 4 = 131

Let us express 131 as sum of exponents of 2 and 3:

131 = 31 + 128 (can be expressed as 27 )

131 = 32 + 122 (cannot be expressed as an exponent of 2)

131 = 33 + 104 (cannot be expressed as an exponent of 2)

131 = 34 + 50 (cannot be expressed as an exponent of 2)

Since 35 exceeds 131, there are no other possibilities.

Thus, we have:

2a+2b = 27 => a + 2b = 7

Since a and b are positive integers, we have: a = 1, b = 3 or a = 3, b = 2 or a = 5, b = 1.

We also have:
c
!
c
3 4 = 31 => = 1 => c = 4
4
Thus, possible values of (a + b + c) are:

a = 1, b = 3, c = 4 => a + b + c = 8

a = 3, b = 2, c = 4 => a + b + c = 9

a = 5, b = 1, c = 4 => a + b + c = 10

19992016 Manhattan Review www.manhattanreview.com


270 Solutions

Thus, the value of (a + b + c) cannot be uniquely determined. Insufficient

The correct answer is option A.

149. Since (2a b), when divided by 9 leaves remainder 6, we have:

2a b = 9k + 6, where k is a positive integer . . . (i)

Since (a + b), when divided by 9 leaves remainder 6, we have:

a + b = 9l + 6, where l is a positive integer . . . (ii)

Adding (i) and (ii): 3a = 9 (k + l) + 12

=> a = 3 (k + l) + 4 . . . (iii)

Substituting (iii) in (ii):

{3 (k + l) + 4} + b = 9l + 6

=> b = 3 (2l k) + 2 . . . (iv)

Thus, from (iii) and (iv):

a b = {3 (k + l) + 4} {3 (2l k) + 2}

=> a b = 3 (2k l) + 2

Thus, when (a b) is divided by 3, the remainder is 2. Sufficient

From statement 2:

We only know that (a + b) is divisible by 3.

Using only this, we cannot determine the remainder when (a b) is divided by 3.

Insufficient

The correct answer is option A.

150. From statement 1:

There is no information about the time taken by any tap to fill the tank. Insufficient

From statement 2:

There is no relation among the rates of filling of the different taps. Insufficient

www.manhattanreview.com 19992016 Manhattan Review


Solutions 271

Thus, from both statements together:

We know that the rate of filling of each tap, starting from the second tap, is double the

efficiency of the previous tap.

Let the rate of filling of the first tap be 1 liter per hour.

Thus, the rates of filling of the 2nd , 3rd , 4th and 5th taps are: 2, 4, 8 and 16 liters per

hour, respectively.

Thus, total rate of filling when the first five taps are working together

= (1 + 2 + 4 + 8 + 16) liters per hour = 31 liters per hour.

Since time taken by the first five taps to fill the tank is 1 hour, total capacity of the tank

= 1 31 = 31 liters.

Since the rate of filling of the 4th tap is 8 liters per hour, time taken by the 4th tap to fill

the tank alone


31
= hours. Sufficient
8
The correct answer is option C.

19992016 Manhattan Review www.manhattanreview.com


272 Solutions

www.manhattanreview.com 19992016 Manhattan Review


Chapter 7

Talk to Us

Have a Question?
Email your questions to info@manhattanreview.com. We will be happy to answer you. Your
questions can be related to a concept, an application of a concept, an explanation of a ques-
tion, a suggestion for an alternate approach, or anything else you wish to ask regarding the
GMAT.

Please mention the page number when quoting from the book.

Best of luck!

Professor Dr. Joern Meissner


& The Manhattan Review Team

273

You are a unique candidate with unique experience.
We help you to sell your story to the admissions committee.

Manhattan Admissions is an educational consulting firm that


guides academic candidates through the complex process of
applying to the worlds top educational programs. We work
with applicants from around the world to ensure that they
represent their personal advantages and strength well and get
our clients admitted to the worlds best business schools, gra-
duate programs and colleges.

We will guide you through the whole admissions process:

Personal Assessment and School Selection


Definition of your Application Strategy
Help in Structuring your Application Essays
Unlimited Rounds of Improvement
Letter of Recommendation Advice
Interview Preparation and Mock Sessions
Scholarship Consulting

To schedule a free 30-minute consulting and candidacy evalua-


tion session or read more about our services, please visit or call:

www.manhattanadmissions.com +1.212.334.2500

About the Turbocharge Your GMAT Series (6th Edition)
The Turbocharge Your GMAT Series is carefully designed accessible manner. We delve deeply into the content of
to be clear, comprehensive, and content-driven. Long every single testing area and zero in on exactly what you
regarded as the gold standard in GMAT prep worldwide, need to know to raise your score. The full series is
Manhattan Reviews GMAT prep books offer professional comprised of 16 guides that cover concepts in mathemat-
GMAT instruction for dramatic score improvement. Now in ics and grammar from the most basic through the most
its updated 6th edition, the full series is designed to advanced levels, making them a great study resource for
provide GMAT test-takers with complete guidance for all stages of GMAT preparation. Students who work
highly successful outcomes. As many students have through all of our books benefit from a substantial boost
discovered, Manhattan Review's GMAT books break down to their GMAT knowledge and develop a thorough and
the different test sections in a coherent, concise, and strategic approach to taking the GMAT.

Turbocharge Your GMAT Series


About Manhattan Review About the Author


Manhattan Reviews origin can be traced directly back to Professor Dr. Joern Meissner has more than 25 years of
an Ivy League MBA classroom in 1999. While teaching teaching experience at the graduate and undergraduate
advanced quantitative subjects to MBAs at Columbia levels. He is the founder of Manhattan Review, a world-
Business School in New York City, Professor Dr. Joern wide leader in test prep services, and he created the origi-
Meissner developed a reputation for explaining compli- nal lectures for its first GMAT preparation classes. Prof.
cated concepts in an understandable way. Remembering Meissner is a graduate of Columbia Business School in
their own less-than-optimal experiences preparing for New York City, where he received a PhD in Management
the GMAT, Prof. Meissner's students challenged him to Science. He has since served on the faculties of prestigious
assist their friends, who were frustrated with conventional business schools in the United Kingdom and Germany. He
GMAT preparation options. In response, Prof. Meissner is a recognized authority in the areas of supply chain man-
created original lectures that focused on presenting GMAT agement, logistics, and pricing strategy. Prof. Meissner
content in a simplified and intelligible manner, a method thoroughly enjoys his research, but he believes that
vastly different from the voluminous memorization and grasping an idea is only half of the fun. Conveying knowl-
so-called tricks commonly offered by others. The new edge to others is even more fulfilling. This philosophy was
approach immediately proved highly popular with GMAT crucial to the establishment of Manhattan Review, and
students, inspiring the birth of Manhattan Review. remains its most cherished principle.

Since its founding, Manhattan Review has grown into a www.manhattanreview.com


multi-national educational services firm, focusing on info@manhattanreview.com
+1 (212) 316 -2000
GMAT preparation, MBA admissions consulting, and appli- +1 (800) 246-4600
cation advisory services, with thousands of highly satis-
Manhattan Review, 275 Madison Avenue, Suite 1429, New York, NY 10016.
fied students all over the world. The original lectures have
been continuously expanded and updated by the Manhat-
tan Review team, an enthusiastic group of master GMAT
professionals and senior academics. Our team ensures
that Manhattan Review offers the most time-efficient and
cost-effective preparation available for the GMAT. Please
visit www.ManhattanReview.com for further details.

Anda mungkin juga menyukai